Упрощение тригонометрических выражений примеры с решением 10 класс: Урок по алгебре в 10-м классе по теме «Преобразование тригонометрических выражений»

Алгебра Упрощение тригонометрических выражений

Материалы к уроку

Конспект урока

14. Упрощение тригонометрических выражений

Равенства

Часто они используются при упрощении  и доказательстве тригонометрических  выражений.

 Рассмотрим примеры использования этих формул при упрощении тригонометрических выражений.

( вынесем за скобку общий множитель косинус квадрат тэ, в скобках получим разность единицы и квадрата косинуса тэ, что равно  по первому тождеству квадрату синуса тэ.  Получим  сумму синус четвертой степени тэ произведения косинус квадрат тэ и синус квадрат тэ.  общий множитель синус квадрат тэ вынесем за скобки, в скобках получим сумму квадратов косинуса и синуса, что по основному тригонометрическому  тождеству равно единице. В итоге получим квадрат  синуса тэ).

( Вынесем общий множитель косинус тэ за скобки, а в скобках приведем к общему знаменателю, который представляет собой произведение один минус синус тэ на один плюс синус тэ.

В числителе получим:  единица плюс синус тэ плюс единица минус синус тэ, приводим подобные, числитель равен  двум после приведения подобных.

В знаменателе можно применить формулу сокращенного умножения (разность квадратов) и получить разность единицы и квадрата синуса тэ, что по основному тригонометрическому тождеству 

 равно квадрату косинуса тэ. После сокращения на косинус тэ получим конечный ответ : два деленное на косинус тэ).

 

Рассмотрим примеры использования этих формул при доказательстве тригонометрических выражений.

ПРИМЕР 4.Найти значение выражения tg 2 t + ctg 2 t ,если  tg t + ctg t = 6.

( сумма квадратов тангенса тэ и котангенса тэ, если сумма тангенса и котангенса равна шести).

Решение. (tg t + ctg t)2 = 62

tg 2 t + 2 ∙ tg t ∙ctg t + ctg 2 t = 36

tg 2 t + 2 + ctg 2 t = 36

tg 2 t + ctg 2 t = 36-2

tg 2 t + ctg 2 t = 34

Возведем обе части исходного равенства в квадрат:

(tg t + ctg t)2 = 62 ( квадрат суммы тангенса тэ и котангенса тэ равна шести в квадрате).   Вспомним формулу сокращённого умножения: Квадрат суммы двух величин равен квадрату первой плюс удвоенное произведение первой на вторую плюс квадрат второй. (a+b)2=a2+2ab+b2    Получим tg 2 t + 2 ∙ tg t ∙ctg t + ctg 2 t = 36 (тангенс квадрат тэ плюс удвоенное произведение тангенса тэ на котангенс тэ плюс котангенс квадрат тэ равно тридцати шести).

Так как произведение тангенса тэ на котангенс тэ равно единице, то  tg 2 t + 2 + ctg 2 t = 36 ( сумма квадратов тангенса тэ и котангенса тэ и двух равна тридцати шести),

 значит  tg 2 t + ctg 2 t = 34 (сумма квадратов тангенса тэ и котангенса тэ равна тридцати четырем).  Ответ: 34.

Остались вопросы по теме? Наши репетиторы готовы помочь!

  • Подготовим к ЕГЭ, ОГЭ и другим экзаменам

  • Найдём слабые места по предмету и разберём ошибки

  • Повысим успеваемость по школьным предметам

  • Поможем подготовиться к поступлению в любой ВУЗ

Выбрать репетитораОставить заявку на подбор

Упрощение тригонометрических выражений по основным формулам тригонометрии.

Часть 14

12+

6 месяцев назад

Математика от Баканчиковой296 подписчиков

Геометрия 9 класс. Как применять основные формулы тригонометрии для упрощения тригонометрических выражений в 9 и 10 классах? Сегодня мы ответим на этот вопрос. Если Вы не видели наши предыдущие уроки по теме: «Тригонометрические функции в геометрии», то обязательно посмотрите их, тогда этот урок будет Вам очень понятен. Вы обратим Ваше внимание на то, что чтобы легко упрощать тригонометрические выражения, Вы должны чётко знать 20 основных формул тригонометрии и практически все темы алгебры, включая действия с многочленами, дробями, формулы сокращенного умножения многочленов, законы математики, приведение подобных слагаемых и т.п. А чтобы Вы закрепили и запомнили материал этого урока, мы предлагаем Вам в качестве домашнего задания самостоятельно решить те упражнения, решение которых было показано на этом уроке, а потом сверить получившиеся ответы. Подробный план урока и ссылки на предыдущие уроки Вы можете найти в описании под видео. 00:00 Начало видео. 00:22 Упражнение 1. 03:00 Упражнение 2. 07:06 Упражнение 3. 12:44 Упражнение 4. 14:25 Упражнение 5. 16:42 Упражнение 6. 20:24 Тема следующего урока. Если Вы впервые на нашем канале или не смотрели наши предыдущие уроки, то рекомендуем Вам посмотреть следующие видео: Тригонометрические функции в геометрии. Что это такое. Сколько тригонометрических функций и почему. Геометрия 9 класс. Часть 1. https://rutube.ru/video/b99256c0e2a5f1411c87731142e2a822/ Как запомнить формулы тригонометрических функций. Стандартные обозначения этих функций, треугольника и длин его сторон. Тригонометрические функции в геометрии. Часть 2. Геометрия 9 класс. https://rutube.ru/video/cb235fc7ef53f468f18b151435d18c77/ Как найти sin, cos, tg и ctg угла по двум сторонам треугольника. Как построить угол по sin, cos или tg. Тригонометрические функции в геометрии. Часть 3. Геометрия 9 класс. https://rutube.ru/video/3c8642f0072caa41866cb44fe5cf1eb2/ Как найти значение тригонометрических функций тремя способами. Тригонометрические функции в геометрии. Часть 4. Геометрия 9 класс. https://rutube.ru/video/70f16a0f13b974194b59d3327a03a403/ Как найти значения sin, cos, tg и ctg для углов в 30°, 45° и 60°. Тригонометрические функции в геометрии. Часть 5. Геометрия 9 класс. https://rutube.ru/video/9393cb6043b9878b49883195db02d251/ Как найти противо- и прилежащие катеты по углу и гипотенузе. Как найти площадь треугольника, параллелограмма и трапеции. Тригонометрические функции. Часть 6. Геометрия 9 класс. https://rutube.ru/video/01cfdff903c6c37227f510c5f4bf7984/ Как найти объем пирамиды (конуса) по боковому ребру (образующей) и углу между боковым ребром (образующей) и плоскостью основания. Тригонометрические функции. Часть 7. Геометрия 10-11 класс. https://rutube.ru/video/a246217e2de6f5960fb55a3e77b904d0/ Как найти противо- и прилежащие катеты по углу и другому катету. Как найти площадь треугольника и параллелограмма. Тригонометрические функции. Часть 8. Геометрия 9 класс. https://rutube.ru/video/5b374b4961980532582185a1803b6be0/ Тангенс и котангенс в решении задач по стереометрии. Тригонометрические функции. Часть 9. Геометрия 10-11 класс. https://rutube.ru/video/22ce4c90678e252c22f2084eb1ec14a3/ Как найти гипотенузу по катету и противолежащему или прилежащему углу. Тригонометрические функции. Часть 10. Геометрия 9 класс. https://rutube.ru/video/d4f247808a953357f1ed1c2a66e6fe73/ Как найти значения длин сторон прямоугольного треугольника по значениям тригонометрических функций. Тригонометрические функции. Часть 11. Геометрия 9 класс. https://rutube.ru/video/3f37a5cab19ba4a197de56e09b20237f/ Основное тригонометрическое тождество. Упрощение тригонометрических выражений. Тригонометрия. Геометрия 9 класс. Алгебра 10 класс. Часть 12. https://rutube.ru/video/458402dea2fc63846652e01cf35d69f3/ Основные тригонометрические формулы синуса, косинуса, тангенса и котангенса. Тригонометрия 9 класс. Часть 13. https://youtu.be/EUVFgJNnH0I #УпрощениеТригонометрическихВыражений #ОсновноеТригонометрическоеТождествоПримеры #напишиданномутригонометрическомувыражениютождественноевыражение #тригонометрическиевыражения9класс #тригонометрическиевыражения10класс #преобразованиетригонометрическихвыражений #упроститьтригонометрическоевыражение #МатематикаОтБаканчиковой упрощение тригонометрических выражений, преобразование тригонометрических выражений, упростить тригонометрическое выражение, тригонометрические выражения 9 класс, тригонометрические выражения 10 класс, тригонометрические выражения, тригонометрические формулы 9 класс, тригонометрические формулы 10 класс, напиши данному тригонометрическому выражению тождественное выражение

Объяснение урока: Упрощение тригонометрических выражений

В этом объяснении мы узнаем, как упростить тригонометрическое выражение.

Эти выражения часто упрощаются при применении одного или нескольких тригонометрических тождеств, которые связывают различные тригонометрические и обратные тригонометрические функции и их аргументы. Их мотивация математическая, но они также приложения в реальных задачах.

Тригонометрические тождества имеют несколько реальных применений в различных областях, таких как физика, инженерия, архитектура, робототехника, теория музыки и навигация, и это лишь некоторые из них. В физике их можно использовать в движении снарядов, моделируя механику электромагнитных волн, анализируя переменные и постоянные токи и находя траекторию движения массы вокруг массивного тела под сила тяжести.

Начнем с напоминания о тригонометрических функциях, пифагорейские тождества которых мы рассмотрим в этом толкователе. Учитывать следующий прямоугольный треугольник.

Тригонометрические функции могут быть выражены через отношение сторон треугольника как sinOHcosAHtanOA𝜃=,𝜃=,𝜃=.

Эти функции удовлетворяют следующему тригонометрическому тождеству: тансинкос𝜃=𝜃𝜃.

Отметим, что эти тригонометрические соотношения определены для острых углов 0𝜃90∘∘, а тригонометрические функции для всех значений 𝜃 определены на единичной окружности.

Предположим, что точка движется по единичной окружности против часовой стрелки. В определенной позиции (𝑥,𝑦) на единичной окружности с углом 𝜃 функция синуса определяется как 𝑦=𝜃sin и функция косинуса как 𝑥=𝜃cos, как показано на диаграмме выше. Другими словами, тригонометрические функции определяются с помощью координат точки пересечения единичной окружности. с конечной стороной 𝜃 в стандартном положении.

Взаимные тригонометрические уравнения определяются в терминах стандартных тригонометрических уравнений следующим образом.

Определение: обратные тригонометрические функции

Функции косеканса, секанса и котангенса определяются как cscsinseccoscottancossin𝜃=1𝜃,𝜃=1𝜃,𝜃=1𝜃=𝜃𝜃.

Тригонометрические функции являются периодическими, что означает, что если мы добавим целое число, кратное 2𝜋, в радианы или 360∘ на угол 𝜃, значение функции остается прежней: sincoscostantan(360+𝜃)=𝜃,(360+𝜃)=𝜃,(360+𝜃)=𝜃.∘∘∘

Мы можем видеть это непосредственно из определения единичного круга тригонометрических функций. На самом деле, касательная функция периодическая по 𝜋, в радианах, или 180∘, так как у нас есть tantan(180+𝜃)=𝜃.∘

Аналогично, для обратных тригонометрических функций имеем csccscsecseccotcot(360+𝜃)=𝜃,(360+𝜃)=𝜃,(360+𝜃)=𝜃.∘∘∘

Подобно функции тангенса, функция котангенса периодична по 𝜋, в радианы или 180∘, так как у нас есть коткот(180+𝜃)=𝜃.∘

Тригонометрические тождества, которые мы рассмотрим в этом объяснении, выполняются для любого угла 𝜃 в области функции в градусах или радианы. В частности, мы можем преобразовать угол между степени и радианах по следующему правилу: если у нас есть угол 𝜃степень, мы можем преобразовать его в радианы через 𝜃=𝜋180𝜃. radiansdegree

При работе с тригонометрическими выражениями полезно переписать обратные тригонометрические тождества в терминах синуса и косинус для упрощения.

Рассмотрим пример, в котором мы должны использовать обратные тригонометрические функции для определения значения тригонометрическое выражение.

Пример 1. Использование взаимных тождеств для вычисления тригонометрических выражений

Найдите значение 8𝜃×−5𝜃sincsc.

Ответ

В этом примере мы хотим найти значение конкретного выражения, включающего тригонометрические и обратные числа. тригонометрические функции.

Один из способов вычисления тригонометрического выражения состоит в том, чтобы записать его в терминах функций синуса и косинуса, используя следующие определение функции косеканса, входящей в данное выражение: cscsin𝜃=1𝜃.

Следовательно, выражение можно упростить, чтобы дать 8𝜃×−5𝜃=8𝜃×−5=8𝜃×−5𝜃=−40×𝜃𝜃=−40.sincscsinsinsinsinsinsin

Теперь давайте рассмотрим пример, в котором мы упрощаем конкретное тригонометрическое выражение.

Пример 2. Упрощение тригонометрических выражений с использованием тригонометрических тождеств

Упростить coscscsin𝜃𝜃𝜃.

Ответ

В этом примере мы хотим упростить конкретное выражение, включающее тригонометрические и обратные тригонометрические функции.

Один из способов упростить тригонометрическое выражение — записать его в терминах функций синуса и косинуса, используя следующие определение функции косеканса, входящей в данное выражение: cscsin𝜃=1𝜃.

Данное тригонометрическое выражение становится coscscsincossinsincos𝜃𝜃𝜃=𝜃×1𝜃×𝜃=𝜃.

В следующем примере мы упростим тригонометрическое выражение, записав его с помощью функций синуса и косинуса.

Пример 3. Упрощение тригонометрических выражений с использованием тригонометрических тождеств

Упрощение tansinsec𝜃𝜃𝜃.

Ответ

В этом примере мы хотим упростить конкретное выражение, включающее тригонометрические и обратные тригонометрические функции.

Один из способов упростить тригонометрическое выражение — записать его в терминах функций синуса и косинуса, используя следующие определения для функций тангенса и секанса, которые появляются в данном выражении: тансинкоссеккос𝜃=𝜃𝜃𝜃=1𝜃.

Данное тригонометрическое выражение становится tansinsectansincossincossincossincossin𝜃𝜃𝜃=𝜃𝜃÷1𝜃=𝜃𝜃×𝜃𝜃=𝜃×𝜃𝜃=𝜃.

В следующем примере используется произведение тригонометрических и обратных тригонометрических функций, которое мы можем просто использовать, используя определение обратных функций, а затем переписать окончательный ответ в терминах другой обратной функции.

Пример 4. Упрощение тригонометрических выражений с использованием взаимных тождеств

Simplify cosseccsc𝜃𝜃𝜃.

Ответ

В этом примере мы хотим упростить конкретное выражение, включающее тригонометрические и обратные тригонометрические функции.

Один из способов упростить тригонометрическое выражение — записать его в терминах функций синуса и косинуса, используя следующие определения функций косеканса и секанса, входящих в данное выражение: cscsinseccos𝜃=1𝜃,𝜃=1𝜃.

Таким образом, выражение можно упростить как cosseccsccoscossincossin𝜃𝜃𝜃=𝜃×1𝜃×1𝜃=𝜃𝜃.

Теперь, используя определение функции котангенса, коткосин𝜃=𝜃𝜃.

Данное выражение может быть выражено через функцию котангенса как cossecccscossincot𝜃𝜃𝜃=𝜃𝜃=𝜃.

Тригонометрические и обратные тригонометрические функции являются четными и нечетными функциями, поскольку они удовлетворяют свойствам 𝑓(−𝜃)=𝑓(𝜃) для четных функций и 𝑓(−𝜃)=−𝑓(𝜃) для нечетных функций. В частности, функция синуса нечетная, а функция косинуса четно, так как они удовлетворяют sinsincoscos(−𝜃)=−𝜃,(−𝜃)=𝜃, для любого значения 𝜃 в градусах или радианы. Отсюда мы также можем определить четность других тригонометрические функции, которые определяются с их точки зрения. В частности, для касательной функции имеем tansincossincostan(-𝜃)=(-𝜃)(-𝜃)=-𝜃𝜃=-𝜃.

Таким образом, функция тангенса нечетна, и мы можем вывести четность других тригонометрических функций аналогичным образом. Мы можем обобщить их следующим образом.

Четные и нечетные тождества для тригонометрических функций

Функции косинуса и секанса четны, что означает, что для любого значения 𝜃 в соответствующих областях определения они удовлетворяют тождества coscossecsec(−𝜃)=𝜃,(−𝜃)=𝜃.

А функции синуса, тангенса, косеканса и котангенса нечетны, что означает, что они удовлетворяют следующим тождествам для любого значение 𝜃 в соответствующих доменах: sinsintantancsccsccotcot(-𝜃)=-𝜃,(-𝜃)=-𝜃,(-𝜃)=-𝜃,(-𝜃)=-𝜃.

Теперь давайте рассмотрим пример, в котором мы должны применить четность тригонометрической функции, чтобы просто определить конкретную тригонометрическое выражение.

Пример 5. Упрощение тригонометрических выражений, включающих нечетные и четные тождества

Упростить tancsc(−𝜃)𝜃.

Ответ

В этом примере мы хотим упростить конкретное выражение, включающее тригонометрические и обратные тригонометрические функции используя четную/нечетную идентичность.

Один из способов упростить тригонометрическое выражение — записать его в терминах функций синуса и косинуса, используя следующие определение функции косеканса, входящей в данное выражение: cscsin𝜃=1𝜃.

Касательная функция нечетная, поэтому тождество тантан(-𝜃)=-𝜃.

Мы можем переписать функцию тангенса, используя ее определение в терминах функций синуса и косинуса: тансинкос𝜃=𝜃𝜃.

Таким образом, выражение можно упростить как tancsctancscsincossincos(−𝜃)𝜃=−𝜃𝜃=−𝜃𝜃×1𝜃=−1𝜃.

Наконец, мы можем переписать это выражение в терминах функции секущей, определяемой как секкос𝜃=1𝜃.

Таким образом, выражение принимает вид tancscsec(−𝜃)𝜃=−𝜃.

Функция синуса эквивалентна функции косинуса смещением на 90∘ влево, что можно визуализировать, сравнив график обеих функций.

В частности, для углов 𝜃 и 90+𝜃∘: sincoscossin(90+𝜃)=𝜃,(90+𝜃)=−𝜃.∘∘

Мы также можем проиллюстрировать их на единичной окружности, как показано.

Аналогично, заменяя 𝜃 на −𝜃, мы получаем следующие тождества кофункций для дополнительных углов 𝜃 и 90−𝜃∘: sincoscossin(90−𝜃)=𝜃,(90−𝜃)=𝜃.∘∘

Мы можем проиллюстрировать это, как показано.

На рисунке изображен прямоугольный треугольник с углом 𝐴𝑂𝐵 в стандартном положении, который пересекает единичную окружность в точке 𝐵(𝑥,𝑦) и имеет остроугольную меру 0𝜃90∘∘.

Мы можем комбинировать эти тождества и использовать их для определения тождеств для других тригонометрических функций, определенных в функции синуса и косинуса.

Определение: тригонометрические тождества коррелированных углов

Тригонометрические функции удовлетворяют тождествам кофункций для всех 𝜃 в своих областях определения. В частности, у нас есть sincoscossintancottancscsecseccsc(90±𝜃)=𝜃,(90±𝜃)=∓𝜃,(90±𝜃)=∓𝜃,(90±𝜃)=∓𝜃,(90±𝜃)=𝜃,(90±𝜃)= ∓𝜃.∘∘∘∘∘∘

Например, для функции касательной имеем tansincossincossincot(90±𝜃)=(90±𝜃)(90±𝜃)=𝜃∓𝜃=∓𝜃𝜃=∓𝜃. ∘∘∘

Все эти тождества также выполняются в радианах, в частности, заменив 90∘ на 𝜋2 в радианы.

Теперь давайте рассмотрим пример, где мы используем это тождество вместе с четностью тригонометрической функции, чтобы упростить выражение.

Пример 6. Упрощение тригонометрических выражений с использованием коррелированных и четных тождеств

Упростить sinsec𝜋2+𝜃(−𝜃).

Ответ

В этом примере мы хотим упростить конкретное выражение, включающее обратные тригонометрические функции.

Мы также будем использовать тождество коррелированного угла синкос𝜋2+𝜃=𝜃 и даже тождество сексек(−𝜃)=𝜃.

Один из способов упростить тригонометрическое выражение — записать его в терминах функций синуса и косинуса, используя следующее определение функции секущей: секкос𝜃=1𝜃.

Используя их, выражение становится sinseccosseccoscos𝜋2+𝜃(−𝜃)=𝜃𝜃=𝜃×1𝜃=1.

Теперь предположим, что мы хотим определить sin(180−𝜃)∘. Мы можем найти это, многократно используя приведенные выше тождества. Если мы позволим 𝜃=90−𝑥∘, тогда sinsinsincos(180−𝜃)=(180−[90−𝑥])=(90+𝑥)=𝑥.∘∘∘∘

Теперь, подставляя обратно 𝑥=90−𝜃, получаем sincossin(180−𝜃)=(90−𝜃)=𝜃.∘∘

Аналогично находим coscos(180−𝜃)=−𝜃.∘

Повторно применяя эти тождества или используя единичную окружность, мы также получаем тождества для углов 𝜃 и 𝜃±180∘: sinsincoscos(180±𝜃)=∓𝜃,(180±𝜃)=−𝜃.∘∘

Для 𝜃 и 180−𝜃∘, имеем следующее.

А для 𝜃 и 180+𝜃∘ имеем следующее.

У нас также есть тождества для других тригонометрических функций, которые следуют из тождеств для функций синуса и косинуса, из их определения: tantancotcotcsccscsecsec(180±𝜃)=±𝜃,(180±𝜃)=±𝜃,(180±𝜃)=∓𝜃,(180±𝜃)=-𝜃.∘∘∘∘

В следующем примере используются определения обратных тригонометрических функций вместе с тождествами кофункций в радианах, чтобы упростить выражение.

Пример 7. Использование периодических тождеств и тождеств кофункций для упрощения тригонометрического выражения

Simplify seccot−𝜃(𝜋−𝜃).

Ответ

В этом примере мы хотим упростить конкретное выражение, включающее обратные тригонометрические функции.

Мы также будем использовать кофункцию и коррелированные тождества: seccsccotcot𝜋2−𝜃=𝜃,(𝜋−𝜃)=−𝜃.

Один из способов упростить тригонометрическое выражение — записать его в терминах функций синуса и косинуса, используя следующие определения для функций косеканса и котангенса, которые появляются в числителе и знаменателе: cscsincotcossin𝜃=1𝜃,𝜃=𝜃𝜃.

Числитель выражения можно упростить как secscssin𝜋2−𝜃=𝜃=1𝜃.

И знаменатель как cotcotcossin(𝜋−𝜃)=−𝜃=−𝜃𝜃.

Таким образом, выражение можно упростить как seccotsinsincoscos−𝜃(𝜋−𝜃)=−1𝜃×𝜃𝜃=−1𝜃.

Наконец, мы можем переписать это выражение в терминах функции секущей, определяемой как секкос𝜃=1𝜃.

Таким образом, получаем seccotsec−𝜃(𝜋−𝜃)=−𝜃.

Аналогично, для углов 𝜃 и 270±𝜃∘ имеем sincoscossintancotcottancscsecseccsc(270±𝜃)=−𝜃,(270±𝜃)=±𝜃. (270±𝜃)=∓𝜃,(270±𝜃)=∓𝜃,(270±𝜃)=-𝜃,(270±𝜃 )=±𝜃.∘∘∘∘∘∘

Это можно представить следующим образом.

Используя периодичность тригонометрических функций и единичный круг, мы имеем sinsincoscostantancotcsccscsecsec(360±𝜃)=±𝜃,(360±𝜃)=𝜃,(360±𝜃)=±𝜃,(360±𝜃)=±𝜃,(360±𝜃)=±𝜃,(360±𝜃) =𝜃.∘∘∘∘∘∘

Все тождества также выполняются в радианах, заменив 360∘ на 2𝜋 в радианах. Их также можно визуализировать с помощью единичного круга, как показано на рисунке.

Все тождества с коррелированными углами можно визуализировать, используя следующее.

Теперь давайте рассмотрим несколько примеров, где мы должны применить тождества кофункций, чтобы упростить тригонометрическое выражение. В следующем примере мы будем многократно использовать это тождество для функций косинуса и синуса, в градусов.

Пример 8. Упрощение тригонометрических выражений с использованием тождеств кофункций

Упростить sincos𝜃+(270+𝜃)∘.

Ответ

В этом примере мы хотим упростить конкретное выражение, включающее тригонометрические функции.

Чтобы упростить данное выражение, мы используем тождество коррелированного угла cossin(270+𝜃)=𝜃.∘

Следовательно, имеем sincossinsinsin𝜃+(270+𝜃)=𝜃+𝜃=2𝜃.∘

В последнем примере мы хотим повторно применить тождества кофункций к функциям тангенса и котангенса, в градусов, чтобы упростить тригонометрическое выражение.

Пример 9. Использование тригонометрических тождеств для упрощения тригонометрического выражения

Упростить сектанту𝜃𝜃(270+𝜃)∘.

Ответ

В этом примере мы хотим упростить конкретное выражение, включающее тригонометрические и обратные тригонометрические функции.

Мы также будем использовать тождество коррелированного угла tancot(270+𝜃)=−𝜃.∘

Поскольку по определению функции котангенса мы имеем хлопок𝜃=1𝜃, коррелированное тождество можно записать в терминах касательной функции как tancottan(270+𝜃)=−𝜃=−1𝜃.∘

Следовательно, выражение можно упростить как сектантансектансектансек𝜃𝜃(270+𝜃)=𝜃𝜃×−1𝜃=−𝜃×𝜃𝜃=−𝜃.

Давайте закончим повторением нескольких важных ключевых моментов из этого объяснения.

Ключевые моменты

  • Мы можем выразить тангенс и обратные тригонометрические функции через синус и косинус как tansincoscsscsinseccoscottancossin𝜃=𝜃𝜃,𝜃=1𝜃,𝜃=1𝜃,𝜃=1𝜃=𝜃𝜃. и мы можем использовать их для упрощения тригонометрических выражений.
  • Все эти тригонометрические функции либо четные, либо нечетные. В частности, для функций синуса и косинуса имеем коскоссин(-𝜃)=𝜃,(-𝜃)=-𝜃, и аналогично для других тригонометрических функций, следующих из определений. Мы можем использовать четность тригонометрические функции, которые помогут нам упростить тригонометрические выражения.
  • Единичный круг позволяет нам определить тождества коррелированных углов для синуса и косинуса.
    Например, тождества кофункций (в радианах): sincoscossin𝜋2−𝜃=𝜃,𝜋2−𝜃=𝜃. Соответствующие тождества для касательной и обратной тригонометрических функций находятся по их определениям. через функции синуса и косинуса.
  • Нам часто приходится применять более одного тождества или типа тождества, чтобы упростить тригонометрическое выражение.

Тригонометрические тождества — Все тригонометрические тождества с доказательствами

Тригонометрические тождества являются фундаментальным аспектом тригонометрии, изучающей отношения между углами и сторонами треугольников. Эти тождества представляют собой математические уравнения, которые включают тригонометрические функции, такие как синус, косинус и тангенс, и верны для всех значений задействованных переменных.

Тригонометрические тождества полезны для упрощения выражений, решения уравнений и доказательства математических теорем в различных областях науки и техники. Понимание свойств и приложений этих тождеств важно для студентов и специалистов в таких областях, как математика, физика и инженерия.

1. Что такое тригонометрические тождества?
2. Взаимные тождества
3. Пифагорейские тригонометрические тождества
4. Дополнительные и дополнительные идентификаторы
5. Тождества суммы и разности
6. ​​ Периодические тождества
7. Двойные и половинчатые удостоверения
8. Трёхугольные удостоверения
9. Идентичности суммы и произведения
10. Правило синуса и косинуса
11. Часто задаваемые вопросы о тригонометрических тождествах

Что такое тригонометрические тождества?

Тригонометрические тождества являются уравнениями, содержащими тригонометрические функции, и справедливы для любого значения вовлеченных переменных при условии, что обе части равенства определены. Эти уравнения верны для любого значения переменной, которая есть в области.

Тригонометрические тождества связывают 6 тригонометрических функций: синус, косинус, тангенс, косеканс, секанс и котангенс. Давайте узнаем обо всех тригонометрических тождествах подробно, которые упомянуты ниже.

  • Взаимные тождества
  • Пифагорейские тождества
  • Тождества с противоположными углами
  • Дополнительные тождества углов
  • Дополнительные удостоверения углов
  • Тождества суммы и разности
  • Периодические удостоверения
  • Идентификаторы с двойным и половинным углом
  • Трёхугольные удостоверения
  • Суммировать идентификаторы продуктов
  • Продукт для суммирования идентификаторов
  • Закон синусов и закон косинусов

Взаимные тождества

Мы уже знаем, что обратные величины синуса, косинуса и тангенса равны косекансу, секансу и котангенсу соответственно.

Таким образом, взаимные тождества задаются как

  • sin θ = 1/cosec θ (OR) cosec θ = 1/sin θ
  • cos θ = 1/с θ (ИЛИ) с θ = 1/cos θ
  • tan θ = 1/кот θ (OR) кроватка θ = 1/tan θ

Пифагорейские тригонометрические тождества

Тригонометрические тождества Пифагора в тригонометрии выводятся из теоремы Пифагора. Ниже приведены 3 пифагорейских тригонометрических тождества.

  • sin 2 θ + cos 2 θ = 1. Это также можно записать как 1 — sin 2 θ = cos 2 θ ⇒ 1 — 9 3 sin 9 2 90 θ
  • сек 2 θ — загар 2 θ = 1. Это также можно записать как сек 2 θ = 1 + тангенс 2 θ ⇒ сек 2 θ — 1 = тангенс 2 θ
  • csc 2 θ — детская кроватка 2 θ = 1. Это также может быть записано как csc 2 θ = 1 + детская кроватка 2 θ ⇒ csc 2 θ 9 039 детская кроватка

Давайте посмотрим, как доказать эти тождества.

3 Доказательство тождества Пифагора

Рассмотрим прямоугольный треугольник ABC с прямым углом в точке B, как показано ниже.

Применяя к этому треугольнику теорему Пифагора, получаем

Противоположная 2 + Прилежащая 2 = Гипотенуза 2 … (1)

Разделив обе стороны на гипотенузу 26 395 2 / Гипотенуза 2 + Смежное 2 / Гипотенуза 2 = Гипотенуза 2 / Гипотенуза 2

Используя определения тригонометрических соотношений, приведенное выше уравнение принимает вид

    395 2 θ + cos 2 θ = 1

Это одно из пифагорейских тождеств. Таким же образом мы можем вывести два других пифагорейских тригонометрических тождества.

  • tan 2 θ + 1 = сек 2 θ (это можно получить, разделив обе части (1) на «Смежные 2 »)
  • 1 + кроватка 2 θ = cosec 2 θ (это можно получить, разделив обе части (1) на «Противоположные 2 »)

Дополнительные и дополнительные идентификаторы

Дополнительные углы — это пара двух углов, сумма которых равна 90°. Дополнение угла θ равно (90 — θ). Тригонометрические отношения дополнительных углов (также известные как тождества кофункций):

  • sin (90°-θ) = cos θ
  • cos (90°- θ) = sin θ
  • загар (90°- θ) = детская кроватка θ
  • косек (90°-θ) = сек θ
  • сек (90°- θ) = cosec θ
  • раскладушка (90°- θ) = загар θ

Дополнительные углы представляют собой пару двух углов, сумма которых равна 180°. Дополнение угла θ равно (180 — θ). Тригонометрические отношения дополнительных углов:

  • sin (180°-θ) = sinθ
  • cos (180°- θ) = -cos θ
  • тангенс (180°- θ) = -тангенс θ
  • косек (180°-θ) = косек θ
  • сек (180°- θ)= -сек θ
  • раскладушка (180°- θ) = -раскладушка θ

Тождества суммы и разности

Тождества суммы и разности включают формулы sin(A+B), cos(A-B), tan(A+B) и т. д.

  • sin (A+B) = sin A cos B + cos A sin Б
  • sin (A-B) = sin A cos B — cos A sin B
  • cos (A+B) = cos A cos B — sin A sin B
  • cos (A-B) = cos A cos B + sin A sin B
  • тангенс (A+B) = (тангенс A + тангенс B)/(1 — тангенс A тангенс B)
  • загар (AB) = (загар A — загар B)/(1 + загар A загар B)

Периодические тождества

Периодические тождества в тригонометрии представляют собой набор тождеств, описывающих периодический характер тригонометрических функций. Периодическая функция — это функция, которая повторяет свои значения через определенный интервал, известный как ее период. Вот периодические тождества sin, cos и tan.

  • грех(х + 2π) = грех(х)
  • потому что (х + 2π) = потому что (х)
  • тангенс (х + π) = тангенс (х)

Мы можем попытаться вывести их либо с помощью единичного круга, либо с помощью вышеупомянутых тождеств суммы и разности.

Тождества с двойными и половинными углами

Формулы двойного угла: Тригонометрические тождества двойного угла можно получить, используя формулы суммы и разности.

Например, из приведенных выше формул:

sin (A+B) = sin A cos B + cos A sin B

Подставим здесь A = B = θ с обеих сторон, получим:

sin (θ + θ) = sinθ cosθ + cosθ sinθ
sin 2θ = 2 sinθ cosθ

Таким же образом мы можем вывести другие тождества двойного угла.

  • sin 2θ = 2 sinθ cosθ
  • cos 2θ = cos2θ — sin 2θ
    = 2 cos 2 θ — 1
    = 1 — 2sin 2 θ
  • тангенс 2θ = (2тангенс θ)/(1 — тангенс 2 θ)

Формулы половинного угла: Используя одну из приведенных выше формул двойного угла,

cos 2θ = 1 — 2 sin 2 θ

2 sin 2 θ = 1- cos 2θ

sin 2 θ = (1 — cos2θ)/(2)

sin θ = ±√ 2[(1 — cos θ) 2[(1 — cos 2θ) ]

Замена θ на θ/2 с обеих сторон,

sin (θ/2) = ±√[(1 — cos θ)/2]

Это формула половинного угла sin.

Таким же образом можно вывести и другие формулы половинного угла.

  • sin (θ/2) = ±√[(1 — cos θ)/2]
  • cos (θ/2) = ±√(1 + cos θ)/2
  • тангенс (θ/2) = ±√[(1 — cos θ)(1 + cos θ)]

Трехугольные тождества

Тождества тройного угла — это тригонометрические тождества, связывающие значения тригонометрических функций трехкратного угла со значениями тригонометрических функций самого угла. Формула тройного угла для синуса может быть получена следующим образом.

Мы можем записать sin 3x как:

sin (3x) = sin (2x + x)

= sin 2x cos x + cos 2x sin x

Используя формулу двойного угла для синуса,

sin 3x = 2 sin x cos x cos x + cos 2x sin x

Теперь, используя тождество Пифагора и формулу двойного угла для cos,

= 2 sin x (1 — sin 2 x) + ( 1 — 2sin 2 x) sin x

= 2 sin x — 2 sin 3 x + sin x — 2 sin 3 x

= 3 sin x — 4 sin 3 x

3 9 таким образом, мы можем вывести и другие формулы тройного угла.

  • sin(3x) = 3sin(x) — 4sin 3 (x)
  • cos(3x) = 4cos 3 (x) — 3 cos x
  • tan(3x) = (3 tan x — tan 3 x)/(1 — 3tan 2 x)

Идентичности суммы и произведения

Эти тождества используются либо для преобразования «суммы в произведение» или «произведения в сумму» в случае тригонометрических функций.

Сумма тождеств продукта: Эти тождества равны

  • sin A + sin B = 2[sin((A + B)/2)cos((A − B)/2)]
  • грех А — грех В = 2 [cos ((А + В) / 2) грех ((А — В) / 2)]
  • потому что А + потому что В = 2 [потому что ((А + В) / 2) потому что ((А — В) / 2)]
  • потому что A — потому что B = -2[sin((A + B)/2)sin((A — B)/2)]

Произведение на сумму тождеств: Эти тождества таковы:

  • sin A⋅cos B = [sin(A + B) + sin(A − B)]/2
  • потому что A⋅cos B = [cos(A + B) + cos(A − B)]/2
  • sin A⋅sin B = [cos(A − B) − cos(A + B)]/2

Правило синусов и косинусов

Тригонометрические тождества, которые мы узнали, выводятся с помощью прямоугольных треугольников. Есть несколько других тождеств, которые мы используем в случае непрямоугольных треугольников.

Правило синусов: Правило синусов определяет соотношение между углами и соответствующими сторонами треугольника. Для непрямоугольных треугольников нам придется использовать правило синусов и правило косинусов. Для треугольника со сторонами «a», «b» и «c» и соответствующими противоположными углами, равными A, B и C, правило синусов может быть задано как 9.0003

  • а/sinA = b/sinB = c/sinC
  • sinA/a = sinB/b = sinC/c
  • а/б = sinA/sinB; а/с = sinA/sinC; б/с = sinB/sinC

Правило косинуса: Правило косинуса дает отношение между углами и сторонами треугольника и обычно используется, когда даны две стороны и угол между ними. Правило косинуса для треугольника со сторонами «a», «b» и «c» и соответствующими противоположными углами A, B и C, правило синуса может быть указано как,

  • а 2 = b 2 + с 2 — 2bc·cosA
  • b 2 = c 2 + a 2 — 2ca·cosB
  • с 2 = а 2 + b 2 — 2ab·cosC

Важные замечания по тригонометрическим тождествам:

  • Чтобы записать тригонометрические отношения дополнительных углов, мы рассмотрим следующие пары: (sin, cos), (cosec, sec) и (tan, cot).
  • При записи тригонометрических отношений дополнительных углов тригонометрическое соотношение не изменится. Знак можно определить, используя тот факт, что только sin и cosec положительны во втором квадранте, где угол имеет форму (180-θ).
  • Есть 3 формулы для формулы cos 2x. Вы можете запомнить только первое, потому что два других можно получить с помощью тождества Пифагора sin 2 x + cos 2 x = 1.
  • Формула половинного угла для tan получается путем применения тождества tan = sin/cos и последующего использования формул половинного угла для sin и cos.

Связанные темы:

  • Тригонометрические тождества для класса 12
  • Тригонометрические тождества Класс 11
  • Тригонометрические тождества Класс 10

 

Примеры тригонометрических тождеств

  1. Пример 1: Докажите следующее тождество, используя тригонометрические тождества:

    [(sin 3θ + cos 3θ)/(sin θ + cos θ)] + sin θ cos θ = 1

    Решение:

    Мы используем следующее тождество:

    a 3 +b 3 = (a+b)(a 2 -ab+b 2 )

    Для доказательства тождеств Пифагора это тождество.

    Л.Х.С. = [(sin 3θ + cos 3θ)(sin θ + cos θ)] + sin θ cos θ

    = [(sin θ + cos θ)(sin 2 θ — sin θ cos θ + cos 2 θ )(sin θ + cos θ) + sin θ cos θ

    = (sin 2 θ — sin θ cos θ + cos 2 θ) + sin θ cos θ

    = sin 2 θ + cos 2 θ

    = 1 = R.H.S.

    Ответ: Данное тождество доказано.

  2. Пример 2: Докажите следующее тождество, используя тождества тригонометрии:

    (sin θ + cosec θ) 2 + (cos θ + sec θ) 2 = 7 + cottan 5 2 09 9 θ θ

    Решение:

    Мы используем взаимные тождества и тождества Пифагора, чтобы доказать это тождество.

    L H S = (sin 2 θ + cosec 2 θ + 2 sin θ cosec θ) + (cos 2 θ + sec 2 θ + 2 cos θ 9 sec

    θ)

    θ + cos 2 θ + cosec 2 θ + sec 2 θ + 2 + 2

    = 1 + (1 + ctg 2 θ) + (1 + tan 2 + 2 2

    = 7 + загар 2 θ + кроватка 2 θ = правая сторона

    Ответ: Данное тождество доказано.

  3. Пример 3: Найдите точное значение sin 75°, используя тождества триго.

    Решение:

    Мы знаем, что 75° = 30° + 45°

    Применим тождество суммы sin, чтобы найти значение sin 75°.

    sin (A+B) = sin A cos B + cos A sin B

    Замените здесь A = 30° и B = 45° с обеих сторон:

    sin (30° + 45°) = sin 30° cos 45°+ cos 30° + sin 45°

    sin 75° = (1/2)⋅(√2/2) + (√3/2)⋅(√2/2)

    sin 75° = (√ 2 + √6)/4 = (√3 + 1)2√2

    Здесь значения sin 30°, cos 45°, cos 30° и sin 45° можно получить с помощью тригонометрической таблицы.

    Ответ: sin 75°= (√3 + 1)/2√2

перейти к слайдуперейти к слайдуперейти к слайду

Разбивайте сложные концепции с помощью простых визуальных эффектов.

Математика больше не будет сложным предметом, особенно когда вы понимаете концепции с помощью визуализаций.

Запись на бесплатный пробный урок

Практические вопросы по тригонометрическим тождествам

 

перейти к слайдуперейти к слайду

Часто задаваемые вопросы о тригонометрических тождествах

Что такое тригонометрические тождества в тригонометрии?

Тригонометрические тождества — это равенства, включающие тригонометрические функции и истинные для каждого значения вовлеченных переменных таким образом, что обе части равенства определены. Некоторые важные тождества в тригонометрии задаются следующим образом:

  • sin θ = 1/cosec θ
  • cos θ = 1/сек θ
  • загар θ = 1/кот θ
  • sin 2 θ + cos 2 θ = 1
  • 1 + тангенс 2 θ = сек 2 θ
  • 1 + кроватка 2 θ = cosec 2 θ

Что такое 3 тригонометрических тождества?

Три тригонометрических тождества задаются как

  • sin 2 θ + cos 2 θ = 1
  • 1 + тангенс 2 θ = сек 2 θ
  • 1 + кроватка 2 θ = cosec 2 θ

Для чего используются тригонометрические тождества?

Основные области применения тригонометрических тождеств включают:

  • Упрощение выражений: Тригонометрические тождества позволяют нам упростить сложные тригонометрические выражения, заменяя их более простой формой, что может быть полезно при решении тригонометрических уравнений, построении графиков тригонометрических функций и упрощении вычислений. .
  • Решение уравнений: Тригонометрические тождества можно использовать для решения тригонометрических уравнений путем преобразования их в более простые формы. Это часто делается с использованием более чем одного тождества, чтобы привести уравнение к более легко решаемой форме.
  • Доказательство теорем: Тригонометрические тождества часто используются при доказательстве математических теорем, особенно в геометрии и исчислении. Используя их, математики могут установить справедливость различных геометрических и математических формул.
  • Вычисление значений : Тригонометрические тождества используются для вычисления значений тригонометрических функций, таких как синус, косинус и тангенс, для различных углов. Это важно во многих приложениях, таких как навигация, геодезия и инженерия.

Как доказать тригонометрические тождества?

Тригонометрические тождества можно доказать, используя другие известные пифагорейские и тригонометрические тождества. Мы также можем использовать некоторые тригонометрические соотношения и формулы для доказательства тригонометрических тождеств.

Что такое тождества противоположных углов в тригонометрии?

Тождества противоположного угла говорят о том, что происходит с коэффициентами триггера, когда угол отрицательный. Они следующие:

  • sin(-x) = — sin x; csc(-x) = — csc х
  • cos(-x) = cosx; сек(-х) = сек х
  • тангенс(-х) = — тангенс х; детская кроватка (-x) = — детская кроватка x

Эти тождества могут быть получены из определений тригонометрических функций и свойств единичной окружности.

Как вы решаете уравнения с триггерными тождествами?

С помощью тригонометрических тождеств можно решать различные математические задачи. Мы можем преобразовать уравнения в параметрические уравнения, а затем применить тригонометрические тождества для их решения.

Какие тригонометрические тождества мы должны знать?

При решении задач используются все триггерные тождества.

Ax формула: Скорость прямолинейного равноускоренного движения: график скорости

О формулах | Microsoft Learn

  • Статья

Применимо к: Microsoft Dynamics AX 2012 R3, Microsoft Dynamics AX 2012 R2, Microsoft Dynamics AX 2012 Feature Pack, Microsoft Dynamics AX 2012

Формула состоит из обязательных компонентов и количеств, которые необходимы для производства установленного количества номенклатуры-формулы. В зависимости от выполняемой задачи доступ к функциональным возможностям формулы открывается с формы Управление запасами и складом или Управление сведениями о продукте.

Формулы и строки формул

В Непрерывное производство и логистика формула состоит из одной или нескольких строк формулы, идентифицирующих компоненты или номенклатуры, которые образуют формулу. В строке формулы могут содержаться номенклатуры спецификации, номенклатуры-формулы, номенклатуры , учитываемые в двух единицах измерения, приобретенные товары, сопутствующие или побочные продукты. Поскольку многие номенклатуры задействованы в нескольких продуктах, любая номенклатура может использоваться более чем в одной формуле.

Примером формулы может служить формула печенья с шоколадной крошкой. В этой формуле используется несколько строк ингредиентов, таких как мука, сахар, яйца, масло и шоколадная крошка. Формула печенья с шоколадной крошкой содержит ингредиенты, которые, весьма вероятно, используются в других формулах. Например, в процессе изготовления печенья с шоколадной крошкой возможны остатки, такие как крошки либо отдельные недопеченные или перепеченные печенья. Эти номенклатуры могли бы быть настроены как сопутствующие или побочные продукты в зависимости от производственных операций.

Версии формулы

Чтобы создать новую формулу, необходимо сначала создать версию формулы, прежде чем добавлять номенклатуры строк формулы с их особыми характеристиками. Каждая формула должна иметь хотя бы одну версию. Кнопка Утверждено для версии формулы становится доступной только после успешного сохранения записи версии. Каждая запись версии формулы связана с одним или несколькими сопутствующими и побочными продуктами, которые могут изготавливаться в процессе производства конечного продукта. Многие продукты могут изготавливаться из одних и тех же ингредиентов с разными размерами партий, составами или с разными выработками. Можно создавать любое необходимое число версий формулы.

Для управления несколькими активными версиями формул используют поля диапазонов дат действия или дат поставки количества. Несколько активных версий формулы могут существовать только в том случае, если диапазон дат и дата поставки количества не перекрываются.

Утверждение и активация формул и версий формул

Формулы и версии формул должны быть утверждены, прежде чем их можно будет использовать для планирования и производства. Хотя формулы обычно активируются до их использования, можно выбрать во время производства версию формулы, которая утверждена, но не активирована.

Чтобы защитить формулу или версию формулы, можно установить параметры Блокировать изменение и Блокировать удаление утверждения на форме Параметры управления производством.

Если выбрать параметр Блокировать изменения когда формула утверждена, никакие поля в строках формулы не будут доступны для удаления и изменения. Однако если удалить утверждение формулы, строки формулы можно будет удалять и изменять. Можно также создать новые формулы и новые версии формул.

Если установить флажок Блокировать удаление утверждения, удаление утверждения для утвержденной формулы или версии формулы будет невозможно. Однако можно создать новые формулы и новые версии формул, а также можно удалить активацию версии формулы.

Можно добавить дополнительные уровни контроля, используя для этого возможности электронной подписи. Если для утверждения формулы настроено требование электронной подписи, то при активации формулы откроется форма Подпись. Необходимо пройти авторизацию, чтобы поставить электронную подпись, и сертификат должен быть успешно подтвержден для фиксации изменения. Если проверка подлинности подписи не может быть выполнена, утверждение или удаление утверждения отклоняется, и инициированное изменение возвращается в исходное состояние.

Использование свойства масштабируемости

Свойство масштабируемости доступно только в том случае, если в формуле для всех компонентов номенклатуры установлена настройка Переменное потребление. Свойство не доступно для настроек Фиксированное потребление и Ступенчатое потребление. При использовании функции масштабируемости любое изменение, вносимое в ингредиент в формуле, будет также приводить к регулировке количества других выбираемых ингредиентов. Размер формулы также регулируется. Подобным образом любое изменение размера формулы вызывает изменение количества всех ингредиентов, которые являются масштабируемыми. Это свойство специально предназначено для создания и поддержки формул и не показывает, будет ли количество ингредиента увеличиваться или уменьшаться в партионном заказе.

Использование ступенчатого потребления

Ступенчатое потребление исключает необходимость ввода для ингредиента количества на вкладке Строка формулы. Вместо этого для ступенчатого потребления настраиваются параметры Из серии и Количество. На основе количества партионного заказа выбирается информация из записи ступенчатого потребления на серию, которое удовлетворяет этому количеству. Это полезно, когда норма потребления не изменяется линейно с размером партионного заказа, а только приводит к повышению спроса при достижении некоторого порога в количестве. Чтобы включить эту функцию для новой формулы, измените для применимого ингредиента настройку формулы в разделе Группа расчета потребления со значения Стандартно на значение Ступенчато. Этот метод потребления задают на вкладке Настройка формы Строка формулы.

См. также

О типах строк

Формула (форма)

Строка формулы (форма)

Версия формулы (форма)

Параметры управления производства (форма)

Пересчёт астигматики

Главная \ Сервис \ Пересчет астигматики

  

Одну и ту же рефракцию одной и той же линзы можно записать как с положительным цилиндром, так и с отрицательным значением цилиндра, при этом оптическая сила данной линзы не меняется. При транспозиции оба варианта записи очкового рецепта относятся к одной и той же очковой линзе.

Для вашего удобства Вы можете воспользоваться удобный калькулятором пересчета астигматики, нужно лишь ввести ваши параметры. Либо ознакомиться с правилами пересчета астигматизма ниже 

 

Ваш рецепт:

OD:

Сфера(Sph) +12.00+11.75+11.50+11.25+11.00+10.75+10.50+10.25+10.00+9.75+9.50+9.25+9.00+8.75+8.50+8.25+8.00+7.75+7.50+7.25+7.00+6.75+6.50+6.25+6.00+5.75+5.50+5.25+5.00+4.75+4.50+4.25+4.00+3.75+3.50+3.25+3.00+2.75+2.50+2.25+2.00+1.75+1.50+1.25+1.00+0.75+0.50+0.250.00-0.25-0.50-0.75-1.00-1.25-1.50-1.75-2.00-2.25-2.50-2.75-3.00-3.25-3.50-3.75-4.00-4.25-4.50-4.75-5.00-5.25-5.50-5.75-6.00-6.25-6.50-6.75-7.00-7.25-7.50-7.75-8.00-8.25-8.50-8.75-9.00-9.25-9.50-9.75-10.00-10.25-10.50-10.75-11.00-11.25-11.50-11.75-12.00

Цилиндр(Cyl) +4.00+3.75+3.50+3.25+3.00+2.75+2.50+2.25+2.00+1.75+1.50+1.25+1.00+0.75+0.50+0.250.00-0.25-0.50-0.75-1.00-1.25-1.50-1.75-2.00-2.25-2.50-2. 75-3.00-3.25-3.50-3.75-4.00

Ось(ax) 0°10°20°30°40°50°60°70°80°90°100°110°120°130°140°150°160°170°180°

OS:

Сфера(Sph) +12.00+11.75+11.50+11.25+11.00+10.75+10.50+10.25+10.00+9.75+9.50+9.25+9.00+8.75+8.50+8.25+8.00+7.75+7.50+7.25+7.00+6.75+6.50+6.25+6.00+5.75+5.50+5.25+5.00+4.75+4.50+4.25+4.00+3.75+3.50+3.25+3.00+2.75+2.50+2.25+2.00+1.75+1.50+1.25+1.00+0.75+0.50+0.250.00-0.25-0.50-0.75-1.00-1.25-1.50-1.75-2.00-2.25-2.50-2.75-3.00-3.25-3.50-3.75-4.00-4.25-4.50-4.75-5.00-5.25-5.50-5.75-6.00-6.25-6.50-6.75-7.00-7.25-7.50-7.75-8.00-8.25-8.50-8.75-9.00-9.25-9.50-9.75-10.00-10.25-10.50-10.75-11.00-11.25-11.50-11.75-12.00

Цилиндр(Cyl) +4.00+3.75+3.50+3.25+3.00+2.75+2.50+2.25+2.00+1.75+1.50+1.25+1.00+0.75+0.50+0.250.00-0.25-0.50-0.75-1.00-1.25-1.50-1.75-2.00-2.25-2.50-2.75-3.00-3.25-3.50-3.75-4.00

Ось(ax) 0°10°20°30°40°50°60°70°80°90°100°110°120°130°140°150°160°170°180°

 

 

Так же у нас есть калькулятор для быстрого пересчета диаметра линз, чтобы воспользоваться перейдите по ссылке.

 

Правила транспозиции

Для примера возьмем рецепт: Sph +2,0 D cyl +1,0 D ax 90°. Для транспозиции данной записи необходимо сделать следующие шаги:

А) новое значение сферы после транспозиции равняется сумме сферы и цилиндра , то есть +2,0 +1,0= +3,0 D;

Б) Новое значение цилиндра после транспозиции равняется имеющейся силе цилиндра, но с противоположным знаком, или значению цилиндра умноженном, то есть +1,0x (1)= 1,0 D;

В) Новое значение оси после транспозиции равняется имеющейся оси, изменённой на ±90; если ось цилиндра в рецепте 90v или меньше, следует прибавить 90, а если ось цилиндра 91 или больше, следует вычесть 90, то есть 90 + 90=180°.

В данном примере после транспозиции у нас получилось следующая запись: Sph +3,0 D cyl -1,0 D  ax 180°

 

Примеры:

-2,00/+1,00 ax 90°      ⟷     -1,00/-1,00 ax 180°

-1,50/+4,00 ax 180°    ⟷   +2,50/-4,00 ax 90°

-2,75/+2,75 ax 135°    ⟷    0,0/-2,75 ax 45°

+6,00/-0,25 ax 5°        ⟷    +5,75/+0,25 ax 95°

+3,50/+3,50 ax 100°   ⟷   +7,00/-3,50 ax 10°

0,00/+3,75 ax 180°     ⟷    +3,75/-3,75 ax 90°

+4,00/-2,00 ax 130°    ⟷   +2,00/+2,00 ax 40°

 

Рекомендация:

Чтобы было проще сделать транспозицию, можно воспользоваться следующим правилом: если рецепт выписан с отрицательным цилиндром, то указанной в рецепте оси будет соответствовать значение сферы, а второй оси – алгебраическая сумма сферы и цилиндра.

Пример:

Sph – 3,0 D cyl -1,0 D ax 45°

На оси 45° (именно эта ось будет указана в рецепте) оптическая сила будет -3,0, а на другой оси 135° будет значение

-4,0: -3,0 + (-1,0) = — 3,0 = -4,0 D

Таким образом, после транспозиции запись будет следующей:

Sph -4,0 D cyl +1,0 ax 135°

 

У Вас остались вопросы?

Наши специалисты с удовольствием помогут Вам,
тел.: 8(3852)404-890
Или напишите:  [email protected]

Прозрение оптика Барнаул

 

CBD Synergies-AX Успокаивающая формула — Mountain States Health Products Inc

CBD Synergies-AX — это премиальная успокаивающая смесь растительных и нутрицевтических средств, которая быстро переводит ваше тело из истощающего здоровье режима «бей или беги» в восстанавливающий режим «отдых и восстановление». » режим.

Центральным элементом нашей формулы является богатый КБД экстракт конопли широкого спектра действия, известный своим успокаивающим, уравновешивающим действием на разум и тело. Бета-кариофиллен, уникальный терпен на основе конопли, усиливает успокаивающее действие КБД, поддерживая общее самочувствие.

  • Обзор 1″>s
  • Размер:   1,7 унции
  • Производитель:   Quicksilver Scientific
  • Артикул:   QS-CBDCALMING
  • Наличие на складе:   27
Дополнительные надстройки

product.variants[0].price»>НАБОР

Добавить Удалить

Дополнительные надстройки

product. variants[0].price»>НАБОР

  • Описание
  • Ингредиенты
  • Дозировка

КБД, растительные компоненты и аминокислоты для сбалансированного дзен-состояния тела и разума

Быстрый переход от режима «бей или беги» к режиму отдыха и восстановления помпы ежедневно натощак, не менее чем за 10 минут до еды. Обязательно подержите жидкость во рту в течение 30 секунд для оптимального всасывания.

×

Информация о продукте

Вам также может понравиться

product. variants[0].price»>НАБОР

Вам также может понравиться

product.variants[0].price»>НАБОР

Корзина для покупок

  • ${опция.имя}: ${опция.значение}

Периодическая доставка каждые ${ item.properties[‘shipping_interval_frequency’] } ${ item.properties[‘shipping_interval_unit_type’] }.
Изменить или отменить в любое время.

Ваша корзина пуста.

  • product_options» v-if=»option.value !== ‘Default Title'»> ${опция.имя}: ${опция.значение}

Продолжить просмотр Касса

Жидкая ошибка (сниппеты/modal-cart-upsell, строка 49).): в этом контексте использование include запрещено.

Основные уравнения прямых и плоскостей

Основные уравнения прямых и плоскостей

Уравнение прямой

Важным предметом школьной алгебры является «уравнение прямой». Это означает уравнение относительно x и y, набор решений которого представляет собой линию в (x, y) самолет.

Самая популярная форма в алгебре — это форма «пересечение наклона».

у = тх + б.

Это фактически использует x как параметр и записывает y как функцию x: y = f(x) = мх+б. Когда x = 0, y = b и точка (0,b) является пересечением прямой с осью Y.

Думая о линии как о геометрическом объекте, а не о графике функции, имеет смысл относиться к x и y более беспристрастно. Общее уравнение для строка (нормальная форма)

топор + по = с,

с условием, что по крайней мере один из a или b отличен от нуля. Это может легко преобразовать в форму пересечения наклона, решив для y:

у = (-а/б) + с/б,

, за исключением особого случая b = 0, когда линия параллельна оси y.

Если коэффициенты в нормальной форме умножаются на ненулевую константу, множество решений точно такое же, поэтому, например, все эти уравнения имеют ту же строку, что и решение.

2x + 3 y = 4
4x + 6y = 8
-x — (3/2) y = -2
(1/2)x + (3/4)y = 1

В общем случае, если k ненулевая константа, то это уравнений для одна и та же строка , так как они имеют одинаковые решения.

ax + by = c
(ka)x + (kb)y = kc.

Популярный выбор для k в случае, когда c не равно нулю, это k = (1/с). Тогда уравнение становится

(а/с)х + (б/с)у = 1.

Другая полезная форма уравнения — разделить на |(a,b)|, квадратный корень из 2 + b 2 . Этот выбор будет объяснен в разделе Normal Vector.

Упражнение . Если на прямой стоит O, покажите, что уравнение принимает вид ax + by = 0 или y = mx.

Упражнение: Найдите точки пересечения этой прямой с координатные оси.

Упражнение : Уравнение прямой, проходящей через (0,0) и точка (h,k)?


Нахождение уравнения прямой через 2 точки на плоскости

Для любых двух точек P и Q через эти точки проходит ровно одна прямая PQ. Если известны координаты точек P и Q, то коэффициенты a, b, c уравнение для прямой можно найти, решив систему линейных уравнений.

Пример : Для P = (1, 2), Q = (-2, 5) найдите уравнение ax + by = c строки PQ.

Поскольку точка P находится на прямой, ее координаты удовлетворяют уравнению: a1 + b2 = c, или а + 2b = с.
Поскольку Q находится на прямой, его координаты удовлетворяют уравнению: a(-2) + b5 = c, или -2 а + 5b = с.

Умножьте первое уравнение на 2 и прибавьте, чтобы исключить a из уравнения: 4b + 5b = 9b = 2c + c = 3c, поэтому b = (1/3)c. Затем подставляя в первый уравнение, a = c — 2b = c — (2/3)c = (1/3)c.

Это дает уравнение [(1/3)c]x + [(1/3)c}y = c . Почему нет с решено для? Помните, что существует бесконечное количество уравнений для линии, каждая из которых кратна другой. Мы можем вынести c (или положить c = 1 для того же результата) и получить (1/3)x + (1/3)y =1 как один из вариантов уравнение для прямой. Другим выбором может быть c = 3: x+y = 3 , что очистил знаменатели.

Этот метод всегда работает для любых различных P и Q. Конечно, существует формула также для а, б, в. Это можно найти в виде определители , или перекрестное произведение .

Упражнения : Найдите уравнения этих линий. Обратите внимание на особые случаи.

Линия через (3, 4) и (1, -2).
Линия через (3, 4) и (-6, -8).
Линия через (3, 4) и (3, 7).


Соединение с параметрической формой линии

Для двух точек P и Q точки прямой PQ можно записать как F(t) = (1-t)P + tQ, поскольку t охватывает все действительные числа. Если и P, и Q удовлетворяют одному и тому же уравнение ax+by = c, то вычисление показывает, что это верно и для (1-t)P + tQ для любого выбора t.

Вот это вычисление. Пусть P = (p 1 , p 2 ), Q = (q 1 , q 2 ). Тогда, поскольку точки лежат на прямой, мы знаем, что оба

ap 1 + bp 2 = c
aq 1 + bq 2 = c.

Для точки F(t) мы должны проверить a[(1-t)p 1 +tq 1 ] + b[(1-t)p 2 +tq 2 ] = с. Но левую часть можно переставить как (1-t)(ap 1 + bp 2 ) + t(aq 1 + bq 2 ), и это равно (1-t)c + tc = c. Итак равенство выполняется. Сравните это явное вычисление с вычислением, данным для плоскости, использующей скалярное произведение. Вычисления те же, но показывает больше деталей, а один скрывает координаты и показывает более концептуальный картина.



Уравнение плоскости

Плоскость в трехмерном пространстве имеет уравнение

топор+бы+ч=д,

, где хотя бы одно из чисел a, b, c должно быть ненулевым.

Что касается линии, если уравнение умножить на любую ненулевую константу k до получаем уравнение kax+kby+kcz=kd, плоскости решений совпадают.

Если с не равно нулю, часто полезно думать о плоскости как о графике функция z от x и y. Уравнение можно переставить так:

z = -(a/c)x + (-b/c)y + d/c

Еще один полезный вариант, когда d не равен нулю, — разделить на d так, чтобы константа срок = 1,

(a/d)x + (b/d)y + (c/d)z = 1.

Другой полезной формой уравнения является деление на |(a,b,c)|, квадрат корень a 2 + b 2 + c 2 . Этот выбор будет объяснено в разделе «Вектор нормалей».

Упражнение: Где плоскость ax + by + cz = d пересекает координату топоры?

Упражнение: Что особенного в уравнении плоскости, проходящей через 0,


Нахождение уравнения плоскости через 3 точки в космос

Учитывая точки P, Q, R в пространстве, найдите уравнение плоскости через 3 точки.

Пример : P = (1, 1, 1), Q = (1, 2, 0), R = (-1, 2, 1). Ищем коэффициенты уравнения ax + by + cz = d, где P, Q и R удовлетворяют уравнениям, таким образом:

а + b + с = d
а + 2b + 0с = d
-а + 2b + с = d

Вычитание первого уравнения из второго и последующее сложение первого уравнения к третьему, мы исключаем a, чтобы получить

б — в = 0
4б + в = 2d

Сложение уравнений дает 5b = 2d или b = (2/5)d, затем решение для c = b = (2/5)d и тогда a = d — b — c = (1/5)d.

Таким образом, уравнение (с ненулевой константой, оставленной для выбора) имеет вид d(1/5)x + d(2/5)y + d(2/5)z = d, поэтому один выбор константы дает

х + 2у + 2г = 5

или другой вариант: (1/5)x + (2/5)y + (2/5)z = 1

Учитывая координаты P, Q, R, есть формула для коэффициентов плоскость, которая использует определители или перекрестное произведение .

Упражнение. Какое уравнение плоскости через точки I, J, K?

Упражнение: Какое уравнение плоскости через (1, 1, 1), (-1, 1, -1) и (1, -1, -1)?

Упражнение: Сравните этот метод нахождения уравнения плоскости с векторным произведением метод.


Соединение с параметрической формой плоскости

Для 3 точек P, Q, R все точки плоскости можно записать в параметрическом форма F(s,t) = (1 — s — t)P + sQ + tR, где s и t варьируются по всем действительным числам.

Вычисление, подобное приведенному выше для уравнения прямой, показывает, что если P, Q, R удовлетворяют одному и тому же уравнению ax + by + cz = d, тогда все точки F(s,t) также удовлетворяют тому же уравнению.

Решить уравнения онлайн с решением: Решение квадратных уравнений онлайн

2
Функция — Квадрат x
ctg(x)
Функция — Котангенс от x
arcctg(x)
Функция — Арккотангенс от x
arcctgh(x)
Функция — Гиперболический арккотангенс от x
tg(x)
Функция — Тангенс от x
tgh(x)
Функция — Тангенс гиперболический от x
cbrt(x)
Функция — кубический корень из x
gamma(x)
Гамма-функция
LambertW(x)
Функция Ламберта
x! или factorial(x)
Факториал от x
DiracDelta(x)
Дельта-функция Дирака
Heaviside(x)
Функция Хевисайда

Интегральные функции:

Si(x)
Интегральный синус от x
Ci(x)
Интегральный косинус от x
Shi(x)
Интегральный гиперболический синус от x
Chi(x)
Интегральный гиперболический косинус от x

В выражениях можно применять следующие операции:

Действительные числа
вводить в виде 7. 3
— возведение в степень
x + 7
— сложение
x — 6
— вычитание
15/7
— дробь

Другие функции:

asec(x)
Функция — арксеканс от x
acsc(x)
Функция — арккосеканс от x
sec(x)
Функция — секанс от x
csc(x)
Функция — косеканс от x
floor(x)
Функция — округление x в меньшую сторону (пример floor(4.5)==4.0)
ceiling(x)
Функция — округление x в большую сторону (пример ceiling(4.5)==5.0)
sign(x)
Функция — Знак x
erf(x)
Функция ошибок (или интеграл вероятности)
laplace(x)
Функция Лапласа
asech(x)
Функция — гиперболический арксеканс от x
csch(x)
Функция — гиперболический косеканс от x
sech(x)
Функция — гиперболический секанс от x
acsch(x)
Функция — гиперболический арккосеканс от x

Постоянные:

pi
Число «Пи», которое примерно равно ~3. 14159..
e
Число e — основание натурального логарифма, примерно равно ~2,7183..
i
Комплексная единица
oo
Символ бесконечности — знак для бесконечности

Solving equations with variables on both sides calculator

  • Expression
  • Equation
  • Inequality
  • Contact us
  • Simplify
  • Factor
  • Expand
  • GCF
  • LCM
  • Solve
  • Graph
  • Система
  • Решить
  • График
  • Система
  • Математический решатель на вашем сайте

решение уравнений с переменными в обе стороны калькулятор
Связанные темы:
prentice hall алгебра 1 ответов | вопросы и ответы по алгебре | решить для неизвестного в задаче о распределительном свойстве с дробями | радикальные представители | как заниматься алгеброй | рабочий лист дробных показателей | алгебра практики матрицы данных | математические полиномы 9 класса

Автор Сообщение
Лидандо

Дата регистрации: 08. 12.2002
Откуда: Швеция

Размещено: Пятница, 03 августа, 08:59

Привет всем, я только начал решать уравнения с переменными в классе калькулятора с обеих сторон. Мальчик! Эта вещь действительно ужасна! Я просто, кажется, никогда не понимаю смысл какой-либо концепции. Результат? Мои рейтинги падают. Есть ли гуру, который может протянуть мне руку помощи?
Наверх
AllejHat

Зарегистрирован: 16. 07.2003
Откуда: Оденсе, Дания

Размещено: Пятница, 03 августа, 16:20

У меня есть для вас выход, и он может оказаться лучше, чем покупка нового учебника. Попробуйте Algebrator, он охватывает довольно сложный список математических тем и настоятельно рекомендуется. Это поможет вам решить различные типы вопросов, а также ответит на все ваши вопросы о том, как он пришел к конкретному ответу. Я попробовал его, когда у меня были трудности с решением задач, основанных на решении уравнений с переменными на обеих сторонах калькулятора, и мне очень понравилось его использовать.
Наверх
СжберАлием

Дата регистрации: 06.03.2002
Откуда: Macintosh HD

Размещено: Суббота, 04 августа, 13:35

Здравствуйте, я учитель математики. Я использую эту программу всякий раз, когда застреваю в какой-либо проблеме. Алгебратор, несомненно, очень удобная программа.
Наверх
Blanded

Зарегистрирован: 18. 01.2004
Откуда: Джексонвилл, Флорида

Размещено: Понедельник, 06 августа, 10:57

Звучит как то, что я искал все это время! Спасибо, ребята, только один последний вопрос, может ли кто-нибудь дать мне ссылку, где я могу заказать свою копию этой программы?
Наверх
Хиинидам

Дата регистрации: 06. 07.2001
Откуда: Грили, Колорадо, США

Размещено: Понедельник, 06 Авг, 14:15

Algebrator — невероятное программное обеспечение, и его, безусловно, стоит попробовать. Там вы найдете несколько интересных вещей. Я использую его в качестве справочного программного обеспечения для решения математических задач и могу поклясться, что изучение математики стало намного увлекательнее.
Наверх
Утафниминтджо

Зарегистрирован: 22.

Умножение матриц 3 на 3: Умножение матриц онлайн

Линейная алгебра на Python. [Урок 3]. Действия над матрицами

Тема третьего урока: действия над матрицами. В рамках нее будут рассмотрены следующие вопросы: умножение матрицы на число, сложение и умножение матриц.

  • Действия над матрицами
    • Умножение матрицы на число
    • Сложение матриц
    • Умножение матриц

Действия над матрицами

Умножение матрицы на число

При умножении матрицы на число, все элементы матрицы умножаются на это число:

Численный пример

Пример на Python

>>> A = np.matrix('1 2 3; 4 5 6')
>>> C = 3 * A
>>> print(C)
[[ 3  6 9]
[12 15 18]]

 

Рассмотрим свойства операции умножения матрицы на число.

Свойство 1. Произведение единицы и любой заданной матрицы равно заданной матрице:

Численный пример

Пример на Python

>>> A = np. matrix('1 2; 3 4')
>>> L = 1 * A
>>> R = A
>>> print(L)
[[1 2]
[3 4]]

>>> print(R)
[[1 2]
[3 4]]

 

Свойство 2. Произведение нуля и любой матрицы равно нулевой матрице, размерность которой равна исходной матрицы:

Численный пример

Пример на Python

>>> A = np.matrix('1 2; 3 4')
>>> Z = np.matrix('0 0; 0 0')
>>> L = 0 * A
>>> R = Z

>>> print(L)
[[0 0]
[0 0]]

>>> print(R)
[[0 0]
[0 0]]

 

Свойство 3. Произведение матрицы на сумму чисел равно сумме произведений матрицы на каждое из этих чисел:

Численный пример

Пример на Python

>>> A = np.matrix('1 2; 3 4')
>>> p = 2
>>> q = 3

>>> L = (p + q) * A
>>> R = p * A + q * A

>>> print(L)
[[ 5 10]
[15 20]]

>>> print(R)
[[ 5 10]
[15 20]]

 

Свойство 4. Произведение матрицы на произведение двух чисел равно произведению второго числа и заданной матрицы, умноженному на первое число:

Численный пример

Пример на Python

>>> A = np.matrix('1 2; 3 4')
>>> p = 2
>>> q = 3

>>> L = (p * q) * A
>>> R = p * (q * A)

>>> print(L)
[[ 6 12]
[18 24]]

>>> print(R)
[[ 6 12]
[18 24]]

 

Свойство 5. Произведение суммы матриц на число равно сумме произведений этих матриц на заданное число:

Численный пример

Пример на Python

>>> A = np.matrix('1 2; 3 4')
>>> B = np.matrix('5 6; 7 8')
>>> k = 3

>>> L = k * (A + B)
>>> R = k * A + k * B

>>> print(L)
[[18 24]
[30 36]]

>>> print(R)
[[18 24]
[30 36]]

 

Сложение матриц

Складывать можно только матрицы одинаковой размерности — то есть матрицы, у которых совпадает количество столбцов и строк.

Численный пример

Пример на Python

>>> A = np.matrix('1 6 3; 8 2 7')
>>> B = np.matrix('8 1 5; 6 9 12')
>>> C = A + B

>>> print(C)
[[ 9  7 8]
[14 11 19]]

 

Рассмотрим свойства сложения матриц.

Свойство 1. Коммутативность сложения. От перестановки матриц их сумма не изменяется:

Численный пример

Пример на Python

>>> A = np.matrix('1 2; 3 4')
>>> B = np.matrix('5 6; 7 8')

>>> L = A + B
>>> R = B + A

>>> print(L)
[[ 6  8]
[10 12]]

>>> print(R)
[[ 6  8]
[10 12]]

 

Свойство 2. Ассоциативность сложения. Результат сложения трех и более матриц не зависит от порядка, в котором эта операция будет выполняться:

Численный пример

Пример на Python

>>> A = np. matrix('1 2; 3 4')
>>> B = np.matrix('5 6; 7 8')
>>> C = np.matrix('1 7; 9 3')

>>> L = A + (B + C)
>>> R = (A + B) + C
>>> print(L)

[[ 7 15]
[19 15]]

>>> print(R)
[[ 7 15]
[19 15]]

 

Свойство 3. Для любой матрицы существует противоположная ей , такая, что их сумма является нулевой матрицей :

Численный пример

Пример на Python

>>> A = np.matrix('1 2; 3 4')
>>> Z = np.matrix('0 0; 0 0')

>>> L = A + (-1)*A

>>> print(L)
[[0 0]
[0 0]]

>>> print(Z)
[[0 0]
[0 0]]

 

Умножение матриц

Умножение матриц это уже более сложная операция, по сравнению с рассмотренными выше. Умножать можно только матрицы, отвечающие следующему требованию: количество столбцов первой матрицы должно быть равно числу строк второй матрицы.

Для простоты запоминания этого правила можно использовать диаграмму умножения, представленную на рисунке 1.

Рисунок 1 — Диаграмма матричного умножения

Рассмотрим умножение матриц на примере.

Численный пример

Каждый элемент cij новой матрицы является суммой произведений элементов i-ой строки первой матрицы и j-го столбца второй матрицы. Математически это записывается так:

Пример на Python

Решим задачу умножения матриц на языке Python. Для этого будем использовать функцию dot() из библиотеки Numpy:

>>> A = np.matrix('1 2 3; 4 5 6')
>>> B = np.matrix('7 8; 9 1; 2 3')

>>> C = A.dot(B)

>>> print(C)
[[31 19]
[85 55]]

 

Ниже представлены свойства произведения матриц. Примеры свойств будут показаны для квадратной матрицы.

Свойство 1. Ассоциативность умножения. Результат умножения матриц не зависит от порядка, в котором будет выполняться эта операция:

Численный пример

Пример на Python

>>> A = np.matrix('1 2; 3 4')
>>> B = np.matrix('5 6; 7 8')
>>> C = np.matrix('2 4; 7 8')

>>> L = A.dot(B.dot(C))
>>> R = (A.dot(B)).dot(C)

>>> print(L)
[[192 252]
[436 572]]

>>> print(R)
[[192 252]
[436 572]]

 

Свойство 2. Дистрибутивность умножения. Произведение матрицы на сумму матриц равно сумме произведений матриц:

Численный пример

Пример на Python

>>> A = np.matrix('1 2; 3 4')
>>> B = np.matrix('5 6; 7 8')
>>> C = np.matrix('2 4; 7 8')

>>> L = A. dot(B + C)
>>> R = A.dot(B) + A.dot(C)

>>> print(L)
[[35 42]
[77 94]]
>>> print(R)
[[35 42]
[77 94]]

 

Свойство 3. Умножение матриц в общем виде не коммутативно. Это означает, что для матриц не выполняется правило независимости произведения от перестановки множителей:

Численный пример

Пример на Python

>>> A = np.matrix('1 2; 3 4')
>>> B = np.matrix('5 6; 7 8')

>>> L = A.dot(B)
>>> R = B.dot(A)

>>> print(L)
[[19 22]
[43 50]]

>>> print(R)
[[23 34]
[31 46]]

 

Свойство 4. Произведение заданной матрицы на единичную равно исходной матрице:

Численный пример

Пример на Python

>>> A = np.matrix('1 2; 3 4')
>>> E = np. matrix('1 0; 0 1')

>>> L = E.dot(A)
>>> R = A.dot(E)

>>> print(L)
[[1 2]
[3 4]]

>>> print(R)
[[1 2]
[3 4]]

>>> print(A)
[[1 2]
[3 4]]

 

Свойство 5. Произведение заданной матрицы на нулевую матрицу равно нулевой матрице:

Численный пример

Пример на Python

>>> A = np.matrix('1 2; 3 4')
>>> Z = np.matrix('0 0; 0 0')

>>> L = Z.dot(A)
>>> R = A.dot(Z)

>>> print(L)
[[0 0]
[0 0]]

>>> print(R)
[[0 0]
[0 0]]

>>> print(Z)
[[0 0]
[0 0]]

 

P.S.

Вводные уроки по “Линейной алгебре на Python” вы можете найти соответствующей странице нашего сайта. Все уроки по этой теме собраны в книге “Линейная алгебра на Python”.

Если вам интересна тема анализа данных, то мы рекомендуем ознакомиться с библиотекой Pandas.  Для начала вы можете познакомиться с вводными уроками. Все уроки по библиотеке Pandas собраны в книге “Pandas. Работа с данными”.

двух матриц умножение

Вы искали двух матриц умножение? На нашем сайте вы можете получить ответ на любой математический вопрос здесь. Подробное решение с описанием и пояснениями поможет вам разобраться даже с самой сложной задачей и двух умножение матриц, не исключение. Мы поможем вам подготовиться к домашним работам, контрольным, олимпиадам, а так же к поступлению в вуз. И какой бы пример, какой бы запрос по математике вы не ввели — у нас уже есть решение. Например, «двух матриц умножение».

Применение различных математических задач, калькуляторов, уравнений и функций широко распространено в нашей жизни. Они используются во многих расчетах, строительстве сооружений и даже спорте. Математику человек использовал еще в древности и с тех пор их применение только возрастает. Однако сейчас наука не стоит на месте и мы можем наслаждаться плодами ее деятельности, такими, например, как онлайн-калькулятор, который может решить задачи, такие, как двух матриц умножение,двух умножение матриц,единичную матрицу умножить на матрицу,как матрицу умножить на дробь,как матрицу умножить на матрицу,как матрицу умножить на столбец,как найти произведение матриц,как перемножать матрицы,как перемножать матрицы 3х3,как перемножаются матрицы,как перемножить две матрицы,как перемножить матрицы разных размеров,как перемножить три матрицы,как умножать матрицы 2 на 2,как умножать матрицы друг на друга,как умножать матрицы разных размеров,как умножить две матрицы,как умножить единичную матрицу на матрицу,как умножить матрицу,как умножить матрицу 2х3 на 2х3,как умножить матрицу на единичную матрицу,как умножить матрицу на матрицу,как умножить матрицу на матрицу 3 на 3,как умножить матрицу на обратную матрицу,как умножить матрицы,как умножить на матрицу столбец,как умножить столбец на матрицу,как умножить три матрицы,какие можно матрицы складывать,матриц двух умножение,матриц умножение двух,матрица примеры умножение,матрица умножение,матрица умножение на матрицу,матрицу умножить на единичную матрицу,матрицы примеры умножение,матрицы произведение,матрицы умножение,матрицы умножение примеры,матричное умножение,перемножение двух матриц,перемножение матриц примеры,правила перемножения матриц,правила умножения матриц,правила умножения матрицы на матрицу,правило перемножения матриц,правило умножение матриц,правило умножения матриц,правило умножения матрицы на матрицу,примеры матриц умножения,примеры матрицы умножение,примеры произведение матриц,примеры умножение матрицы,примеры умножение матрицы на матрицу,примеры умножения матриц,произведение матриц примеры,произведение матриц примеры и решения,произведение матриц формула,произведение матрицу на матрицу,произведение матрицы,произведения матриц,свойства матриц умножение,свойства умножение матриц,свойства умножения матриц,сложение и умножение матриц,умножение 2х2 матриц,умножение двух матриц,умножение и сложение матриц,умножение квадратных матриц,умножение матриц 2х2,умножение матриц 2х2 на 2х2,умножение матриц 3 на 3,умножение матриц 3х3,умножение матриц двух,умножение матриц друг на друга,умножение матриц квадратных,умножение матриц на матрицу,умножение матриц правила,умножение матриц правило,умножение матриц пример,умножение матриц примеры,умножение матриц примеры с решением,умножение матриц разных размеров,умножение матриц свойства,умножение матриц третьего порядка,умножение матриц формула,умножение матрица на матрицу,умножение матрицы,умножение матрицы 2х2 на 2х2,умножение матрицы на матрицу,умножение матрицы на матрицу примеры,умножение матрицы на столбец,умножение матрицы примеры,умножение на единичную матрицу,умножения матриц,умножения матриц правила,умножения матриц примеры,умножения матриц свойства,умножения матриц формула,умножить матрицу на единичную матрицу,умножить матрицу на матрицу столбец,формула произведение матриц,формула умножение матриц,формула умножения матриц,формула умножения матрицы. На этой странице вы найдёте калькулятор, который поможет решить любой вопрос, в том числе и двух матриц умножение. Просто введите задачу в окошко и нажмите «решить» здесь (например, единичную матрицу умножить на матрицу).

Решить задачу двух матриц умножение вы можете на нашем сайте https://pocketteacher.ru. Бесплатный онлайн решатель позволит решить онлайн задачу любой сложности за считанные секунды. Все, что вам необходимо сделать — это просто ввести свои данные в решателе. Так же вы можете посмотреть видео инструкцию и узнать, как правильно ввести вашу задачу на нашем сайте. А если у вас остались вопросы, то вы можете задать их в чате снизу слева на странице калькулятора.

Матрицы умножения — примеры

М. Борн

На этой странице вы можете увидеть множество примеров умножения матриц.

Вы можете перезагружать эту страницу сколько угодно раз и каждый раз получать новый набор чисел и матриц. Вы также можете выбрать матрицы разного размера (внизу страницы).

(Если вам сначала нужна справочная информация о матрицах, вернитесь к разделу «Введение в матрицы» и «4. Умножение матриц»).

Умножение матриц А и В .

А =   6 0  
7 4
, Б =   5 -2  
0 -3

Ответить

Чтобы сэкономить работу, мы сначала проверяем, можно ли их умножить.

У нас есть (2×2) × (2×2) и поскольку количество столбцов в A такое же, как количество строк в B (в данном случае два средних числа равны 2), мы можем продолжить и умножить эти матрицы. Нашим результатом будет матрица (2×2).

Первым шагом является запись двух матриц рядом следующим образом:

АБ =   6 0  
7 4
  5 -2  
0 -3

Мы умножаем отдельные элементы в первой строке матрицы A на соответствующие элементы в первом столбце матрицы B и складываем результаты. Это дает нам число, которое нам нужно поместить в первую строку, позицию первого столбца в матрице ответов.

  6 0  
7 4
  5 -2  
0 -3

6×5 + 0×0 = 30

После этого умножаем элементы по первой строке матрицы A на соответствующие элементы вниз по второй столбец матрицы B затем добавьте результаты. Это дает нам ответ, который нам нужно поместить в первую строку, второй столбец матрицы ответов.

  6 0  
7 4
  5 -2  
0 -3

6×-2 + 0×-3 = -12

Продолжаем по строкам и столбцам следующим образом:

  6 0  
7 4
  5 -2  
0 -3

=   6×5 + 0×0 6×-2 + 0×-3  
7×5 + 4×0 7×-2 + 4×-3

=   30 -12  
35 -26

См.

другой пример?

Вы можете обновить эту страницу, чтобы увидеть другой пример с матрицами другого размера и другими числами; ИЛИ

Выберите интересующие вас размеры матрицы и нажмите кнопку .

Матрица 3×3, умноженная на матрицу 3×3
, матрица 2×3, умноженная на матрицу 3×4,
матрица 1×4, умноженная на матрицу 4×1,
матрица 4×2, умноженная на матрицу 2×3,

Умножение матриц 3×3 — Примеры

Эта статья будет о умножении матриц порядка 3×3, примерах и процедуре получения произведения.

Быстрый доступ

!Нажмите на кнопки ниже, чтобы перейти прямо к разделу статьи, которую вы ищете!

Как умножать матрицы 3×3

В этой статье мы собираемся разработать различные примеры того, как умножать матрицу 3×3. Когда мы умножаем 2 матрицы, важно проверить, что одна из матриц имеет такое же количество строк, как и столбцы другой матрицы, это означает, что если одна из матриц имеет 3 строки, другая матрица должна иметь 3 столбца, иначе , мы не можем умножать матрицы.

Основываясь на предыдущем объяснении, мы всегда можем перемножить две матрицы 3×3, потому что предыдущее правило всегда выполняется. Результатом умножения двух матриц 3×3 будет другая матрица того же порядка.

А 11 А 12 А 13
А 21 А 22 А 23
А 31 А 32 А 33

Б 11 Б 12 Б 13
Б 21 Б 22 Б 23
Б 31 Б 32 Б 33

Умножение между матрицами выполняется путем умножения каждой строки первой матрицы на каждый столбец второй матрицы, а затем сложения результатов, как в следующем примере.

  • Ряд 1
  • С 11 = (А 11 * В 11 ) + (А 12 * В 21 ) + (А 13 * В 31 )
  • С 12 = (А 11 * В 12 ) + (А 12 * В 22 ) + (А 13 * В 32 )
  • С 12 = (А 11 * В 13 ) + (А 12 * В 23 ) + (А 13 * В 33 )
  • Ряд 2
  • С 21 = (А 21 * В 11 ) + (А 22 * В 21 ) + (А 23 * В 31 )
  • С 22 = (А 21 * В 12 ) + (А 22 * В 22 ) + (А 23 * В 32 )
  • С 22 = (А 21 * В 13 ) + (А 22 * В 23 ) + (А 23 * В 33 )
  • Ряд 3
  • С 31 = (А 31 * В 11 ) + (А 32 * В 21 ) + (А 33 * В 31 )
  • С 32 = (А 31 * В 12 ) + (А 32 * В 22 ) + (А 33 * В 32 )
  • С 32 = (А 31 * В 13 ) + (А 32 * В 23 ) + (А 33 * Б 33 )

Теперь, увидев это, мы собираемся сделать пример того, как умножить две матрицы 3×3

Матрица A

2 3 1
7 4 1
9 -2 1

Матрица B

9 -2 -1
5 7 3
8 1 0

  • Теперь находим результирующую матрицу
  • Ряд 1
  • С 11 = (2*9) + (3*5) + (1*8)
  • С 11 = 18 + 15 + 8
  • С 11 = 41
  • С 12 = (2*-2) + (3*7) + (1*1)
  • С 12 = -4 + 21 + 1
  • С 12 = 18
  • С 13 = (2*-1) + (3*3) + (1*0)
  • С 13 = -2 + 9 + 0
  • С 13 = 7
  • Ряд 2
  • С 21 = (7*9) + (4*5) + (1*8)
  • С 21 = 63 + 20 + 8
  • С 21 = 91
  • С 22 = (7*-2) + (4*7) + (1*1)
  • С 22 = -14 + 28 + 1
  • С 22 = 15
  • С 23 = (7*-1) + (4*3) + (1*0)
  • С 23 = -7 + 12 + 0
  • С 23 = 5
  • Ряд 3
  • С 31 = (9*9) + (-2*5) + (1*8)
  • С 31 = 81 -10 + 8
  • С 31 = 79
  • С 32 = (9*-2) + (-2*7) + (1*1)
  • С 32 = -18 -14 + 1
  • С 32 = -31
  • С 33 = (9*-1) + (-2*3) + (1*0)
  • С 33 = -9 -6 + 0
  • С 33 = -15

Результирующая матрица

41 18 7
91 15 5
79 -31 -15

Примеры умножения матриц 3×3

Пример 1: Умножьте следующие матрицы 3×3.

Матрица А

5 5 0
2 2 1
3 3 2

Матрица В

0 -1 -1
-1 0 -1
0 0 -1

Теперь с матрицей, которую мы собираемся умножить, мы собираемся умножить каждую строку первой матрицы на каждый столбец второй матрицы, и таким образом мы найдем каждую позицию в результирующей матрице (C)

  • Сначала решаем первую строку
  • С 11 = (5*0) + (5*-1) + (0*0)
  • С 11 = 0 — 5 + 0
  • С 11 = -5
  • С 12 = (5*-1) + (5*0) + (0*0)
  • С 12 = -2 + 0 + 0
  • С 12 = -2
  • С 13 = (5*-1) + (5*-1) + (0*-1)
  • С 13 = -2 — 5 + 0
  • С 13 = -7
  • Теперь второй ряд
  • С 21 = (2*0)+(2*-1)+(1*0)
  • С 21 = 0 -2 0
  • С 21 = -2
  • С 22 = (2*-1)+(2*0)+(1*0)
  • С 22 = -2 + 0 + 0
  • С 22 = -2
  • С 23 = (2*-1)+(2*-1)+(1*-1)
  • С 23 = -2 — 2 -1
  • С 23 = -5
  • А теперь третий ряд
  • С 31 = (3*0)+(3*-1)+(2*0)
  • С 31 = 0 -3 + 0
  • С 31 = -3
  • С 32 = (3*-1)+(3*0)+(2*0)
  • С 32 = -3 + 0 + 0
  • С 32 = -3
  • С 33 = (3*-1)+(3*-1)+(2*-1)
  • С 33 = -3 — 3 -2
  • С 33 = -8

Результирующая матрица (C)

-5 -2 -7
-2 -2 -5
-3 -3 -8

Иисус любит тебя

Иисус — сын Божий, который был послан на смерть, чтобы каждый, кто верит в него, имел жизнь вечную.

График y tg4x: Найдите определения функции y=tg4x — ответ на Uchi.ru

Найти область определения функции y= tg4x. Является ли эта функция четной? — Знания.site

Последние вопросы

  • Русский язык

    58 секунд назад

    Разница работы ОЗУ
  • Русский язык

    1 минута назад

    Вставьте пропущенные буквы и объясните написание
  • Физика

    1 минута назад

    4. Как определяется индукция магнитного поля на оси бесконеч-но длинного соленоида?
  • Обществознание

    1 минута назад

    Верны ли следующие суждения о роли СМИ в политической системе общества?
  • Математика

    1 минута назад

    Формула (3. 5) при расчёте восстановительной стоимости объекта недвижимости это вообще как? Что надо сделать?
  • Физика

    1 минута назад

    5. В чем состоит метод магнетрона для определения ?
  • Английский язык

    1 минута назад

    Circle the correct words to complete the sentences. 21 I’d like four LETTUCE / LETTUCES please. 22 Are you interested WITH / IN Chinese cooking? 23 I want to read the MENU / TRAY to see what food I can order. 24 The CUSTOMER / CHEF is famous for his desserts. 25 This MELON / MELONS is very fresh. 26 You eat a lot of BREAD / BREADS! 27 Can we have the LAMBS / LAMB please? 28 The fridge is KEEN / FULL of food. ​
  • Литература

    1 минута назад

    1. Укажіть хронологічні межі доби Відродження Назвіть видатних художників, скульпторів, письменників цієї доби
  • Математика

    1 минута назад

    Розвяжи рівння а) x : 3/14 — 1/6 = -5/9; б) 1 8/15x +0,3=-7/15 срочно пж даю 50 б
  • Биология

    1 минута назад

    чому не можна близько нахилятись під час читання і писання​
  • Українська мова

    1 минута назад

    БУДЬ ЛАСКА ДОПОМОЖІТЬ! Напишіть короткий твір роздум за планом: 1. Дивний світ книг 2. Книги покращують ваш словниковий запас 3. Знайомство з різними героями 4. Книга допомагає людині в реальному житті 5. Читати книги — це цікаво і корисно
  • Информатика

    1 минута назад

    Срочно! 20 балів Символ «= =» в мові програмування Python має назву: а)запрошення б)нерівність в)дорівнює г)присвоєння
  • Английский язык

    1 минута назад

    Помогите пожалуйста, надо написать to be going to or Present Continues how much money/you/ spend on Saturday/? ? 2 my dad / look for a new job soon 3 1/ play tennis / with Erica at 2.30 today 4 when/you/ arrive back from holiday/? 5 I/try/ to do my homework this evening
  • Английский язык

    1 минута назад

    Read the text. Match paragraphs A–E with questions 1–5. Are celebrity chefs good for us? A Celebrity chefs are the new rock and roll! People are used to thinking of singers and actors as celebrities, but more and more kitchen chefs are becoming famous. In South Korea, they even have a special name for them – ‘cheftainers’. B Celebrity chefs become famous names when they make popular TV shows. They also write bestselling cookbooks. Some supermarkets use chefs to sell special food items and many chefs open chains of restaurants. Cooking is big money. UK chef Jamie Oliver and his wife are on the UK ‘Rich List’. C Celebrity chefs don’t just make money. They can make important changes to the way we think about food and the food we buy. Jamie Oliver is famous for his work on ‘food education’ and for helping to put healthy food on UK school menus. Jamie wants all school children to be able to have a hot, healthy meal during the day. He thinks it’s good for their health and also that good food helps children to learn better. D He also wants to change the way that adults eat. He understands that many people are ‘time poor’. A lot of parents work and don’t have time to cook big meals at the end of the day. His TV show 15-Minute Meals helps people prepare fresh, healthy meals in a small amount of time. E Not everyone is very keen on celebrity chefs though. Some people say that their food is actually less healthy than ready meals from supermarkets. For example, famous UK chef Nigella Lawson regularly uses much more butter and sugar than the cakes we find in shops. So choose your celebrity chef carefully – some of their food can be bad for you! In which paragraph does the writer … 29 describe how healthy eating and effective studying are connected? ___ 30 talk about people who can’t spend all day cooking? ___ 31 explain that some chefs use unhealthy items to cook with? ___ 32 introduce us to a new type of celebrity? ___ 33 describe the different ways to get rich from cooking? ___ ​
  • Математика

    1 минута назад

    Oblicz objętość i pole całkowite graniastosłupa prawidłowego trójkątnego o boku 3 cm i wysokości dwa razy dłuższej. z rozwiązaniem. ​

Все предметы

Выберите язык и регион

English

United States

Polski

Polska

Português

Brasil

English

India

Türkçe

Türkiye

English

Philippines

Español

España

Bahasa Indonesia

Indonesia

Русский

Россия

How much to ban the user?

1 hour 1 day 100 years

404 Cтраница не найдена

Мы используем файлы cookies для улучшения работы сайта МГТУ и большего удобства его использования. Более подробную информацию об использовании файлов cookies можно найти здесь. Продолжая пользоваться сайтом, вы подтверждаете, что были проинформированы об использовании файлов cookies сайтом ФГБОУ ВО «МГТУ» и согласны с нашими правилами обработки персональных данных.

Размер:

AAA

Изображения Вкл. Выкл.

Обычная версия сайта

К сожалению запрашиваемая страница не найдена.

Но вы можете воспользоваться поиском или картой сайта ниже

  • Университет

    Майкопский государственный технологический университет – один из ведущих вузов юга России.

    • История университета
    • Анонсы
    • Объявления
    • Медиа
      • Представителям СМИ
      • Газета «Технолог»
      • О нас пишут
    • Ректорат
    • Структура
      • Филиал
      • Политехнический колледж
      • Медицинский институт
        • Лечебный факультет
        • Педиатрический факультет
        • Фармацевтический факультет
        • Стоматологический факультет
        • Факультет послевузовского профессионального образования
      • Факультеты
      • Кафедры
    • Ученый совет
    • Дополнительное профессиональное образование
    • Бережливый вуз – МГТУ
      • Новости
      • Объявления
      • Лист проблем
      • Лист предложений (Кайдзен)
      • Реализуемые проекты
      • Архив проектов
      • Фабрика процессов
      • Рабочая группа «Бережливый вуз-МГТУ»
    • Вакансии
    • Профсоюз
    • Противодействие терроризму и экстремизму
    • Противодействие коррупции
    • WorldSkills в МГТУ
    • Научная библиотека МГТУ
    • Реквизиты и контакты
    • Управление имущественным комплексом
    • Опрос в целях выявления мнения граждан о качестве условий оказания образовательных услуг
    • Работа МГТУ в условиях предотвращения COVID-19
    • Документы, регламентирующие образовательную деятельность
    • Система менеджмента качества университета
    • Региональный центр финансовой грамотности
    • Аккредитационно-симуляционный центр
  • Абитуриентам
    • Подача документов онлайн
    • Абитуриенту 2023
    • Экран приёма 2022
    • Иностранным абитуриентам
      • Международная деятельность
      • Общие сведения
      • Кафедры
      • Новости
      • Центр международного образования
      • Академическая мобильность и международное сотрудничество
        • Академическая мобильность и фонды
        • Индивидуальная мобильность студентов и аспирантов
        • Как стать участником программ академической мобильности
    • Дни открытых дверей в МГТУ
      • День открытых дверей online
      • Университетские субботы
      • Дни открытых дверей на факультетах
    • Подготовительные курсы
      • Подготовительное отделение
      • Курсы для выпускников СПО
      • Курсы подготовки к сдаче ОГЭ и ЕГЭ
      • Онлайн-курсы для подготовки к экзаменам
      • Подготовка школьников к участию в олимпиадах
    • Малая технологическая академия
      • Профильный класс
        • Социально-экономический профиль
        • Медико-фармацевтический профиль
        • Инженерно-технологический профиль
        • Эколого-биологический профиль
        • Агротехнологический профиль
      • Индивидуальный проект
      • Кружковое движение юных технологов
      • Олимпиады, конкурсы, фестивали
    • Веб-консультации для абитуриентов и их родителей
      • Веб-консультации для абитуриентов
      • Родительский университет
    • Олимпиады для школьников
      • Отборочный этап
      • Заключительный этап
      • Итоги олимпиад
    • Профориентационная работа
    • Стоимость обучения
  • Студентам
    • Студенческая жизнь
      • Стипендии
      • Организация НИРС в МГТУ
      • Студенческое научное общество
      • Студенческие научные мероприятия
      • Конкурсы
      • Академическая мобильность и международное сотрудничество
    • Образовательные программы
    • Расписание занятий
    • Расписание звонков
    • Онлайн-сервисы
    • Социальная поддержка студентов
    • Общежития
    • Трудоустройство обучающихся и выпускников
      • Вакансии
    • Обеспеченность ПО
    • Инклюзивное образование
      • Условия обучения лиц с ограниченными возможностями
      • Доступная среда
    • Ассоциация выпускников МГТУ
    • Перевод из другого вуза
    • Вакантные места для перевода
    • Студенческое пространство
      • Студенческое пространство
      • Запись на мероприятия
    • Отдел по социально-бытовой и воспитательной работе
  • Наука и инновации
    • Научная инфраструктура
      • Проректор по научной работе и инновационному развитию
      • Научно-технический совет
      • Управление научной деятельностью
      • Управление аспирантуры и докторантуры
      • Точка кипения МГТУ
        • О Точке кипения МГТУ
        • Руководитель и сотрудники
        • Документы
        • Контакты
      • Центр коллективного пользования
      • Центр народной дипломатии и межкультурных коммуникаций
      • Студенческое научное общество
    • Новости
    • Научные издания
      • Научный журнал «Новые технологии»
      • Научный журнал «Вестник МГТУ»
      • Научный журнал «Актуальные вопросы науки и образования»
    • Публикационная активность
    • Конкурсы, гранты
    • Научные направления и результаты научно-исследовательской деятельности
      • Основные научные направления университета
      • Отчет о научно-исследовательской деятельности в университете
      • Результативность научных исследований и разработок МГТУ
      • Финансируемые научно-исследовательские работы
      • Объекты интеллектуальной собственности МГТУ
      • Результативность научной деятельности организаций, подведомственных Минобрнауки России (Анкеты по референтным группам)
    • Студенческое научное общество
    • Инновационная инфраструктура
      • Федеральная инновационная площадка
      • Проблемные научно-исследовательские лаборатории
        • Научно-исследовательская лаборатория «Совершенствование системы управления региональной экономикой»
        • Научно-исследовательская лаборатория проблем развития региональной экономики
        • Научно-исследовательская лаборатория организации и технологии защиты информации
        • Научно-исследовательская лаборатория функциональной диагностики (НИЛФД) лечебного факультета медицинского института ФГБОУ ВПО «МГТУ»
        • Научно-исследовательская лаборатория «Инновационных проектов и нанотехнологий»
      • Научно-техническая и опытно-экспериментальная база
      • Центр коллективного пользования
      • Научная библиотека
    • Экспортный контроль
    • Локальный этический комитет
    • Конференции
      • Международная научно-практическая конференция «Актуальные вопросы науки и образования»
      • VI Международная научно-практическая онлайн-конференция
    • Наука и университеты
  • Международная деятельность
    • Иностранным студентам
    • Международные партнеры
    • Академические обмены, иностранные преподаватели
      • Академическая мобильность и фонды
      • Индивидуальная мобильность студентов и аспирантов
    • Факультет международного образования
      • Новости факультета
      • Информация о факультете
      • Международная деятельность
      • Кафедры
        • Кафедра русского языка как иностранного
        • Кафедра иностранных языков
      • Центр Международного образования
      • Центр обучения русскому языку иностранных граждан
        • Приказы и распоряжения
        • Курсы русского языка
        • Расписание
      • Академическая мобильность
      • Контактная информация
    • Контактная информация факультета международного образования
  • Сведения об образовательной организации
    • Основные сведения
    • Структура и органы управления образовательной организацией
    • Документы
    • Образование
    • Образовательные стандарты и требования
    • Руководство. Педагогический (научно-педагогический) состав
    • Материально-техническое обеспечение и оснащённость образовательного процесса
    • Стипендии и меры поддержки обучающихся
    • Платные образовательные услуги
    • Финансово-хозяйственная деятельность
    • Вакантные места для приёма (перевода)
    • Международное сотрудничество
    • Доступная среда
    • Организация питания в образовательной организации

Мэтуэй | Популярные задачи

92
1 Найти точное значение грех(30)
2 Найти точное значение грех(45)
3 Найти точное значение грех(30 градусов)
4 Найти точное значение грех(60 градусов)
5 Найти точное значение загар (30 градусов)
6 Найти точное значение угловой синус(-1)
7 Найти точное значение грех(пи/6)
8 Найти точное значение cos(pi/4)
9 Найти точное значение грех(45 градусов)
10 Найти точное значение грех(пи/3)
11 Найти точное значение арктан(-1)
12 Найти точное значение cos(45 градусов)
13 Найти точное значение cos(30 градусов)
14 Найти точное значение желтовато-коричневый(60)
15 Найти точное значение csc(45 градусов)
16 Найти точное значение загар (60 градусов)
17 Найти точное значение сек(30 градусов)
18 Найти точное значение cos(60 градусов)
19 Найти точное значение cos(150)
20 Найти точное значение грех(60)
21 Найти точное значение cos(pi/2)
22 Найти точное значение загар (45 градусов)
23 Найти точное значение arctan(- квадратный корень из 3)
24 Найти точное значение csc(60 градусов)
25 Найти точное значение сек(45 градусов)
26 Найти точное значение csc(30 градусов)
27 Найти точное значение грех(0)
28 Найти точное значение грех(120)
29 Найти точное значение соз(90)
30 Преобразовать из радианов в градусы пи/3
31 Найти точное значение желтовато-коричневый(30)
32
35 Преобразовать из радианов в градусы пи/6
36 Найти точное значение детская кроватка(30 градусов)
37 Найти точное значение арккос(-1)
38 Найти точное значение арктан(0)
39 Найти точное значение детская кроватка(60 градусов)
40 Преобразование градусов в радианы 30
41 Преобразовать из радианов в градусы (2 шт. )/3
42 Найти точное значение sin((5pi)/3)
43 Найти точное значение sin((3pi)/4)
44 Найти точное значение тан(пи/2)
45 Найти точное значение грех(300)
46 Найти точное значение соз(30)
47 Найти точное значение соз(60)
48 Найти точное значение соз(0)
49 Найти точное значение соз(135)
50 Найти точное значение cos((5pi)/3)
51 Найти точное значение cos(210)
52 Найти точное значение сек(60 градусов)
53 Найти точное значение грех(300 градусов)
54 Преобразование градусов в радианы 135
55 Преобразование градусов в радианы 150
56 Преобразовать из радианов в градусы (5 дюймов)/6
57 Преобразовать из радианов в градусы (5 дюймов)/3
58 Преобразование градусов в радианы 89 градусов
59 Преобразование градусов в радианы 60
60 Найти точное значение грех(135 градусов)
61 Найти точное значение грех(150)
62 Найти точное значение грех(240 градусов)
63 Найти точное значение детская кроватка(45 градусов)
64 Преобразовать из радианов в градусы (5 дюймов)/4
65 Найти точное значение грех(225)
66 Найти точное значение грех(240)
67 Найти точное значение cos(150 градусов)
68 Найти точное значение желтовато-коричневый(45)
69 Оценить грех(30 градусов)
70 Найти точное значение сек(0)
71 Найти точное значение cos((5pi)/6)
72 Найти точное значение КСК(30)
73 Найти точное значение arcsin(( квадратный корень из 2)/2)
74 Найти точное значение загар((5pi)/3)
75 Найти точное значение желтовато-коричневый(0)
76 Оценить грех(60 градусов)
77 Найти точное значение arctan(-( квадратный корень из 3)/3)
78 Преобразовать из радианов в градусы (3 пи)/4 
79 Найти точное значение sin((7pi)/4)
80 Найти точное значение угловой синус(-1/2)
81 Найти точное значение sin((4pi)/3)
82 Найти точное значение КСК(45)
83 Упростить арктан(квадратный корень из 3)
84 Найти точное значение грех(135)
85 Найти точное значение грех(105)
86 Найти точное значение грех(150 градусов)
87 Найти точное значение sin((2pi)/3)
88 Найти точное значение загар((2pi)/3)
89 Преобразовать из радианов в градусы пи/4
90 Найти точное значение грех(пи/2)
91 Найти точное значение сек(45)
92 Найти точное значение cos((5pi)/4)
93 Найти точное значение cos((7pi)/6)
94 Найти точное значение угловой синус(0)
95 Найти точное значение грех(120 градусов)
96 Найти точное значение желтовато-коричневый ((7pi)/6)
97 Найти точное значение соз(270)
98 Найти точное значение sin((7pi)/6)
99 Найти точное значение arcsin(-( квадратный корень из 2)/2)
100 Преобразование градусов в радианы 88 градусов

Когда и где вы могли оказаться на съемочной площадке

Многие жители Нью-Йорка сталкивались со съемочной площадкой, но для тех, у кого нет, наше новейшее исследование данных покажет вам, куда идти, чтобы увеличить шансы встретить любимого актера/актрису. Используя данные о разрешениях на киносъемку за 2012-2016 годы, собранные Управлением кино, театра и вещания мэра из открытых данных Нью-Йорка, мы смогли определить сезонные тенденции и районы, в которых снималось больше всего телешоу и художественных фильмов (также известных как фильмы). Когда мы копались в данных, мы обнаружили устойчивый рост примерно на 15% за эти годы подано разрешений на телесъемку. Разрешения на художественные фильмы сильно различались, с большим падением разрешений в 2014 году. Это совпадает с падением годовых кассовых сборов, которое наблюдалось в 2014 году у Box Office Mojo.

Съемочная группа должна подать заявку на получение разрешения на съемку при использовании производственного оборудования помимо ручной камеры или камеры на штативе, запрашивая права на парковку и/или заявляя об исключительном использовании городской собственности, реквизитного оружия, транспортных средств, актеров в полицейской форме или трюки на съемочной площадке.

Разрешения рассматриваются и обрабатываются за 3–4 дня до начала съемки, за исключением съемок, расположенных на Таймс-сквер, где разрешения необходимо подавать не менее чем за 7 рабочих дней. Производственные бригады должны предоставить одно разрешение на дату съемки, независимо от количества съемочных площадок.

*Среднее количество разрешений на киносъемку, поданных для телешоу каждый месяц в году с использованием данных о разрешениях на киносъемку с 2012 по 2016 год

В зависимости от шоу и сети премьера телешоу обычно начинается осенью. Сериалы, рассчитанные на 13 серий, начинают снимать в июле и обычно заканчивают съемки до зимы. Точно так же 22-серийные телешоу (редко снимаются, но случаются) также начинают снимать примерно в июле, но продолжают снимать до весны с сезонным перерывом зимой на праздничный сезон.

Беглый взгляд на наш график показывает, что:

  • Октябрь — самый загруженный месяц для съемочных групп: в среднем в течение месяца происходит 403 съемок фильмов.
  • На май и июнь снимается меньше всего телешоу. В среднем в эти месяцы происходит всего 124-133 киносъемки. Обычно большинство съемочных групп ждут в режиме ожидания, пока сети объявят, какие телешоу они будут снимать.

* Среднее количество разрешений на съемку художественных фильмов, поданных за каждый месяц года с использованием данных о разрешениях на киносъемку с 2012 по 2016 год.

Хотя фильмы не ограничены теми же сроками производства в сети, что и телешоу, режиссеры сталкиваются с необходимостью завершить съемку до начала свалки в январе и феврале (иногда в августе и сентябре), когда коммерческие и критические ожидания относительно выхода новых фильмов ниже. . Как свидетельствуют данные, ясно, что в упомянутые месяцы свалки наблюдается резкий спад разрешений на съемку фильмов.

  • В среднем было снято 153 фильма, Июнь — месяц, когда вы увидите больше всего съемочных групп бегает по Нью-Йорку.
  • «
  • Октябрь» занимает второе место, в среднем снято 132 фильма.

Факторы, влияющие на место съемок фильма, включают: цену, сценарий, атмосферу района и пешеходное движение в этом районе. Беглый взгляд на карту показывает, что почтовые индексы, расположенные в Нижнем Манхэттене и вдоль Ист-Ривер в Бруклине, были очень желанными местами для продюсеров телешоу.

  • 10005 (Финансовый округ)
  • 11222 (Гринпойнт)
  • 10007 (Общественный центр)
  • 10075 (Верхний Ист-Сайд)
  • 10012 (Вест-Виллидж)

Интересный факт: Такие шоу, как Boardwalk Empire и Girls, снимались в Гринпойнте, Бруклин. Самая продолжительная криминальная драма на телевидении «Закон и порядок» в основном снимается на Фоли-сквер и 1 Police Plaza, расположенных в районе Гражданского центра в нижнем Манхэттене. Среди других популярных шоу — «Секс в большом городе» и «Белый воротничок».

Наиболее востребованные почтовые индексы для съемок фильмов в основном находятся на Манхэттене в Финансовом районе, Адской кухне и Пенсильванском вокзале.

  • 10005 (Финансовый отдел)
  • 10121/10119 (Пенсильванский вокзал)
  • 10041 (Мемориал ветеранов Вьетнама в Нью-Йорке/наконечник Фиди)
  • 10036 (Адская кухня)
  • 10006 (Всемирный торговый центр)

Забавный факт: Популярные фильмы, снятые в Нью-Йорке, включают «Волк с Уолл-Стрит», сериалы «Новый Человек-Паук», «Большой Коротышка» и т. д.

  • Мистер Робот: Адская Кухня, 25-я улица и 5-я авеню
  • Girls: Greenpoint в Бруклине, Silvercup Studios в Лонг-Айленд-Сити и Emmons Ave & Norstrand Ave в Бруклине
  • Закон и порядок: Foley Square, 1 Police Plaza, Chelsea Piers
  • Limitless: West 35-36th St & 7-8th Ave, Ridgewood in Queens и Waverly & 6th Ave
  • Elementary: 121st St и 453 West 23rd St в Нью-Йорке, 42-22 22nd St в Лонг-Айленд-Сити
  • Готэм: 630 Flushing Ave и Steiner Studios в Бруклине, Bay St & Vanderbilt Ave в Staten Island
  • Черный список: 1 Hanson Place, 11th Ave и West 20th St, Rugby & Beverly в Бруклине
  • Слепая зона: 16 Waverly Ave и 284 Clermont Ave в Бруклине и Ludlow St между Stanton & Rivington St
  • Оттенки синего: 62-27 Fresh Pond Rd и 15th Ave между 117-119th St в Queens и Broadway Stages в Бруклине
  • Оранжевый — это новый черный: Kaufman Astoria Studios и 39th Ave & 52nd St в Квинсе, а также Детский психологический центр Rockland в Оринджбурге
  • Нерушимая Кимми Шмидт: 79th St & Amsterdam, Crescent & Ditmars Blvd в Лонг-Айленд-Сити и Central Park West & West 80-84th St
  • Лицо, представляющее интерес: 116-я улица и Морнингсайд-авеню, Монро и Маркет-стрит и 18-я улица и 5-я авеню
  • Billions: Bingham Rd и West 252nd St в Бронксе, 8th Ave между Union & Berkley в Бруклине и West 4th St & Broadway
  • Внутри Эми Шумер: Западная 123-я улица и Ленокс-авеню, 30-я улица и 30-я авеню, а также Бликер-стрит и Кристофер-стрит,

Растущие стимулы в виде ежегодных скидок до 420 миллионов долларов США для киноиндустрии и бесплатной помощи полиции штата Нью-Йорк привели к росту растущего бизнеса по производству теле- и кинопродукции в штате.

Примеры программы на делфи: Страница не найдена

Уроки Delphi начинающим с нуля

   Уроки Delphi предназначены начинающим Delphi программистам. Обычно начинающим необходимо сначала изучить теорию, даже не осознавая, зачем нужна новая информация. Начинающим эти Уроки Delphi даются сразу с применением на практике. Эти Уроки Delphi дадут начинающим программистам в Delphi почувствовать, какой информации им не хватает для дальнейшего совершенствования в Delphi. Для профессионала правильный вопрос уже содержит в себе ответ. На множестве форумов и других Delphi-ресурсов Delphi профессионалы будут рады помочь Delphi начинающим советом. Поэтому есть лишь одно условие для успеха в программировании — нужно не только читать, но и думать самому. Хотя эти Уроки Delphi и ставят перед собой цель заменить вашу первую книгу, но всё же думать самому для работы в Delphi необходимо.

НАВИГАЦИЯ
















   Почему именно Delphi? В Delphi сделать первый шаг очень просто, она интуитивно понятна. Конечно, небольшому числу разработчиков по долгу службы нужны глубокие специфические знания, которые приходят со временем. А начинающим Delphi позволяет начать создавать программы сразу, не углубляясь в изучение внутренностей операционной системы, и даже собственной среды разработки. Поэтому программист может сразу сосредоточиться на логике работы будущей программы.

   Delphi — прекрасная система визуального объектно-ориентированного проектирования, одинаково радующая и новичков в программировании, и профессионалов. Начинающим Delphi позволяет сразу, с небольшими затратами времени и сил создавать прикладные программы, которые внешне неотличимы от программ, созданных профессионалами. А для опытного программиста Delphi открывает неограниченные возможности для создания сколь угодно сложных программ любого типа, в том числе, распределённых приложений, работающих с любыми базами данных.

   Более подробно о том, что такое Delphi и почему её стоит изучать, можно прочесть на специальной страничке и этом переводе статьи «Why Delphi? (Delphi For Beginners)»:

Эта статья о Delphi имеет цель объяснить, что такое Delphi и что он может сделать для вас.

   Прежде всего, я предполагаю, что вы умеете работать с компьютером, и знаете что такое программирование в целом. Ведь если это не так, вас здесь бы не было в любом случае! Скорее всего, у вас уже есть базовые знания в области программирования. По-моему, я прав! Теперь сядьте поудобнее, расслабьтесь и читайте дальше.

   Borland Delphi представляет собой средство разработки приложений для Microsoft Windows. Delphi является мощным и простым в использовании инструментом для создания автономных программ, обладающих графическим интерфейсом (GUI), или 32-битных консольных приложений (программ, которые не имеют графического интерфейса).

   В сочетании с Borland Kylix, программисты Delphi могут создавать из одного исходного текста приложения и для Windows и для Linux, и это открывает новые возможности и увеличивает потенциальную отдачу от усилий, вложенных в изучение Delphi. В Delphi используется кросс-платформенная библиотека компонентов CLX и визуальные дизайнеры для создания высокопроизводительных приложений для Windows, которые повторной компиляцей можно легко превратить в приложения для Linux.

   Delphi является первым языком программирования, обладающим простой в использовании средой для быстрой разработки приложений, разрушающей барьеры между языками высокого уровня, и языками, на низком уровне разговаривающими с системой на языке битов и байтов.

   При создании графического интерфейса приложений Delphi, у вас все возможности языка программирования Object Pascal, «завернутого» в среду RAD. Такие компоненты окна графического пользовательского интерфейса, как формы, кнопки и списки объектов, включены в состав Delphi. Это означает, что вам не нужно писать никакого кода при добавлении их в ваше приложение. Вы просто «кладёте» их на вашу Форму, как в графическом редакторе. Вы можете также добавить на Форму элементы управления ActiveX, для создания в считанные минуты специализированных программ таких, например, как веб-браузеры. Delphi позволяет разработчикам дизайна внедрять в интерфейс новые элементы и кодировать их события одним щелчком мыши.

   Delphi поставляется в различных конфигурациях, настроенных на потребности различных предприятий. В Delphi вы можете писать программы для Windows быстрее и легче, чем это было возможно раньше.

Паскаль
   Лучшим способом представить что такое Delphi является Object Pascal на основе визуальной среды разработки. Delphi основан на Object Pascal, языке, аналогичном объектно-ориентированному C++, а в некоторых случаях даже лучше. Для разработчиков не имеющих опыта работы в Паскале, Delphi имеет шаблоны своих структур на Паскале, что ускоряет процесс изучения языка.

   Компилятор Delphi упаковывает приложения в компактные исполняемые файлы, причем нет необходимости в громоздких библиотеках DLL — большое удобство, я должен сказать.

   Библиотека Visual Component Library (автономные бинарные части программного обеспечения, которые выполняют некоторые конкретные предопределенные функции), или VCL, Delphi является объектно-ориентированной базой. В этой богатой библиотеке вы найдете классы для таких визуальных объектов Windows как окна, кнопки и т.д., а также классы для пользовательских элементов управления таких как таймер и мультимедийный плеер, наряду с невизуальными объектами, такими как список строк, таблицы базы данных, потоки и т.д.

Базы данных
   Delphi может получать доступ ко многим типам баз данных. Используя BDE (Borland Database Engine — механизм доступа к базам данных), формы и отчеты, которые вы создаете, получают доступ к локальным базам данных, таким как Paradox и DBase, сетевых баз данных SQL Server, InterBase, также как и SysBase, и любые источники данных, доступные даже через ODBC (открытая связь с базами данных).

Итак, Delphi — прекрасная среда разработки Windows- и Linux-программ любого типа. Поэтому единственное, что вас должно уже сейчас интересовать — Как Начать Работу.

   Урок Delphi 1. Windows-программа нажатием одной кнопки!

   Итак, запустим Delphi. Перед нами четыре окошка. Вверху во вcю ширину экрана окно управления проектом и средой разработки — главное окно Delphi. При его сворачивании сворачиваются и все остальные. Слева — инспектор объектов. В нём задаются свойства составляющих нашу программу компонентов. И наконец, в центре одно над другим два окна, окно формы будущей программы и окно программной начинки. < перейти… >

   Урок Delphi 2. Компоненты Delphi — основа визуального проектирования!

   В пустой форме смысла нет. Наполнить её содержанием помогут компоненты Delphi. Они располагаются на главном окне, на нескольких вкладках. Все основные компоненты находятся на первых четырёх вкладках: Standard, Additional, Win32 и System. Их названия всплывают в виде подсказок при наведении мышки на пиктограммы. < перейти… >

   Урок Delphi 3. События Delphi

   Структура программы для Windows представляет собой набор подпрограмм, каждая из которых ответственна за обработку конкретного события и вызывается только при его получении. Удобство Delphi состоит в том, что мы избавлены от необходимости получать сообщения от Windows сами, Delphi это делает за нас. Каждый компонент имеет впечатляющий набор событий, на которые он может реагировать. Программист сам определяет, какие события в программе требуется обрабатывать. < перейти… >

   Урок Delphi 4. Переменные, константы и их типы

   Данные в компьютере можно рассматривать как ячейки памяти, имеющие свои имена (идентификаторы). Все данные в программе на языке Delphi должны быть описаны до их первого использования. И компилятор следит, чтобы в программе они использовались в соответствии с этим описанием, что позволяет избежать ошибок. < перейти… >

   Урок Delphi 5. Собственные и структурные типы данных

   При создании любой серьёзной программы не обойтись без дополнительных, более сложных, чем числа и строки, типов данных. В Delphi программист может для своих целей конструировать собственные типы данных. Чтобы ввести в программу (описать) новый тип данных, применяется оператор с ключевым словом type. < перейти… >

   Урок Delphi 6. Выражения и операторы Delphi

   Для создания приложений, реализующих сложную алгоритмическую логику, нужны средства управления ходом работы программы: Математически доказано, что любой алгоритм можно закодировать с помощью оператора присваивания, условного оператора и оператора цикла. < перейти… >
   В качестве примера рассматривается программа сортировки одномерного массива методом пузырька. Приводится скриншот программы и подробные комментарии. Также можно скачать проект целиком в виде архива.

   Урок Delphi 7. Работа с файлами в Delphi

   Начало. Работа с файлами важная вещь в любом языке программирования. Для начала нужно упомянуть компоненты, которые умеют работать с файлами, считывать и сохранять своё содержимое, строки типа String, в файл текстового формата. Это компоненты ListBox, ComboBox и Memo, расположенные на первой же вкладке палитры компонентов. < перейти… >
   Продолжение. В Delphi реализовано несколько способов прямой работы с файлами. Познакомимся с классическим способом, связанным с использованием файловых переменных. Прежде всего файл должен быть открыт. < перейти… >
   Окончание. То, что мы узнали в предыдущей части урока, позволяет работать с файлами по адресу, жёстко записанному в тексте программы. Мы же хотим просматривать любые файлы по нашему выбору. В Delphi есть компоненты, позволяющие в работающей программе осуществлять выбор файлов. < перейти… >
   Поиск файлов в Delphi производится в три этапа. На первом этапе функция FindFirst находит первый файл, удовлетворяющий критериям отбора. На втором этапе функция FindNext в цикле поочерёдно находит остальные файлы. На третьем этапе функция FindClose освобождает память, выделенную для осуществления поиска. < перейти… >
   Задачка. Теперь, пользуясь всем вышеизложенным материалом, можно ответить, например, на вопрос: «как средствами Delphi определить размер файла». Есть несколько альтернативных способов, как подсчитать размер файла с помощью Delphi. < перейти… >
   Список подпрограмм используемых при работе с файлами в Delphi. Содержит функции и процедуры создания, поиска, преобразования и удаления папок и файлов.

   Урок Delphi 8. Работа с дополнительными формами Delphi

   Редкая программа обходится одной формой. Delphi позволяет с лёгкостью создавать дополнительные формы, предоставляющие возможность, например, вести диалог с пользователем, принимать и выводить любую необходимую информацию. < перейти… >

   Урок Delphi 9. Создаём и используем подпрограммы Delphi

   Всё богатство стандартных процедур и функций Delphi не исчерпывает встающих перед программистом задач. Для выполнения повторяющихся операций с меняющимися данными Delphi программист может создавать собственные подпрограммы — процедуры и функции Delphi. < перейти… >
Пример. Вычисление факториала. Выполняя повторяющиеся участки кода, подпрограммы эффективно заменяют циклы. Классический пример — вычисление факториала числа, демонстрирующий также использование рекурсии. Этот проект также можно скачать.

   Урок Delphi 10. Исключительные ситуации в Delphi

   Исключительные ситуации в Delphi встречаются постоянно. Исключительная ситуация это такая ситуация, в результате которой генерируется ошибка, и выполнение программы прерывается. Например, деление на ноль — классический пример исключительной ситуации. Для контроля исключительных ситуаций ввода-вывода также могут применяться директивы компилятора {$I}. < перейти… >

   Урок Delphi 11. Обзор компонентов Delphi

   Применение компонентов Delphi позволяет избежать рутинного ручного кодирования. Компоненты Delphi охватывают практически все аспекты применения современных информационных технологий. Конечно, для работы в Delphi прежде всего требуется изучить базовые компоненты Delphi, которые требуются при подготовке практически любого приложения. < перейти… >
Страница Standart
   Изучение Delphi естественным образом начинается со страницы палитры компонентов Standart. На этой странице расположены стандартные для Windows интерфейсные элементы, такие как главное и всплывающее меню, кнопка, однострочный и многострочный редакторы, переключатели, метки, списки, и некоторые другие компоненты, которые применяются наиболее часто. Рассматривается пример на переопределение символов, вводимых в компонент Edit, что может использоваться в формах для ввода пароля.
Страница Additional
    На страницу палитры компонентов Additional помещены дополнительные компоненты, без некоторых из которых сегодня трудно представить программу для Windows: кнопки с дополнительными свойствами, таблицы, компоненты для размещения изображений и многие другие.
Страница Win32
   Страница палитры компонентов Win32 содержит компоненты, представляющие собой интерфейсные элементы для 32-разрядных операционных систем Windows 95/98/NT (В версии системы Delphi 2 эта страница называлась Win95). Использующие эти компоненты программы выглядят в стилистике последних версий операционных систем Windows.
Страница System
    На странице палитры компонентов System представлены компоненты, которые имеют различное функциональное назначение (например, Timer — очень важный в любой программе компонент), в том числе компоненты, поддерживающие стандартные для Windows технологии межпрограммного обмена данными OLE и DDE.

   Урок Delphi 12. Работа со строками Delphi

   Работа со строками Delphi позволяет извлечь из строки необходимую информацию и представить её в нужном виде. Delphi предоставляет весь спектр необходимых функций для работы со строками и преобразования строк Delphi в необходимые форматы. < перейти… >
   Продолжение. Использование списка строк. Список строк Delphi TStringList — это структура данных, напоминающая компонент ListBox, но не визуальная, а просто хранящая в памяти и имеющая свойства и методы для работы со строками типа TString. < перейти… >

   Урок Delphi 13. Создание интерфейса пользователя

   Создание интерфейса пользователя сводится к выбору из палитры компонентов необходимых для работы программы компонентов Delphi, служащих интерфейсом управления, а также интерфейсом отображения информации, и перенесению их на Форму с последующей компоновкой. < перейти… >

   Урок Delphi 14. Графика в Delphi

   Работа с графикой в Delphi это не только линии и рисунки, но также и и печать текстовых документов. Поэтому в Delphi работе с графикой нужно уделить немного времени. Работа с графикой в Delphi предполагает знакомство с канвой (Canvas) компонентов. Канва это тот холст, который умеет управлять состоянием каждого своего пиксела, и это свойство позволяет программисту отобразить то, что требуется. < перейти… >

   Урок Delphi 15. Многопоточность в Delphi

   Потоки в Delphi выполняют функцию имитации псевдопараллельной работы приложения. Как известно, для организации многозадачности операционная система выделяет каждому приложению, выполняющемуся в настоящий момент, определённые кванты времени, длина и количество которых определяется его приоритетом. Поэтому объём работы, который приложение может выполнить, определяется тем, сколько таких квантов оно сможет получить в единицу времени. < перейти… >

   Урок Delphi 16. Динамически создаваемые компоненты

   Динамически создаваемые компоненты — это компоненты, место в памяти под которые выделяется по мере необходимости в процессе работы приложения. Этим они и отличаются от компонентов, которые помещаются на Форму при проектировании приложения. Возможность создавать компоненты динамически это очень большое удобство для программиста. < перейти… >



   Новые Уроки Delphi будут появляться с течением времени. Кроме того, будут корректироваться уже готовые уроки с целью избавиться от всегда возможных неточностей и ошибок. Как говорится:


Последние добавления
Переменные — указатели в Delphi и их использование
Совместная работа StringGrid и Excel

Полезные ссылки
Форум по Delphi
Как это делается в Delphi
Примеры кода на Delphi
Статьи о Delphi
Учебник HTML
Решение задач в Delphi


Карта сайта

Исходники Pascal, Delphi — CyberGuru.ru

  • Исходники
  • Исходники Pascal, Delphi

Исходники программ на Delphi и Pascal. Приведены примеры кода для работы со стандартными классами Delphi, реализации стандартных алгоритмов, реализации пользовательского интерфейса, взаимодействия с базами данных, операционной системой и железом.

  • [Delphi] Базы данных. ADO

    Исходники программ на Pascal и Delphi для работы с базами данных с использованием ADO.

  • [Delphi] Пользовательский интерфейс. Меню

    Исходники программ на Pascal и Delphi с примерами использования меню.

  • [Delphi] Пользовательский интерфейс. Рисование и графика

    Исходники программ на Pascal и Delphi для работы с графикой и рисования на формах и элементах управления.

  • [Delphi] Система. Клавиатура

    Исходники программ на Pascal и Delphi для работы с клавиатурой.

  • [Delphi] Пользовательский интерфейс. Элементы управления

    Исходники программ на Pascal и Delphi с примерами использования элементов управления пользовательского интерфейса.

  • [Delphi] Система. Win32

    Исходники программ на Pascal и Delphi с примерами использования средств платформы Win32.

  • [Delphi] Базы данных. Interbase

    Исходники программ на Pascal и Delphi для работы с СУБД Interbase.

  • [Delphi] Работа с DLL

    Исходники программ на Pascal и Delphi с примерами использования динамических библиотек DLL.

  • [Delphi] Система. Принтеры

    Исходники программ на Pascal и Delphi для работы с принтерами.

  • [Delphi] Сети

    Примеры программ на Pascal и Delphi для работы с сетями и сетевыми ресурсами.

  • [Delphi] Пользовательский интерфейс. Окна и формы

    Исходники программ на Pascal и Delphi для работы с формами и отдельными их элементами.

  • [Delphi] Стандартные классы, VCL

    Исходники программ на Pascal и Delphi с примерами использования стандартных классов и VCL.

  • [Delphi] Система. Железо

    Исходники программ на Pascal и Delphi для работы с компьютерным железом.

  • [Delphi] Базы данных

    Исходники программ на Pascal и Delphi для работы с базами данных.

  • [Delphi] Алгоритмы

    Исходники программ на Pascal и Delphi с примерами реализации различных алгоритмов и структур данных.

  • [Delphi] Базы данных. BDE

    Исходники программ на Pascal и Delphi для работы с базами данных с использованием BDE.

  • [Delphi] Базы данных. Access

    Исходники программ на Pascal и Delphi для работы с СУБД Access.

  • [Delphi] Базы данных. ODBC

    Исходники программ на Pascal и Delphi с примерами использования ODBC для работы с базами данных.

  • [Delphi] Система

    Исходники программ на Pascal и Delphi с примерами взаимодействия с операционными системами, использования системных объектов.

  • [Delphi] Система. Файлы и папки

    Исходники программ на Pascal и Delphi для работы с файлами и папками.

  • [Delphi] Пользовательский интерфейс

    Исходники программ на Pascal и Delphi с примерами реализации пользовательского интерфейса.

  • [Delphi] Приложение

    Исходники программ на Pascal и Delphi для управления приложениями.

  • [Delphi] Технология COM

    Примеры программ на Pascal и Delphi для работы с технологиями СОМ, OLE, ActiveX

Язык программирования Delphi

. История | by Visualwebz

Среди сотен различных языков программирования Delphi выдержал испытание временем. Хотя он и не так популярен, как когда-то, он по-прежнему остается языком программирования, который предпочитают использовать многие программисты. Хотя Delphi, возможно, и не вернется в центр внимания, он по-прежнему имеет важное значение в истории для программистов и сегодня. Он был корнем многих известных приложений, таких как Skype, Hamachi, WinRAR, Space Rangers и многих других. Важно узнать, что такое Delphi, его история и почему программисты до сих пор предпочитают использовать его.

Во-первых, история Delphi относительно недавняя. Delphi был впервые выпущен в 1995 году как инструмент для создания приложений Windows. Это был первый в свое время язык, ориентированный на объектную ориентацию в приложениях. Это означает, что программное обеспечение больше сосредоточено на данных или объектах, чем на функциях и логике. Программа была разработана на основе Turbo Pascal, выпущенного в ноябре 1983 года компанией Borland. Создатель Turbo Pascal и Delphi Андерс Хейлсберг познакомил их с интегрированной средой разработки. Это означало, что программу было легко редактировать и исправлять. Программист может написать код, а программа его идентифицирует. Затем это позволит программисту перейти к точной строке ошибок, чтобы исправить их. В 19В 95 году Хейлсберг превратил Turbo Pascal в визуальный язык программирования: Delphi.

Происхождение названия «Delphi».

Происхождение Дельф очень сильно зависит от греческого местоположения. Дельфы были построены как древнее святилище в Греции. Посланные Зевсом орлы встретились на месте будущего расположения Дельф и были отмечены Зевсом камнем. Позже, в шестом и четвертом веках до нашей эры, Дельфийский оракул стал очень влиятельной фигурой. Он сделал много предсказаний, таких как экспедиция аргонавтов и Троянская война. Когда был разработан язык программирования Delphi, название было временным. Delphi был выбран из-за его близкой совместимости с другим языком, известным как Oracle. Андерс Хейлсберг считал, что это подходящее имя, основанное на греческом мифе.

Обзор языка программы

Delphi был создан для работы с приложениями Windows, macOS, iOS, Android и Linux. Это касается каждой крупной существующей платформы. Однако в первую очередь он был разработан для работы с приложениями на базе Windows. С момента своего появления Delphi уделяла особое внимание обратной совместимости. Это означает, что новые программы могут беспрепятственно работать с их преемниками.

Хотя, несмотря на упор на обратную совместимость, Delphi изо всех сил пытается не отставать. Хотя он близок к полной обратной совместимости, в последние годы этого не произошло. Отсутствие полной совместимости связано с тем, что программное обеспечение не поддерживается в целом. В новых версиях языка некоторые функции затрудняют обеспечение полной совместимости. Таким образом, хотя когда-то он был полностью обратно совместим, в настоящее время это не гарантируется.

Использование программы Delphi по-прежнему дает множество преимуществ. Несмотря на то, что он помечен как «устаревший», многие его аспекты все еще работают для некоторых. Одна из наиболее важных причин отказа от смены программного обеспечения заключается в том, что код требует минимального обслуживания. Это уменьшает потребность приложений в переходе на другое программное обеспечение и экономит деньги на его обслуживании. Благодаря экономии средств приложения, использующие Delphi, также очень быстро компилируются и могут запускаться с ограниченными ресурсами. Это означает, что приложения преобразуют программу в более низкий уровень кода, в котором она может быть выполнена.

Одной из упомянутых ранее функций является способность программного обеспечения выявлять ошибки. Несмотря на то, что сейчас эта функция относительно тривиальна, в 1995 году она была новаторской. Наличие программы, которая определяет, где находится ошибка, перенаправило внимание обратно на написание кода. Вместо того, чтобы тратить время на просеивание кода на наличие ошибок, Delphi предоставила программисту простоту. Можно было отредактировать программу, запустить ее и исправить ошибку. Сегодня он по-прежнему служит полезным инструментом для программистов, предпочитающих использовать Delphi.

Возвращаясь к корням Delphi, он был создан для поддержки приложений Windows. Windows широко используется во всем мире как одна из крупнейших операционных систем. Delphi был разработан для поддержки интерфейсов программирования приложений Windows. На рынке наблюдался высокий спрос на программное обеспечение, совместимое с разработкой приложений для Windows. Delphi смогла удовлетворить требования потребителей. Когда этот спрос переместился на Java и .NET, Delphi по-прежнему оставалась на рынке. Это может быть просто небольшой бонус Delphi, но он все еще так широко популярен благодаря поддержке Windows.

Еще одна замечательная особенность Delphi — плавный переход с C++, другим языком программирования. C++ — еще одна объектно-ориентированная программа, разработанная немного позже Pascal. Как и Delphi, C++ отлично подходит для преобразования программы на более низкий уровень. Подобное сходство — вот что делает эти два устройства совместимыми друг с другом. Конечно, C++ гораздо более популярен, чем Delphi, поскольку он является основой для многих других языков. Тем не менее, можно смешивать Delphi и C++.

Этот пример кода Delphi предназначен для создания таймера. Большая часть кода, написанного в Delphi, начинается с «var». При этом код всегда заканчивается сигнификатором: «конец»; Многие из них довольно сложны для понимания. Есть много фраз, которые нужно запомнить, наряду с особым использованием знаков препинания. Запятые и точки с запятой играют жизненно важную роль в обеспечении удобочитаемости кода.

Сравнение Delphi и Python

Многие функции Delphi отличаются от Python. Более примечательной особенностью является использование двух косых черт с пробелом между ними («/ /»). Это символизирует комментарии, которые программист делает, чтобы делать заметки или отделять свой код. В Python для добавления комментариев будет использоваться знак решетки («#»). Хотя разница невелика, она играет важную роль в том, как обрабатывается код. Наряду с этим употребление «конца»; или «конец». фраза сильно отличается от Python. Python не требует от программиста обозначения завершения сегмента кода. Фраза «конец»; означает только завершение определенного блока кода. Пока «конец». завершает всю программу.

Операторы if

На изображении выше показаны операторы if, используемые в Delphi. В Delphi операторы «если» немного сложнее, чем в Python. Несмотря на сложность, они имеют некоторые сходства в структуре. Эта программа не требует, чтобы программист использовал интерлиньяж после оператора if. Как показано на фото, после «тогда» отсутствует точка с запятой. В Python двоеточие должно быть помещено после оператора «if», как показано на рисунке ниже. Чтобы заменить использование двоеточия или точки с запятой, Delphi использует оператор «then». Двоеточие, указанное в коде Python, по существу имеет то же определение, что и «тогда». Это заставляет программу выполнять критерии, отображаемые под оператором «если».

В варианте операторов «если» в Python используется предложение «else if». Предложение «else if» фактически проверяет все выражения в блоке «if». Если блоки «if» и «else if» оба ложны, программа переходит к выполнению оператора «else». Вместо использования оператора «else if» Delphi использует другой оператор «if, else» под оператором «if».

Циклы

Циклы в Python и Delphi совсем не похожи. Хотя структура отчасти похожа, терминология и стиль кода сильно различаются. Наверху изображен пример цикла, написанного в Delphi. Последнее изображение представляет собой цикл, написанный на Python. В Delphi важно разрешить сегменту определять переменные. Есть сегменты для описания того, где код начинается, повторяется и заканчивается. В Python ясно, что он не так сильно разделен. Пока переменные определены, секции в Python разделены новыми строками. Фразы в Delphi, такие как «повторить» и «конец», указывают программе, что делать. В то время как в Python символы новой строки не служат этой цели. Скорее встроенный код в Python делает это за программиста. Python выполняет написанный код и команды и завершает работу, когда код завершен. Во-вторых, фраза «до выхода» — это то, чего нет в Python. Как указывалось ранее, Python продолжает выполнять код до тех пор, пока он не будет выполнен полностью. Delphi требует, чтобы программа была написана программистом прямо и лаконично.

Резюме

Delphi и Python — отличные языки программирования по-своему, но у них больше различий, чем сходства. С технической точки зрения, Python на самом деле имеет рекомендуемый объем памяти всего в один ГБ. Delphi имеет гораздо большую емкость хранилища, рекомендуемый объем хранилища составляет 2 ГБ. Ориентированная на пользователя совместимость Python значительно упрощает его изучение по сравнению с Delphi. Delphi — это программа, ориентированная на более продвинутых программистов. Python отлично подходит для начинающих, которые учатся писать и читать код. Как видно из предыдущих примеров, пробелы и минималистский подход к формулировке делают Python простым. В Delphi представлены новые символы, определения и фразы, которых нет в структуре Python. Python известен своей удобочитаемостью и легкостью для глаз. Delphi известна своей эффективностью и совместимостью со многими различными платформами.

Trends

Еще в 1995 году с выпуском Delphi программисты на базе Windows получили необходимое программное обеспечение. Он был не только эксклюзивным для Windows, но и Delphi также использовался со всеми другими основными платформами. К 2000-м с появлением .NET и Java Delphi стало труднее идти в ногу со временем. Многие застряли с одним программным обеспечением Delphi, но Java поставлялась с интегрированной средой разработки. Интегрированные среды разработки — это приложения, в которых все типы программирования могут выполняться в одном пространстве. IDE были разработаны, чтобы максимизировать производительность программиста. Хотя Delphi не предлагала этого, Delphi продолжала оставаться популярной среди многих пользователей. Это было программное обеспечение, которое предоставило пользователям множество встроенных функций. Такие функции, как редактор кода, визуальный дизайнер, отладчик и поддержка сторонних плагинов. Источники утверждают, что Delphi по-прежнему популярен среди давних пользователей. Проблема заключается в желании программистов изучать Delphi. В течение последних нескольких лет Embarcadero (нынешняя компания-владелец) начала проводить интенсивное обслуживание Delphi. С 2016 года компания выпускает обновления каждые 6 месяцев в попытке вернуть себе популярность.

Вывод

На протяжении более 25 лет компания Delphi преодолевала трудности и трудности. После конкуренции с Java и прохождения текущих обновлений он по-прежнему старается сделать все возможное для своих давних программистов. Из-за низких эксплуатационных расходов и обратной совместимости многие приложения продолжают использовать Delphi и сегодня. Имея короткую, но богатую историю, Delphi продолжает играть важную роль в жизни многих программистов и приложений. Возможно, он уже не в расцвете сил, но он стал поворотным моментом в мире программирования.

Примеры программ Delphi DLL и пример кода

Примеры программ Delphi DLL и пример кода GIF»>

Чилкат НАЧАЛЬНАЯ СТРАНИЦА .NET Core C# Android™ AutoIt С С# С++ Чилкат2-Питон КкПитон Классический АСП DataFlex Delphi ActiveX Делфи DLL Перейти Java Лянджа Моно С# Node.js Цель-C PHP ActiveX PHP-расширение Перл PowerBuilder PowerShell PureBasic Руби SQL-сервер Свифт 2 Свифт 3,4,5… Ткл Юникод С Юникод С++ VB.NET VBScript Visual Basic 6.0 Visual FoxPro Плагин Xojo

OpenSSL 9007 Outlook Contact
PDF Signatures
PEM
PFX/P12
PKCS11
POP3
PRNG
REST
REST Misc
RSA
SCP
SCard
SFTP
SMTP
SSH
SSH Key
SSH Tunnel
ScMinidriver
SharePoint
Socket/SSL/ TLS
Spider
Stream
Tar Archive
ULID/UUID
Upload
WebSocket
XAdES
XML
Цифровые подписи XML
XMP
Zip
curl

 

 

 

Примеры Delphi DLL

Категории веб-API

ASN.1
AWS Misc
Amazon EC2
Amazon Glacier
Amazon S3
Amazon S3 (новый)
Amazon SES
Amazon SNS
Amazon SQS
Async
Azure Service Cloud Storage Service 90 Azure 90907
Base64
Bounced Email
Box
CAdES
CSR
CSV
Certificates
Compression
DKIM / DomainKey
DSA
Diffie-Hellman
Digital Signatures
Dropbox
Dynamics CRM
EBICS
ECC
Ed25519
Email Object
Encryption
FTP
FileAccess
Firebase
GMail REST API
GMail SMTP/IMAP/POP
Геолокация
API Google
Календарь Google
Google Cloud SQL
Google Cloud Storage
Google Диск
Google Photos
Google Sheets
Google Tasks
Gzip
HTML-to-XML/Text
HTTP

HTTP Misc
IMAP
JSON
JSON Web Encryption 900 JSON 9JSON Web Encryption (JWE) 900 Маркер (JWT)
Java KeyStore (JKS)
Электронная почта MHT / HTML
MIME
Поставщики хранилища MS
Microsoft Graph
NTLM
OAuth2
OAuth3
OIDC
Office365 7 Outlook99SL

Щелкните категорию на левой панели, чтобы просмотреть примеры Delphi DLL.

Решить уравнение квадратное уравнение с дробями: Дробные рациональные уравнения — урок. Алгебра, 8 класс.

Решить уравнение в виде дроби – Telegraph

Решить уравнение в виде дроби

Скачать файл — Решить уравнение в виде дроби

Продолжаем разговор про решение уравнений. В этой статье мы подробно остановимся на рациональных уравнениях и принципах решения рациональных уравнений с одной переменной. Сначала разберемся, уравнения какого вида называются рациональными, дадим определение целых рациональных и дробных рациональных уравнений, приведем примеры. Дальше получим алгоритмы решения рациональных уравнений, и, конечно же, рассмотрим решения характерных примеров со всеми необходимыми пояснениями. В начале 8 класса на уроках алгебры начинается всестороннее изучение рациональных выражений. А вскоре, естественно, начинают встречаться уравнения, содержащие рациональные выражения в своих записях. Такие уравнения назвали рациональными. Сформулируем озвученную информацию в виде определения рациональных уравнений. Рациональные уравнения — это уравнения, обе части которого являются рациональными выражениями. Рациональными уравнениями называют уравнения, в левой части которого находится рациональное выражение, а в правой — нуль. Отталкиваясь от озвученных определений, приведем несколько примеров рациональных уравнений. Из показанных примеров видно, что рациональные уравнения, как, впрочем, и уравнения других видов, могут быть как с одной переменной, так и с двумя, тремя и т. В следующих пунктах мы будем говорить о решении рациональных уравнений с одной переменной. Решение уравнений с двумя переменными и их большим числом заслуживают отдельного внимания. Помимо деления рациональных уравнений по количеству неизвестных переменных, их еще разделяют на целые и дробные. Рациональное уравнение называют целым , если и левая, и правая его части являются целыми рациональными выражениями. Если хотя бы одна из частей рационального уравнения является дробным выражением, то такое уравнение называется дробно рациональным или дробным рациональным. Понятно, что целые уравнения не содержат деления на переменную, напротив, дробные рациональные уравнения обязательно содержат деление на переменную или переменную в знаменателе. Завершая этот пункт, обратим внимание на то, что известные к этому моменту линейные уравнения и квадратные уравнения являются целыми рациональными уравнениями. Одним из основных подходов к решению целых уравнений является их сведение к равносильным алгебраическим уравнениям. Это можно сделать всегда, выполнив следующие равносильные преобразования уравнения: В результате получается алгебраическое уравнение, которое равносильно исходному целому уравнению. Так в самых простых случаях решение целых уравнений сводятся к решению линейных или квадратных уравнений, а в общем случае — к решению алгебраического уравнения степени n. Для наглядности разберем решение примера. Сведем решение этого целого уравнения к решению равносильного ему алгебраического уравнения. И, во-вторых, преобразуем выражение, образовавшееся в левой части, в многочлен стандартного вида, выполнив необходимые действия с многочленами: Для полной уверенности выполним проверку найденных корней уравнения. Сначала проверяем корень 6 , подставляем его вместо переменной x в исходное целое уравнение: Степенью целого уравнения называют степень равносильного ему алгебраического уравнения. На этом можно бы было закончить с решением целых рациональных уравнений, если бы ни одно но…. Как известно, решение алгебраических уравнений степени выше второй сопряжено со значительными сложностями, а для уравнений степени выше четвертой вообще не существует общих формул корней. Поэтому для решения целых уравнений третьей, четвертой и более высоких степеней часто приходится прибегать к другим методам решения. В таких случаях иногда выручает подход к решению целых рациональных уравнений, основанный на методе разложения на множители. При этом придерживаются следующего алгоритма: Приведенный алгоритм решения целого уравнения через разложение на множители требует детального разъяснения на примере. Нахождение их корней по известным формулам корней через дискриминант не составляет труда, корни равны. Они являются искомыми корнями исходного уравнения. Для решения целых рациональных уравнений также бывает полезен метод введения новой переменной. В некоторых случаях он позволяет переходить к уравнениям, степень которых ниже, чем степень исходного целого уравнения. Сведение данного целого рационального уравнения к алгебраическому уравнению является, мягко говоря, не очень хорошей идеей, так как в этом случае мы придем к необходимости решения уравнения четвертой степени, не имеющего рациональных корней. Поэтому, придется поискать другой способ решения. Теперь переходим ко второй части метода введения новой переменной, то есть, к проведению обратной замены. По формуле корней квадратного уравнения находим корни первого уравнения. Вообще, когда мы имеем дело с целыми уравнениями высоких степеней, всегда надо быть готовым к поиску нестандартного метода или искусственного приема для их решения. Сначала будет полезно разобраться, как решать дробно рациональные уравнения вида , где p x и q x — целые рациональные выражения. А дальше мы покажем, как свести решение остальных дробно рациональных уравнений к решению уравнений указанного вида. В основе одного из подходов к решению уравнения лежит следующее утверждение: Этому заключению соответствует следующий алгоритм решения дробно рационального уравнения. Разберем пример применения озвученного алгоритма при решении дробного рационального уравнения. К решению дробного рационального уравнения можно подходить с немного другой позиции. То есть, можно придерживаться такого алгоритма решения дробно рационального уравнения: Во-вторых, находим ОДЗ переменной x для исходного уравнения. Остается проверить, входят ли найденные на первом шаге корни в ОДЗ. Следовательно, исходное дробно рациональное уравнение имеет два корня. Это связано с тем, что в таких случаях сделать проверку обычно проще, чем найти ОДЗ. Три из этих уравнений линейные и одно — квадратное, их мы умеем решать. С найденными корнями достаточно легко выполнить их проверку на предмет того, не обращается ли при них в нуль знаменатель дроби, находящейся в левой части исходного уравнения, а определить ОДЗ, напротив, не так просто, так как для этого придется решать алгебраическое уравнение пятой степени. Поэтому, откажемся от нахождения ОДЗ в пользу проверки корней. Найдите корни дробного рационального уравнения. Это уравнение равносильно совокупности двух уравнений: Проверять, не обращается ли в нуль знаменатель при найденных значениях x , достаточно неприятно. А определить область допустимых значений переменной x в исходном уравнении достаточно просто. Поэтому, будем действовать через ОДЗ. Еще полезным будет отдельно остановиться на случаях, когда в дробном рациональном уравнении вида в числителе находится число, то есть, когда p x представлено каким-либо числом. При этом если это число отлично от нуля, то уравнение не имеет корней, так как дробь равна нулю тогда и только тогда, когда ее числитель равен нулю; если это число нуль, то корнем уравнения является любое число из ОДЗ. Так как в числителе дроби, находящейся в левой части уравнения, отличное от нуля число, то ни при каких x значение этой дроби не может равняться нулю. Следовательно, данное уравнение не имеет корней. В числителе дроби, находящейся в левой части данного дробного рационального уравнения, находится нуль, поэтому значение этой дроби равно нулю для любого x , при котором она имеет смысл. Другими словами, решением этого уравнения является любое значение x из ОДЗ этой переменной. Осталось определить эту область допустимых значений. Таким образом, дробно рациональное уравнение имеет бесконечно много решений, которыми являются любые числа, кроме нуля и минус пяти. Наконец, пришло время поговорить о решении дробных рациональных уравнений произвольного вида. Забегая вперед, скажем, что их решение сводится к решению уравнений уже знакомого нам вида. Также мы знаем, что можно любое рациональное выражение преобразовать в рациональную дробь , тождественно равную этому выражению. Выявить и не включать в ответ посторонние корни можно, либо выполнив проверку, либо проверив их принадлежность ОДЗ исходного уравнения. Выполнить действия с дробями и многочленами в левой части уравнения, тем самым преобразовав ее в рациональную дробь вида. Выявить и исключить посторонние корни, что делается посредством их подстановки в исходное уравнение или посредством проверки их принадлежности ОДЗ исходного уравнения. Для большей наглядности покажем всю цепочку решения дробных рациональных уравнений: Давайте рассмотрим решения нескольких примеров с подробным пояснением хода решения, чтобы прояснить приведенный блок информации. Будем действовать в соответствии с только что полученным алгоритмом решения. И сначала перенесем слагаемые из правой части уравнения в левую, в результате переходим к уравнению. На втором шаге нам нужно преобразовать дробное рациональное выражение в левой части полученного уравнения к виду дроби. Для этого выполняем приведение рациональных дробей к общему знаменателю и упрощаем полученное выражение: Так мы приходим к уравнению. Для этого можно сделать проверку или найти ОДЗ переменной x исходного уравнения. Теперь покажем, как последний пункт алгоритма выполняется через ОДЗ. Нам требуется решить дробно рациональное уравнение, пройдем все шаги алгоритма. Во-первых, переносим слагаемое из правой части в левую, получаем. Во-вторых, преобразуем выражение, образовавшееся в левой части: На четвертом шаге остается выяснить, не является ли найденный корень посторонним для исходного дробно рационального уравнения. При его подстановке в исходное уравнение получается выражение. Очевидно, оно не имеет смысла, так как содержит деление на нуль. Откуда заключаем, что 0 является посторонним корнем. Следовательно, исходное уравнение не имеет корней. В заключение добавим, что совсем не обязательно слепо придерживаться приведенного алгоритма решения дробных рациональных уравнений, хотя он и является универсальным. Просто иногда другие равносильные преобразования уравнений позволяют прийти к результату быстрее и проще. Сначала отнять от обеих частей уравнения 7 , что приводит к уравнению. Отсюда можно заключить, что выражение в знаменателе левой части должно быть равно числу, обратному числу из правой части, то есть,. Теперь вычитаем из обеих частей тройки: По аналогии , откуда , и дальше. Проверка показывает, что оба найденных корня являются корнями исходного дробного рационального уравнения. Охраняется законом об авторском праве. Ни одну часть сайта www. Что такое рациональные уравнения? Решение дробно рациональных уравнений. Решите дробное рациональное уравнение. Его корень очевиден — это нуль. Но можно поступить и иначе, например, так.

Решение целых и дробно рациональных уравнений

Должностные инструкции инженер конструктор

Условные буквенные обозначенияв электрических схемах

Бесплатная помощь с домашними заданиями

Учебный план школыи планирование преподавания истории

Расписание электричек наро фоминск

Добрые стихи про кошек

Идеи для квартиры своими руками фото

Как решать уравнения с дробями

Повышенный шум при работе двигателя причины

Таблица энергозатрат человека при различных видах деятельности

Томаты палка описание

Решение дробных уравнений с преобразованием в квадратные уравнения

Характеристики ниссан альмера 2015

Тест драйв хендай h2 видео

Производственные процессы бывают

Как решать уравнения с дробями. Показательное решение уравнений с дробями

До сих пор мы решали только уравнения целые относительно неизвестного, то есть уравнения, в которых знаменатели (если таковые имелись) не содержали неизвестное.

Часто приходится решать уравнения, содержащие неизвестное в знаменателях: такие уравнения называются дробными.

Чтобы решить это уравнение, умножим обе его части на то есть на многочлен, содержащий неизвестное. Будет ли новое уравнение равносильно данному? Чтобы ответить на вопрос, решим это уравнение.

Умножив обе части его на , получим:

Решив это уравнение первой степени, найдём:

Итак, уравнение (2) имеет единственный корень

Подставив его в уравнение (1), получим:

Значит, является корнем и уравнения (1).

Других корней уравнение (1) не имеет. В нашем примере это видно, например, из того, что в уравнении (1)

Как неизвестный делитель должен быть равен делимому 1, разделённому на частное 2, то есть

Итак, уравнения (1) и (2) имеют единственный корень Значит, они равносильны.

2. Решим теперь такое уравнение:

Простейший общий знаменатель: ; умножим на него все члены уравнения:

После сокращения получим:

Раскроем скобки:

Приведя подобные члены, будем иметь:

Решив это уравнение, найдём:

Подставив в уравнение (1), получим:

В левой части получили выражения, не имеющие смысла.

Значит, корнем уравнения (1) не является. Отсюда следует, что уравнения (1) и неравносильны.

Говорят в этом случае, что уравнение (1) приобрело посторонний корень.

Сравним решение уравнения (1) с решением уравнений, рассмотренных нами раньше (см. § 51). При решении этого уравнения нам пришлось выполнить две такие операции, которые раньше не встречались: во-первых, мы умножили обе части уравнения на выражение, содержащее неизвестное (общий знаменатель), и, во-вторых, мы сокращали алгебраические дроби на множители, содержащие неизвестное.

Сравнивая уравнение (1) с уравнением (2), мы видим, что не все значения х, допустимые для уравнения (2), являются допустимыми для уравнения (1).

Именно числа 1 и 3 не являются допустимыми значениями неизвестного для уравнения (1), а в результате преобразования они стали допустимыми для уравнения (2). Одно из этих чисел оказалось решением уравнения (2), но, разумеется, решением уравнения (1) .оно быть не может. Уравнение (1) решений не имеет.

Этот пример показывает, что при умножении обеих частей уравнения на множитель, содержащий неизвестное, и при сокращении алгебраических дробей может получиться уравнение, неравносильное данному, а именно: могут появиться посторонние корни.

Отсюда делаем такой вывод. При решении уравнения, содержащего неизвестное в знаменателе, полученные корни надо проверять подстановкой в первоначальное уравнение. Посторонние корни надо отбросить.

Приглашаем тебя на урок о том, решать уравнения с дробями.Скорее всего, тебе уже приходилось сталкиваться с такими уравнениями в прошлом, так что на этом уроке нам предстоит повторить и обобщить те сведения, которые тебе известны.

Больше уроков на сайте

Дробно-рациональным называется уравнение, в котором есть рациональные дроби, то есть переменная в знаменателе. Скорее всего, тебе уже приходилось сталкиваться с такими уравнениями в прошлом, так что на этом уроке нам предстоит повторить и обобщить те сведения, которые тебе известны.

Сначала я предлагаю обратиться к предыдущему уроку данной темы – к уроку «Решение квадратных уравнений». На том уроке был рассмотрен пример решения дробно-рационального уравнения. Рассмотрим его

Решение этого уравнения выполнено в несколько этапов:

  • Преобразование уравнения, содержащего рациональные дроби.
  • Переход к целому уравнению и упрощение его;
  • Решение квадратного уравнения.

Через первые 2 этапа необходимо пройти при решении любого дробно-рационального уравнения. Третий этап – необязателен, так как уравнение, полученное в результате упрощений, может быть не квадратным, а линейным; решать линейное уравнение – намного проще. Есть еще один важный этап при решении дробно-рационального уравнения. Он будет виден при решении следующего уравнения.

что следует сделать в первую очередь? – Конечно же, привести дроби к общему знаменателю. И очень важным является найти именно наименьший общий знаменатель, иначе, далее, в процессе решения, уравнение будет усложнено. Тут заметим, что знаменатель последней дроби можно разложить на множители у и у+2 . Вот именно это произведение и будет общим знаменателем в данном уравнении. Теперь нужно определить дополнительные множители для каждой из дробей. Вернее, для последней дроби такой множитель не понадобится, так как ее знаменатель равен общему. Вот теперь, когда все дроби имеют одинаковые знаменатели, можно перейти к целому уравнению, составленному из одних числителей. Но необходимо cделать одно замечание, о том, что найденное значение неизвестной не может обращать в ноль ни один из знаменателей . Это – ОДЗ: у≠0, у≠2 . На этом окончен первый из описанных ранее этапов решения и переходим ко второму – упрощаем полученное целое уравнение. Для этого – раскрываем скобки, переносим все слагаемые в одну часть уравнения и приводим подобные. Выполни это самостоятельно и проверь – верны ли мои вычисления, в которых получено уравнение 3у 2 – 12у = 0. Это уравнение – квадратное, оно записано в стандартном виде, и один из его коэффициентов равен нулю.

Решение дробно-рациональных уравнений

Справочное пособие

Рациональные уравнения – это уравнения, в которых и левая, и правая части являются рациональными выражениями.

(Напомним: рациональными выражениями называют целые и дробные выражения без радикалов, включающие действия сложения, вычитания, умножения или деления — например: 6x; (m – n)2; x/3y и т.п.)

Дробно-рациональные уравнения, как правило, приводятся к виду:

Где P (x ) и Q (x ) – многочлены.

Для решения подобных уравнений умножить обе части уравнения на Q(x), что может привести к появлению посторонних корней. Поэтому, при решении дробно-рациональных уравнений необходима проверка найденных корней.

Рациональное уравнение называется целым, или алгебраическим, если в нем нет деления на выражение, содержащее переменную.

Примеры целого рационального уравнения:

5x – 10 = 3(10 – x)

3x
— = 2x – 10
4

Если в рациональном уравнении есть деление на выражение, содержащее переменную (x), то уравнение называется дробно-рациональным.

Пример дробного рационального уравнения:

15
x + — = 5x – 17
x

Дробные рациональные уравнения обычно решаются следующим образом:

1) находят общий знаменатель дробей и умножают на него обе части уравнения;

2) решают получившееся целое уравнение;

3) исключают из его корней те, которые обращают в ноль общий знаменатель дробей.

Примеры решения целых и дробных рациональных уравнений.

Пример 1. Решим целое уравнение

x – 1 2x 5x
— + — = —.
2 3 6

Решение:

Находим наименьший общий знаменатель. Это 6. Делим 6 на знаменатель и полученный результат умножаем на числитель каждой дроби. Получим уравнение, равносильное данному:

3(x – 1) + 4x 5х
—— = —
6 6

Поскольку в левой и правой частях одинаковый знаменатель, его можно опустить. Тогда у нас получится более простое уравнение:

3(x – 1) + 4x = 5х.

Решаем его, раскрыв скобки и сведя подобные члены:

3х – 3 + 4х = 5х

3х + 4х – 5х = 3

Пример решен.

Пример 2. Решим дробное рациональное уравнение

x – 3 1 x + 5
— + — = —.
x – 5 x x(x – 5)

Находим общий знаменатель. Это x(x – 5). Итак:

х 2 – 3х x – 5 x + 5
— + — = —
x(x – 5) x(x – 5) x(x – 5)

Теперь снова освобождаемся от знаменателя, поскольку он одинаковый для всех выражений. Сводим подобные члены, приравниваем уравнение к нулю и получаем квадратное уравнение:

х 2 – 3x + x – 5 = x + 5

х 2 – 3x + x – 5 – x – 5 = 0

х 2 – 3x – 10 = 0.

Решив квадратное уравнение, найдем его корни: –2 и 5.

Проверим, являются ли эти числа корнями исходного уравнения.

При x = –2 общий знаменатель x(x – 5) не обращается в нуль. Значит, –2 является корнем исходного уравнения.

При x = 5 общий знаменатель обращается в нуль, и два выражения из трех теряют смысл. Значит, число 5 не является корнем исходного уравнения.

Ответ: x = –2

Ещё примеры

Пример 1.

x 1 =6, x 2 = — 2,2.

Ответ:-2,2;6.

Пример 2.

Мы уже научились решать квадратные уравнения. Теперь распространим изученные методы на рациональные уравнения.

Что такое рациональное выражение? Мы уже сталкивались с этим понятием. Рациональными выражениями называются выражения, составленные из чисел, переменных, их степеней и знаков математических действий.

Соответственно, рациональными уравнениями называются уравнения вида: , где — рациональные выражения.

Раньше мы рассматривали только те рациональные уравнения, которые сводятся к линейным. Теперь рассмотрим и те рациональные уравнения, которые сводятся и к квадратным.

Пример 1

Решить уравнение: .

Решение:

Дробь равна 0 тогда и только тогда, когда ее числитель равен 0, а знаменатель не равен 0.

Получаем следующую систему:

Первое уравнение системы — это квадратное уравнение. Прежде чем его решать, поделим все его коэффициенты на 3. Получим:

Получаем два корня: ; .

Поскольку 2 никогда не равно 0, то необходимо, чтобы выполнялись два условия: . Поскольку ни один из полученных выше корней уравнения не совпадает с недопустимыми значениями переменной, которые получились при решении второго неравенства, они оба являются решениями данного уравнения.

Ответ: .

Итак, давайте сформулируем алгоритм решения рациональных уравнений:

1. Перенести все слагаемые в левую часть, чтобы в правой части получился 0.

2. Преобразовать и упростить левую часть, привести все дроби к общему знаменателю.

3. Полученную дробь приравнять к 0, по следующему алгоритму: .

4. Записать те корни, которые получились в первом уравнении и удовлетворяют второму неравенству, в ответ.

Давайте рассмотрим еще один пример.

Пример 2

Решить уравнение: .

Решение

В самом начале перенесем все слагаемые в левую сторону, чтобы справа остался 0. Получаем:

Теперь приведем левую часть уравнения к общему знаменателю:

Данное уравнение эквивалентно системе:

Первое уравнение системы — это квадратное уравнение.

Коэффициенты данного уравнения: . Вычисляем дискриминант:

Получаем два корня: ; .

Теперь решим второе неравенство: произведение множителей не равно 0 тогда и только тогда, когда ни один из множителей не равен 0.

Необходимо, чтобы выполнялись два условия: . Получаем, что из двух корней первого уравнения подходит только один — 3.

Ответ: .

На этом уроке мы вспомнили, что такое рациональное выражение, а также научились решать рациональные уравнения, которые сводятся к квадратным уравнениям.

На следующем уроке мы рассмотрим рациональные уравнения как модели реальных ситуаций, а также рассмотрим задачи на движение.

Список литературы

  1. Башмаков М.И. Алгебра, 8 класс. — М.: Просвещение, 2004.
  2. Дорофеев Г.В., Суворова С.Б., Бунимович Е.А. и др. Алгебра, 8. 5-е изд. — М.: Просвещение, 2010.
  3. Никольский С.М., Потапов М.А., Решетников Н.Н., Шевкин А.В. Алгебра, 8 класс. Учебник для общеобразовательных учреждений. — М.: Просвещение, 2006.
  1. Фестиваль педагогических идей «Открытый урок» ().
  2. School.xvatit.com ().
  3. Rudocs.exdat.com ().

Домашнее задание

§ 1 Целое и дробное рациональные уравнение

В этом уроке разберем такие понятия, как рациональное уравнение, рациональное выражение, целое выражение, дробное выражение. Рассмотрим решение рациональных уравнений.

Рациональным уравнением называют уравнение, в котором левая и правая части являются рациональными выражениями.

Рациональные выражения бывают:

Дробные.

Целое выражение составлено из чисел, переменных, целых степеней с помощью действий сложения, вычитания, умножения, а также деления на число, отличное от нуля.

Например:

В дробных выражениях есть деление на переменную или выражение с переменной. Например:

Дробное выражение не при всех значениях входящих в него переменных имеет смысл. Например, выражение

при х = -9 не имеет смысла, так как при х = -9 знаменатель обращается в нуль.

Значит, рациональное уравнение может быть целым и дробным.

Целое рациональное уравнение — это рациональное уравнение, в котором левая и правая части — целые выражения.

Например:

Дробное рациональное уравнение — это рациональное уравнение, в котором или левая, или правая части — дробные выражения.

Например:

§ 2 Решение целого рационального уравнения

Рассмотрим решение целого рационального уравнения.

Например:

Умножим обе части уравнения на наименьший общий знаменатель знаменателей входящих в него дробей.

Для этого:

1. найдем общий знаменатель для знаменателей 2, 3, 6. Он равен 6;

2. найдем дополнительный множитель для каждой дроби. Для этого общий знаменатель 6 делим на каждый знаменатель

дополнительный множитель для дроби

дополнительный множитель для дроби

3. умножим числители дробей на соответствующие им дополнительные множители. Таким образом, получим уравнение

которое равносильно данному уравнению

Слева раскроем скобки, правую часть перенесем налево, изменив знак слагаемого при переносе на противоположный.

Приведем подобные члены многочлена и получим

Видим, что уравнение линейное.

Решив его, найдем, что х = 0,5.

§ 3 Решение дробного рационального уравнения

Рассмотрим решение дробного рационального уравнения.

Например:

1.Умножим обе части уравнения на наименьший общий знаменатель знаменателей входящих в него рациональных дробей.

Найдем общий знаменатель для знаменателей х + 7 и х — 1.

Он равен их произведению (х + 7)(х — 1).

2.Найдем дополнительный множитель для каждой рациональной дроби.

Для этого общий знаменатель (х + 7)(х — 1) делим на каждый знаменатель. Дополнительный множитель для дроби

равен х — 1,

дополнительный множитель для дроби

равен х+7.

3.Умножим числители дробей на соответствующие им дополнительные множители.

Получим уравнение (2х — 1)(х — 1) = (3х + 4)(х + 7), которое равносильно данному уравнению

4.Слева и справа умножим двучлен на двучлен и получим следующее уравнение

5. Правую часть перенесем налево, изменив знак каждого слагаемого при переносе на противоположный:

6.Приведем подобные члены многочлена:

7.Можно обе части разделить на -1. Получим квадратное уравнение:

8.Решив его, найдем корни

Так как в уравнении

левая и правая части — дробные выражения, а в дробных выражениях при некоторых значениях переменных знаменатель может обратиться в нуль, то необходимо проверить, не обращается ли в нуль при найденных х1 и х2 общий знаменатель.

При х = -27 общий знаменатель (х + 7)(х — 1) не обращается в нуль, при х = -1 общий знаменатель также не равен нулю.

Следовательно, оба корня -27 и -1 являются корнями уравнения.

При решении дробного рационального уравнения лучше сразу указать область допустимых значений. Исключить те значения, при которых общий знаменатель обращается в нуль.

Рассмотрим еще один пример решения дробного рационального уравнения.

Например, решим уравнение

Знаменатель дроби правой части уравнения разложим на множители

Получим уравнение

Найдем общий знаменатель для знаменателей (х — 5), х, х(х — 5).

Им будет выражение х(х — 5).

теперь найдем область допустимых значений уравнения

Для этого общий знаменатель приравняем к нулю х(х — 5) = 0.

Получим уравнение, решив которое, найдем, что при х = 0 или при х = 5 общий знаменатель обращается в нуль.

Значит, х = 0 или х = 5 не могут быть корнями нашего уравнения.

Теперь можно найти дополнительные множители.

Дополнительным множителем для рациональной дроби

дополнительным множителем для дроби

будет (х — 5),

а дополнительный множитель дроби

Числители умножим на соответствующие дополнительные множители.

Получим уравнение х(х — 3) + 1(х — 5) = 1(х + 5).

Раскроем скобки слева и справа, х2 — 3х + х — 5 = х + 5.

Перенесем слагаемые справа налево, изменив знак переносимых слагаемых:

Х2 — 3х + х — 5 — х — 5 = 0

И после приведения подобных членов получим квадратное уравнение х2 — 3х — 10 = 0. Решив его, найдем корни х1 = -2; х2 = 5.

Но мы уже выяснили, что при х = 5 общий знаменатель х(х — 5) обращается в нуль. Следовательно, корнем нашего уравнения

будет х = -2.

§ 4 Краткие итоги урока

Важно запомнить:

При решении дробных рациональных уравнений надо поступить следующим образом:

1.Найти общий знаменатель дробей входящих в уравнение. При этом если знаменатели дробей можно разложить на множители, то разложить их на множители и затем найти общий знаменатель.

2.Умножить обе части уравнения на общий знаменатель: найти дополнительные множители, умножить числители на дополнительные множители.

3.Решить получившееся целое уравнение.

4.Исключить из его корней те, которые обращают в нуль общий знаменатель.

Список использованной литературы:

  1. Макарычев Ю.Н., Н. Г. Миндюк, Нешков К.И., Суворова С.Б. / Под редакцией Теляковского С.А. Алгебра: учебн. для 8 кл. общеобразоват. учреждений. — М.: Просвещение, 2013.
  2. Мордкович А.Г. Алгебра. 8 кл.: В двух частях. Ч.1: Учеб. для общеобразоват. учреждений. — М.: Мнемозина.
  3. Рурукин А. Н. Поурочные разработки по алгебре: 8 класс.- М.: ВАКО, 2010.
  4. Алгебра 8 класс: поурочные планы по учебнику Ю.Н. Макарычева, Н.Г. Миндюк, К.И. Нешкова, С.Б. Суворовой / Авт.-сост. Т.Л. Афанасьева, Л.А. Тапилина. -Волгоград: Учитель, 2005.

РЕШЕНИЕ КВАДРАТИЧНЫХ УРАВНЕНИЙ


Примечание:

  • Квадратное уравнение — это полиномиальное уравнение второй степени.
  • U-образный график квадратичного уравнения называется параболой.
  • Квадратное уравнение имеет два решения. Либо два различных реальных решения, одно двойное действительное решение или два мнимых решения.
  • Существует несколько методов решения квадратного уравнения:
    1. Факторинг
    2. Завершение квадрата
    3. Квадратичная формула
    4. График
  • Все методы начинаются с установки уравнения равным нулю.


Найдите x в следующем уравнении.

Пример 1:

Уравнение уже обнулено.

Если вы забыли, как обращаться с дробями, нажмите «Дроби» для обзора.

Удалите все дроби, записав уравнение в эквивалентной форме без дробных коэффициентов. В этой задаче это можно сделать умножив обе части уравнения на 2,


Метод 1: Факторинг

Уравнение не легко факторизуется. Поэтому, мы не будем использовать этот метод.


Способ 2: Заполнение квадрата

Добавьте 10 к обеим частям уравнения


Добавьте к обеим частям уравнения:


Фактор левой части и упрощение правой части:


Возьмите квадратный корень из обеих частей уравнения:


Добавьте 16 к обеим частям уравнения:


Метод 3: квадратичная формула

Квадратичная формула

В уравнении a есть коэффициент члена, b – коэффициент члена x , а c — константа. Замените 1 на на , -32 на на b и -10 на c на квадратичную формулу и упростить.


Метод 4: График

Изобразите левую часть уравнения, и постройте график правой части уравнения. График представляет собой не что иное, как ось x. Итак, что вы будете искать где график пересекает ось х. Другой способ сказать это состоит в том, что x-перехваты являются решения этого уравнения.

На графике видно, что есть два пересечения по оси x, одно в точке 32,309506 и один на -0,309506.

Ответы 32.309506 и Эти ответы могут или могут не быть решениями исходных уравнений. Вы должны убедиться, что эти ответы — решения.


Проверьте эти ответы в исходном уравнении.


Проверьте решение x =32,309506, подставив 32,309506 в оригинал уравнение для х. Если левая часть уравнения

равна правой часть уравнения после замены, вы нашли правильный отвечать.

  • Левая сторона:
  • Правая сторона:
Так как левая часть исходного уравнения равна правой части исходное уравнение после того, как мы подставим значение 32,309506 вместо x, тогда x = 32,309506 — это решение.

Проверьте решение x = -0,309506, заменив -0,309506 в оригинале уравнение для х. Если левая часть уравнения равна правой часть уравнения после замены, вы нашли правильный отвечать.

  • Левая сторона:
  • Правая сторона:
Так как левая часть исходного уравнения равна правой части исходное уравнение после того, как мы подставим значение -0,309506 вместо x, тогда х = — 0,309506 — это решение.


Решения уравнения составляют 32,309506 и — 0,309506.


Комментарий: Вы можете использовать точные решения, чтобы разложить исходное уравнение на множители.


С


Начиная с


Продукт


Начиная с


тогда мы могли бы сказать



However the product of the first terms of the factors does not equal


Multiply


Let’s check to see if







The факторы являются , и


Если вы хотите работать с другим примером, нажмите «Пример».


Если вы хотите проверить себя, решив некоторые задачи, подобные этой например, нажмите Проблема


Если вы хотите вернуться к содержанию уравнения, нажмите Содержание

[Алгебра] [Тригонометрия]
[Геометрия] [Дифференциальные уравнения]
[Исчисление] [Комплексные переменные] [Матричная алгебра]

Домашняя страница S.O.S MATHematics


Вам нужна дополнительная помощь? Пожалуйста, разместите свой вопрос на нашем S.O.S. Математика CyberBoard.

Автор: Нэнси Маркус

Решение квадратных уравнений

  Учебники по алгебре!
   
jpg»>  
года
 
Воскресенье 19число марта
 
   
Дом
Вычисления с отрицательными числами
Решение линейных уравнений
Системы линейных уравнений
Решение линейных уравнений графически
Алгебра Выражения
Вычисление выражений и решение уравнений
Правила дробей
Факторинг квадратных трехчленов
Умножение и деление дробей
Деление десятичных дробей на целые числа
Сложение и вычитание радикалов
Вычитание дробей
Разложение многочленов на множители по группам
Наклоны перпендикулярных линий
Линейные уравнения
Корни — Радикалы 1
График линии
Сумма корней квадратного числа
Написание линейных уравнений с использованием наклона и точки
Факторинг трехчленов со старшим коэффициентом 1
Написание линейных уравнений с использованием наклона и точки
Упрощение выражений с отрицательными показателями
Решение уравнений 3
Решение квадратных уравнений
Родительские и семейные графики
Сбор похожих терминов
-й Корни
Степень частного свойства показателей
Сложение и вычитание дробей
Проценты
Решение линейных систем уравнений методом исключения
Квадратичная формула
Дроби и смешанные числа
Решение рациональных уравнений
Умножение специальных биномов
Округление чисел
Факторинг по группировке
Полярная форма комплексного числа
Решение квадратных уравнений
Упрощение сложных дробей
Алгебра
Общие журналы
Операции с числами со знаком
Умножение дробей в общем
Разделение многочленов
Многочлены
Высшие степени и переменные показатели
Решение квадратных неравенств с помощью графика знаков
Написание рационального выражения в минимальных терминах
Решение квадратных неравенств с помощью графика знаков
Решение линейных уравнений
Квадрат бинома
Свойства отрицательных показателей
Обратные функции
дроби
Вращение эллипса
Умножение чисел
Линейные уравнения
Решение уравнений с одним логарифмическим членом
Объединение операций
Эллипс
Прямые линии
Графическое отображение неравенств с двумя переменными
Решение тригонометрических уравнений
Сложение и вычитание дробей
Простые трехчлены как произведения двучленов
Соотношения и пропорции
Решение уравнений
Умножение и деление дробей 2
Рациональные числа
Разность двух квадратов
Разложение многочленов на множители по группам
Решение уравнений, содержащих рациональные выражения
Решение квадратных уравнений
Деление и вычитание рациональных выражений
Квадратные корни и действительные числа
Порядок действий
Решение нелинейных уравнений подстановкой
Формулы расстояния и средней точки
Линейные уравнения
График с использованием точек пересечения x и y
Свойства показателей степени
Решение квадратных уравнений
Решение одношаговых уравнений с использованием алгебры
Относительно простые числа
Решение квадратного неравенства с двумя решениями
Квадратика
Операции над радикалами
Факторизация разности двух квадратов
Прямые линии
Решение квадратных уравнений с помощью факторинга
Графики логарифмических функций
Упрощение выражений, включающих переменные
Добавление целых чисел
Десятичные числа
Факторинг полностью общих квадратных трехчленов
Использование шаблонов для умножения двух двучленов
Сложение и вычитание рациональных выражений с отличающимися знаменателями
Рациональные показатели
Горизонтальные и вертикальные линии
   

Вы научились решать квадратные уравнения четырьмя различными способами: свойство четных корней, разложение на множители, завершение квадрата и квадратичная формула. Свойство четных корней и факторинг ограничены некоторыми специальными уравнениями, но вы должны использовать эти методы, когда это возможно. Любое квадратное уравнение можно решить путем заполнения квадрата или с помощью квадратичной формулы. Поскольку квадратичная формула обычно быстрее, используется чаще, чем заполнение квадрата. Однако, завершение квадрата является важным навыком для изучения.

 

Методы решения ax 2 + bx + c = 0

Метод Комментарии Примеры
Свойство четных корней Используйте, когда b = 0. (х — 2) 2 = 8

х — 2 = ±

Факторинг Используйте, когда полином можно разложить на множители.

Объем тетраэдра через смешанное произведение: Объем треугольной пирамиды (тетраэдра), построенной на векторах онлайн

2.10. Геометрический смысл смешанного произведения. Задачи, решаемые с помощью смешанного произведения

2.10. Геометрический смысл смешанного произведения. Задачи, решаемые с помощью смешанного произведения Высшая математика > 2. Векторная алгебра > 2.10. Геометрический смысл смешанного произведения. Задачи, решаемые с помощью смешанного произведения

  Геометрический смысл смешанного произведения
, где  — угол между векторами  и .
Построим на векторах  параллелепипед (рис. 16 a) и покажем, что смешанное произведение  равно модулю объема этого параллелепипеда. Так как  равно площади параллелограмма, построенного на векторах  или площади основания параллелепипеда, построенного на векторах , а — проекция вектора  на вектор  или высота h параллелепипеда, то .
Рис. 16 a                                                   Рис. 16 b
Рис. 16 a сделан для векторов, которые образуют правую тройку, в этом случае угол между векторами  и  острый и .
Если тройка векторов  — левая (рис. 16 b), то угол между векторами  и >90o. В этом случае и тогда . Поэтому . При любом расположении векторов .
Геометричеcкий смысл смешанного произведения используют при вычислении объемом параллелепипедов и тетраэдров, построенных на трех векторах. Формула объема параллелепипеда уже получена .
Выведем формулу объема тетраэдра (четырехугольной пирамиды), построенной на векторах .
Рис. 17
Из рис. 17 легко видеть, что, где  — площадь треугольника в основании тетраэдра, а h — его высота. Поскольку площадь треугольника , где — площадь параллелограмма, построенного на векторах  и , а высоты тетраэдра и параллелепипеда, построенных на векторах  совпадают, то
.
  Пример.

 Найти объем тетраэдра , если , , , .

  Решение.

  Тетраэдр построен на векторах

, (Рис. 2.17).

. Вычислим

=.

Тогда .

  Ответ.

.


 
gif»>

Определение смешанного произведения



Смешанное произведение векторов – это произведение трёх векторов пространства:

Вот так вот они выстроились паровозиком и ждут, не дождутся, когда их вычислят.

Определение: смешанным произведением  некомпланарных векторов , взятых в данном порядке, называется объём параллелепипеда, построенного на данных векторах, снабжённый знаком «+», если базис  правый, и знаком «–», если базис  левый.

Выполним рисунок, и ниже я снова подробно разберу определение:

1) Исходные векторы , обозначенные красными стрелками, не компланарны (со случаем компланарности разберёмся отдельно)

2) Векторы  взяты в определённом порядке, то есть перестановка векторов в произведении , как вы догадываетесь, не проходит без последствий.

3) Смешанное произведение векторов является ЧИСЛОМ: . В учебной литературе оформление может быть несколько другим, я привык обозначать смешанное произведение через , а результат вычислений буквой «пэ».

По определению, смешанное произведение – это объём параллелепипеда, построенного на векторах  (фигура прочерчена красными векторами и линиями чёрного цвета; невидимые нам линии изображены пунктиром). То есть, число  равно объему данного параллелепипеда.

Примечание: чертёж является схематическим.

4) Не будем заново «париться» с понятием ориентации базиса и пространства. Смысл заключительной части определения состоит в том, что к объёму  может добавляться знак минус. Простыми словами, смешанное произведение может быть отрицательным: .

Непосредственно из  определения следует формула вычисления объема параллелепипеда, построенного на векторах :

Знак модуля уничтожает возможный «минус» смешанного произведения.

И ещё одна важная формула. В курсе геометрии доказано, что объём тетраэдра (на рисунке отсечён «синей» плоскостью) равен одной шестой объёма параллелепипеда:

Тетраэдр часто называют треугольной пирамидой, поскольку все грани тетраэдра – треугольники.

Теперь случай компланарности. Если векторы  компланарны, то их можно расположить в одной плоскости. В результате параллелепипед «складывается» в плоскость, и объём такого вырожденного параллелепипеда равен нулю: .

1.10.2. Как вычислить смешанное произведение?

1.9.3. Векторное произведение в координатах

| Оглавление |



Автор: Aлeксaндр Eмeлин


векторов — Объем тетраэдра с использованием перекрестного и скалярного произведения

Задавать вопрос

спросил

Изменено 6 месяцев назад

Просмотрено 96 тысяч раз

$\begingroup$

Рассмотрим тетраэдр на изображении: Докажите, что объем тетраэдра равен $\frac16 |a \times b \cdot c|$.

Я знаю, что объем тетраэдра равен произведению площади основания на высоту, а здесь высота равна $h$, а площадь основания я считаю площадью треугольника $BCD$.

Итак, что у меня есть:

$$\begin{align} \text{базовая площадь} &= \frac12 \lvert a \times b \rvert \\ \text{высота $h$} &= \lvert c\rvert \cos \theta \end{выравнивание}$$

Таким образом, объем равен $$V=\frac12 \lvert a \times b\rvert \cdot \lvert c\rvert \cos \theta $$

Но я не знаю, как из этого получить $\frac16 | a \times b \cdot c|$.

Пожалуйста, сообщите.

  • векторы
  • объем
  • перекрестное произведение

$\endgroup$

3

$\begingroup$

Вот один из способов думать об этом. Тетраэдр — это $\dfrac{1}{6}$ объема параллелепипеда, образованного $\vec{a},\vec{b},\vec{c}$. Объем параллелепипеда равен скалярному тройному произведению $|(a \times b) \cdot c|$. Таким образом, объем тетраэдра $\dfrac{1}{6} |(a \times b) \cdot c|$

Чтобы решить вопрос так, как вы пытаетесь, обратите внимание, что $V = \dfrac{1}{3}Bh = \dfrac{1}{6}||a \times b|| \cdot ч$. Тогда $h = ||c|| \cdot |\cos(\theta)|$. Таким образом, имеем $V = \dfrac{1}{6}||a \times b|| \cdot ||с|| \cdot |\cos(\theta)|$. Теперь видим, что $|c \cdot (a \times b)| = ||с|| \cdot ||(a \times b)|| \cdot |\cos(\theta)|$ и, таким образом, $V = \dfrac{1}{6}|(a \times b) \cdot c|$.

Я также хотел бы сказать, что используемые вами обозначения немного странные. Во избежание путаницы $|x|$ обозначает абсолютное значение, а $||x||$ обозначает величину.

$\endgroup$

2

$\begingroup$

Подсказка: $\mathbf{a}\times\mathbf{b}$ «указывает прямо вверх». Следовательно, она параллельна показанной вами линии $h$. Следовательно, он имеет с $c$ тот же угол, что и угол $h$.

$\endgroup$

$\begingroup$

Объем будет (площадь основания) x 1/3 высоты —————-(1) теперь в этом случае, если мы возьмем за основание треугольник BCD, его площадь будет равна

= 1/2 |a||b||sin(m)| = 1/2 |а х б| ———————(2)

где m — угол между a и b, следовательно, b sin(m) — длина проекции b перпендикулярно а , т. е. высоте треугольника BCD, если мы возьмем а за его основание.

теперь высота тетраэдра будет проекцией c перпендикулярно основанию единичный вектор, перпендикулярный основанию BCD, равен = (a x b)/|a x b| = n

, следовательно, высота тетраэдра (h), которая может быть выражена как проекция c на n

h = |c||n||cos(θ)| = c.n = c.(a x b)/|a x b| ————-(3)

из уравнений 1,2 и 3 получаем

объем = 1/2|a x b| 1/3[c. (a x b)]/|a x b| = 1/6[c.(a x b)] = 1/6[(a x b).c]

PS- Я все еще учусь в старшей школе, и это мой первый пост здесь, поэтому извините, если я напортачил.

$\endgroup$

1

аналитическая геометрия — Найдите объем тетраэдра

спросил

Изменено 6 лет, 7 месяцев назад

Просмотрено 5к раз

$\begingroup$

Пусть

$A(2,1,3), B(3,2,5), C(3,3,6), D(4,4,2)$

Найдите объем $V$ тетраэдра $ABCD$.

Мое решение:

$\vec{AB} = (1, 1, 2), \vec{AC} = (1, 2, 3), \vec{AD} = (2, 3, -1) $

$$V=\begin{vmatrix}1&1&2\\1&2&3\\2&3&-1 \end{vmatrix}=-6$$

Мой вопрос: правильно ли мое решение и может ли объем быть отрицательным числом?

  • аналитическая геометрия

$\endgroup$

$\begingroup$

Обратите внимание, что объем тетраэдра = объем пирамиды с треугольным основанием = $$= 1/3\,\text{ * }\,\text{основание}\,\text{область}\,\text{ * }\,\text{высота}$$ Тогда, если $\vec a,\,\vec b,\,\vec c$ являются векторами, соответствующими трем совпадающим сторонам, вышеизложенное получается как $$ V = 1/3\влево| {\ left( {1/2 \, \ vec b \ times \, \ vec a} \right) \ cdot \ vec c} \ right | = 1/6\влево| {\;\ влево| {\начать{массив}{*{20}с} {c_x} и {c_y} и {c_z} \\ {b_x} & {b_y} & {b_z} \\ {a_x} & {a_y} & {a_z} \\ \end{массив} } \right|\;} \right| $$ Вы должны взять абсолютное значение, потому что $\vec b \times \,\vec a$ — вектор, ориентация которого относительно $\vec c$ будет определять положительный или отрицательный скалярный результат двойного произведения.

$\endgroup$

$\begingroup$

Объем трехмерного параллелепипеда со сторонами AB, AC и AD равен

$$\left|\det(AB, AC, AD)\right|.$$

Параллелепипед всегда в $6$ раз больше объемен как соответствующий тетраэдр $ABCD$, поэтому объем тетраэдра $ABCD$ равен

$$\frac{1}{6}\left|\det(AB, AC, AD)\right|.$ $

В данном случае это означает, что тетраэдр имеет объем ровно $1$.

$\endgroup$

$\begingroup$

Если вы сделали это, используя $B$ в качестве исходной точки (используя $BA, BC, BD$ в указанном порядке) ты бы нашел $$ V=\frac16 \влево| \pmatrix{-1&-1&-2\\0&1&1\\1&2&-3} \right| = \frac16 \cdot 6 $$ Определитель равен объему только с точностью до множителя $\pm 1$; вы можете видеть это, потому что, если вы измените порядок любых двух сторон, определитель изменит знак.

Решить задачу с помощью уравнения: Решение задач с помощью уравнений

Решение задач с помощью уравнений | План-конспект урока по математике (3 класс) на тему:

Технологическая карта урока

  1. Тема «Учимся решать задачи с помощью уравнений»
  2. Результаты  деятельности:

2.1. Личностные : — самостоятельно определять и высказывать самые простые общие для всех людей правила поведения при общении и сотрудничестве (этические нормы общения и сотрудничества).

2.2. Метапредметные : выполнять задания на основе рисунков и схем, выполненных самостоятельно;

— использовать (строить) таблицы, проверять по таблице.

2.3. Предметные: учиться, совместно с учителем, обнаруживать и формулировать учебную проблему;

— составлять план решения проблемы (задачи) совместно с учителем;

 — работая по плану, сверять свои действия с целью и, при необходимости, исправлять ошибки с помощью учителя;

 -научиться контролировать свою деятельность по ходу или результатам выполнения задания.

3.  Цель: провести исследование условия нескольких задач и найти новый способ решения

Задачи урока:

  1. Научиться переводить условие задачи с родного языка на язык математики.
  2. Научиться оформлять решение задачи уравнением.
  3. Стремиться к приобретению, запоминанию и применению новых знаний.
  4. Развивать мышление и логику.

Ход урока

Этап урока

/учебная ситуация

Результаты этапа

Цель этапа

Взаимодействие в образовательном процессе

Определяется в зависимости от планируемых результатов и содержания деятельности по их достижению

Система ценностных отношений

Сформированные УУД

Система предметных знаний (ЗУНы в предметной области), действия с предметным содержанием (опыт решения проблем, опыт творческой деятельности, ценностей)

Формирование и развитие системы ценностных отношений к ЛР, МПР и ПР и их достижению

Действия педагога

Действия обучающегося

Организует ситуации решения учебной задачи, направленной на достижение планируемых результатов, управление действиями обучающихся по достижению этих результатов

Решают учебную задачу, выполняют действия, направленные на достижение планируемых результатов и осознание ценности этих достижений

1.  Самоопределение к деятельности

Орг. момент

Формальная готовность к уроку.

Контроль, взаимоконтроль, самоконтроль Оценка, самооценка.

Формирование и развитие ЦО к умению оценивать готовность к предстоящей деятельности (на уроке)

СЛАЙД 1

– Прозвенел уже звонок, начинаем мы урок.

Сегодня на урок математики к нам пришли гости. Поприветствуйте их!

Проверьте готовность к уроку: учебник математики, рабочая тетрадь, пенал, дневник. Кроме принадлежностей нам что-то на уроке ещё понадобится? (знания…)

  Математика — наука трудная. Вы готовы преодолеть трудности на пути к знаниям? Тогда пожелайте друг другу удачи и у нас всё получится! -Я желаю вам хорошего настроения и успехов на уроке.

  • правильность выбора учебных принадлежностей,
  • правильность и аккуратность расположения предметов на парте,
  • самостоятельность подготовки,
  • настрой на урок.

2. Актуализация знаний, мотивация

Воспроизведенные знания и способы действия.

Осознание необходимости получения новых знаний

Воспроизведение знаний и способов действий необходимых и достаточных для открытия новых знаний

Определение границ знания и незнания

–Откройте тетради, запишите сегодняшнее число.

— Ребята, посмотрите на доску!

(505 – 5           Х * 5 = 40        340 * 30                808 + 8                 150 : 3) – работа в парах

— На какие группы можно разделить эти записи? Обсудите с соседом по парте (действия 1 и 2 ступени, и уравнение)

-Какую запись нельзя отнести ни к одной из групп? (уравнение)

— Что такое уравнение? (это равенство, которое содержит неизвестное)

— Ребята, а вы знаете, когда люди начали решать уравнения? Тогда заглянем в прошлое?

Уже 4000 лет тому назад египтяне решали разные задачи землемерия, строительства и военного дела с помощью уравнений. Уравнения умели решать в древности также китайские и индийские ученые. В музее изобразительных искусств в Москве хранится папирус, изготовленный из растений, на котором сделаны записи около 3500 тысяч лет назад.

— Как вы думаете, для чего нужны уравнения? (с их помощью можно легко найти неизвестное число)

-А в каких ещё математических заданиях мы сталкиваемся с неизвестными данными? (в задачах)

Слушают, читают задания, представляют результат, определяют цель и способ деятельности через понимание задание.

Выполняют задание.

Предъявляют результаты, оценивают полученные результаты, соотнося их с целью.

Формулируют свое затруднение (недостаточность знаний, способов действий). 

3. Сообщение темы урока. Постановка цели урока

Сформулированная и зафиксированная (вербально, с помощью схемы, модели…) учебная задача (самостоятельно  или в совместной деятельности с учителем).

Представления обучающихся о том, что нового они узнают на уроке, чему научатся.

Формирование и развитие ЦО к умению определять и формулировать проблему, познавательную цель и тему для изучения (самостоятельно  или в совместной деятельности с учителем).

СЛАЙД 2, 3

– Сегодня мы с вами будем решать задачи с помощью уравнений.

Тема сегодняшнего урока: «Решение задач с помощью уравнений».

Попробуйте сформулировать цель урока (научиться решать задачи с помощью уравнений)

Проговаривают проблему, отделяя знание от незнания.

Самостоятельно или с помощью учителя формулируют проблему, цель и тему для изучения и фиксируют учебную задачу, в соответствии, предполагают свои действия.

4. Изучение нового материала

Новое знание, понятие, правило, способ (алгоритм) деятельности, способ использования алгоритма

Формирование и развитие ЦО к совместной познавательной деятельности по открытию нового знания, понятия, правила, способа деятельности

-Итак, первой целью  нашего урока было-  научиться переводить текст задачи с русского языка на алгебраический. -Прочитайте задачу, которую можно решить уравнением.

СЛАЙД 4,5,6,7

-По таблице научимся переводить  текст задачи на язык алгебры……

-Какие слова помогли вам при решении задачи с помощью уравнения?

СЛАЙД 8

-Молодцы! Вот мы и узнали, как составлять уравнения к задаче. Будем тренироваться!  

Откройте учебники на странице 32, № 79.

— Прочитайте задачу.

— Авторы учебника предлагают нам сделать краткую запись в виде таблицы. Знакома вам такая работа? (Да)

-Начертите таблицу, запишите данные задачи в таблицу, проверьте соседа, продолжите работу в парах. (Дети составляют уравнение)

-Какое уравнение получилось?

 (уравнение Х * 9 = 54)

-Чему равен  корень уравнения? Какой ответ записали? — Хорошо! Молодцы!

Решают проблему, обсуждая и выдвигая гипотезы в совместной (парной, групповой) деятельности, сравнивают, анализируют, осуществляют поиск необходимой информации.

Моделируют, проговаривают правило, понятие, способ деятельности.

Определяют действия и их последовательность; строят и проговаривают алгоритм пошагово.

5.Динамическая пауза

6. Самостоятельная работа с самопроверкой в классе.

Умение применять  алгоритм, правило в самостоятельной деятельности

Формирование и развитие ЦО к закреплению знаний и способов действий.

— Кто понял, как при помощи уравнения можно решать задачи?

-Тогда попробуем усложнить задания, и составить задачу по уравнению. Можно в парах. (№ 80      9*х=36 )

— А теперь поработаем с диаграммой. (№83)

-Сформулируйте задачу, найдите корень уравнения с помощью диаграммы. (х=6)

Предлагаю вам выполнить задание № 49, т.с. 23 самостоятельно. (У доски ученик)

— Проверим, что у вас получилось.

-Сравните свою запись с записью на доске. Оцените работу.

-Молодцы! Вы справились с заданием!

Выполняют задания тетради на печатной основе самостоятельно.

Осуществляют самоконтроль и самооценку своей работы.

7. Обобщение усвоенного Оценка

Использование нового знания, правила, способа (алгоритма) деятельности с ранее в условиях решения задач и выполнения упражений

Формирование и развитие ЦО к обобщению знаний и способов действий, повторению ранее усвоенных ЗУНов.

Ребята, скажите, пожалуйста, для чего нам нужно уметь решать задачи? (на контрольной работе)

— А в жизни эти знания вам могут пригодиться? Где? (где угодно: дома, на улице, в школе, в магазине…)

— Например – Разложите 15 карандашей в 3 стаканчика поровну. Как это можно сделать? (раскладывая по одному или составить уравнение и решить)

Выполняют задания учителя

Придумывают свои задания по изученной теме.

8. Итог занятий. Рефлексия деятельности

Умение соотносить цели и результаты собственной деятельности

Формирование и развитие ЦО к умению анализировать и осмысливать свои достижения.

–Какую цель мы ставили перед собой?

-Мы достигли цели? Что особенно заинтересовало на уроке?

-Какая работа понравилась на уроке?

-Какая работа вызвала затруднение?

  —  Ребята, наклейте стикер на ту часть круга, которая соответствует вашему впечатлению от урока.

Участвуют в диалоге с учителем (организованной дискуссии).

Анализируют и осмысливают, проговаривают (фиксируют) свои достижения.

9. Домашнее задание

— Откройте дневники, запишите домашнее задание: Т. с.  24 № 50, 51

Выполняют задания учителя

класс «Решение задач на движение с помощью уравнения», 6 класс, УМК Н. Я. Виленкина

Мастер — класс

Решение задач на движение с помощью уравнения (6 класс)

Цель: Создание условий для передачи опыта по формированию умения у учащихся по решению задач на движение с помощью уравнения.

Задачи: 1. показать способ решения задач на движение с помощью уравнения;

2. оценить эффективность мастер – класса через рефлексию участников.

Форма проведения: урок — импровизация.

Оборудование: рабочие листы с заданиями, «Билет на выход» для проведения рефлексии.

Ход мастера -класса:

Если человека постоянно приучать усваивать знания и умения в готовом виде, можно и притупить его природные творческие способности — «разучить» думать самостоятельно. В максимальной степени процесс мышления проявляется и развивается при решении проблемных задач.

К сожалению, очень часто мы с вами не предоставляет свободы ученику, когда он пытается ответить на вопрос. Не ждём, а сразу же задаём наводящий вопрос. Можно ли учить так, чтобы каждый ребёнок рассуждал над проблемой своим путём, своим темпом, но при необходимости мог сопоставить свою точку зрения с одноклассниками, может даже изменить её? Конечно же, можно.

Помочь ученику раскрыться, лучше использовать свой творческий потенциал помогает создание проблемных ситуаций на уроке.

Проблемное обучение основывается на теоретических положениях американского философа, психолога, педагога Дж. Дьюи (1859-1952). В России дидактику проблемного обучения разработал И.Я. Лернер.

Сегодня под проблемным обучением понимается такая организация учебных занятий, которая предполагает создание под руководством учителя проблемных ситуаций и активную самостоятельную деятельность учащихся по их разрешению, в результате чего происходит творческое овладение профессиональными знаниями, навыками, умениями и развитие мыслительных способностей.

Как же создавать проблемные ситуации?

Вот проблемная ситуация на сегодня.

Эмблема урока: 28k + 30n + 31m = 365

Комментарий учителя к уравнению: Говорят, уравнение вызывает сомнение, но итогом сомнения может быть озарение!

Задание: Найти хотя бы одно решение уравнения.

(Уравнение, красочно оформленное, вывешивается сверху, в центре доски, к концу занятия будет найдено его решение)

Существует множество приёмов создания проблемных ситуаций. Вот некоторые из них:

Создание проблемных ситуаций через

умышленно допущенные учителем ошибки;

формулирование задания в занимательной форме;

выполнение практических заданий;

решение задач на внимание и сравнение;

противоречие нового материала старому, уже известному;

различные способы решения одной задачи;

выполнение небольших исследовательских заданий;

решение задач, связанных с жизнью.

    Участникам мастер класса предлагается выбрать задачи на движение из предложенного списка задач.

    Задание 1. Выберите задачу на движение и обоснуйте свой ответ.

    Задача №1.

    Лыжник прошел 900 м за 3 минуты, двигаясь с одинаковой скоростью. С какой скоростью двигался лыжник?

    Задача №2.

    Рабочий за 10 часов изготовил 300 деталей. Сколько деталей изготовит рабочий за 40 часов?

    Задача №3.

    Длина прямоугольника 6 м, а ширина в 3 раза меньше. Чему равен периметр и площадь прямоугольника?

    Задача №4.

    Биатлонист пробежал последний круг дистанции за 3 минуты со скоростью, равной 220 м/м. Чему равно данное расстояние?

    После выполнения задания предлагается вопрос:

    По каким признакам вы определили, что это задачи на движение?

    (Ответ: время, скорость, расстояние).

    Задача 

    Двое детей одновременно начали есть кашу. Через некоторое время первый ребенок кашу съел, а второй нет, хотя порции были одинаковые. Почему это произошло?

    (Ответ: Скорость первого ребенка больше, чем скорость второго).

    А эта задача на движение?

    Почему нет, ведь в ней присутствуют время и скорость?

    (Ответ: Нет такой величины как расстояние).

    Данный этап урока (актуализация знаний) помогает определить вид задачи, выделить ее существенные признаки. Но при этом учащимся предлагается задача, которая направлена на то, чтобы ребенок мог увидеть, что не всегда то, на что он привык опираться, ведет по верному пути. В данном случае есть скорость, время, но задача не на движение, так как отсутствуют другие величины.

     

    Задание № 2. Фронтальная работа

    Расстояние между двумя пунктами катер прошел по течению реки за 5 часов, а против течения — за 6 часов. Найдите расстояние между этими пунктами, если скорость течения реки 3 км/ч.

      1. К какому виду задач относится данная задача? (задача на движение)

        1.2. Какие величины характеризуют движение? (Ответ: время, скорость, расстояние).

        1.3. Построим таблицу

         

        Время (ч)

        Скорость (км/ч)

        Расстояние (км)

        по течению реки

        5

        х+3

        5(х+3)

        против течения

        6

        Х-3

        6(х-3)

        1. 4.В верхней строке занесем величины, характеризующие движение.

        1.5. Определим этапы движения. (по течению реки, против течения)

        1.6. Занесем этапы движения в 1-й столбик.

        1.7. Определим известную величину на каждом этапе (время) и занесем в таблицу.

        1.8. Определим величину, которую примем за х: собственная скорость катера. Тогда скорость по течению (х+3), а против течения (х-3).

        1.9. Заполнили два столбца, а третий заполним, исходя из правила нахождения расстояния.

        1.10 Что знаем про расстояние из условия задачи. (На обоих этапах пройдено одинаковое расстояние)

        1.11 Составим и решим уравнение.

        5(х+3)= 6(х-3)

        5х+15=6х-18

        х=33

        33 (км/ч) собственная скорость катера

        33-3=30(км/ч) скорость катера против течения

        30х6 -180 (км) прошёл катер

          Ответ: 180 км

          Половину пути мотоциклист ехал с намеченной скоростью 45 км /ч , затем задержался на 10 мин. , а поэтому , чтобы компенсировать потерянное время, он увеличил скорость на 15 км/ч. Каков весь путь мотоциклиста ?

            1. К какому виду задач относится данная задача? (задача на движение)

              2.2. Какие величины характеризуют движение? (Ответ: время, скорость, расстояние).

              2.3. Построим таблицу

               

              Время (ч)

              Скорость (км/ч)

              Расстояние (км)

              Первая половина пути

              х

              45

              45х

              Вторая половина пути

              Х-1/6

              45+15=60

              60(х-1/6)

              2. 4.В верхней строке занесем величины, характеризующие движение.

              2.5. Определим этапы движения. (Первая половина пути, вторая половина пути)

              2.6. Занесем этапы движения в 1-й столбик.

              2.7. Определим известную величину на каждом этапе (скорость) и занесем в таблицу.

              2.8. Определим величину, которую примем за х: время до увеличения скорости. 10 мин=1/6ч

              2.9. Заполнили два столбца, а третий заполним, исходя из правила нахождения расстояния.

              2.10 Что знаем про расстояние из условия задачи. (На обоих этапах пройдено одинаковое расстояние)

              2.11 Составим и решим уравнение.

              45х=60(х-1/6)

              45х=60х-10

              15х=10

              Х=2/3

              1)2/3 (ч) проехал мотоциклист первую половину пути

              2)45х2/3х2=60(км) путь

              Ответ: 60 км

              Задание № 3. Работа в группах

              Участники мастер-класса разбиваются на 6 групп и каждой группе предлагается решить задачи.

              Из пункта А в пункт В выехал велосипедист со скоростью 12км/ч. После того, как велосипедист проехал 3 км, из пункта А со скоростью 4 км /ч вышел пешеход, который пришёл в пункт В на 5/4 ч позже велосипедиста. Найдите расстояние между пунктами. (12х=4(х+5/4)+3)

              Расстояние между двумя пунктами катер прошел по течению реки за 7 часов, а против течения — за 8 часов. Найдите расстояние между этими пунктами, если скорость течения реки 2,5 км/ч.(7(х+2,5)=8(х+2,5))

              Турист 3 ч ехал на велосипеде, и 2 часа шел пешком, причем пешком он шел на 6 км/ч медленнее, чем ехал на велосипеде. С какой скоростью шел турист, если всего он преодолел 38 км? (2х+3(х+6)=38)

                4.Из двух пунктов реки на встречу друг другу движутся две моторные лодки, собственные скорости которых равны. До встречи лодка, идущая по течению, прошла1 ,1 ч., а лодка, идущая против течения, 1,5 часа. Найдите собственную скорость лодок, если лодка , идущая по течению по течению до встречи прошла на 1 км больше другой лодки . Скорость течения реки 3 км /ч .

                [1,1(х+3) – 1,5(х-3) =1]

                5.Из двух пунктов реки , расстояние между которыми 51 км , на встречу друг другу движутся две моторные лодки , собственные скорости которых равны . Скорость течения реки 3 км/ч. Лодка , идущая по течению , до встречи прошла 1,5 ч., а лодка , идущая против течения , 2 ч.Найдите собственную скорость лодок.

                [1,5(х+3) + 2(х-3) = 51]

                Из Москвы в Ростов – на – Дону вышел пассажирский поезд со скоростью 60 км/ч. Спустя 2 ч. 10 мин. Из Ростова- на- Дону в Москву вышел пассажирский поезд со скоростью 80 км/x . На коком расстоянии от Москвы поезда встретятся , если расстояние между городами считать равным 1250 км ?

                  [ 60х +80(х-21/6=1250]

                  Работа ведется маркерами на листах, листы вывешиваются.

                   

                  Вернемся к эмблеме занятия.

                  28k + 30n + 31m = 365

                  Слова учителя: Озарило?!

                  Ответ: 365 – это количество дней в году, 28 – количество дней в феврале, 30 – количество дней имеют 4 месяца в году, 31 – количество дней имеют 7 месяцев в году. Тогда: 28 ·1 + 30 · 4 + 31 · 7 = 365.

                  Меняется мир непрерывно, неспешно,

                  Меняется всё – от концепций до слов.

                  И тот лишь сумеет остаться успешным,

                  Кто сам вместе с миром меняться готов!

                  П. Калита

                  Рефлексия.

                  Участникам мастер – класса предлагается заполнить «Билет на выход».

                  Уважаемые участники мастер – класса, пожалуйста, выскажите свое мнение, закончив предложение.

                  Положительным моментом в данном мастер – классе является

                    __________________________________________________________________________________________________________________________

                    Я считаю, что такие приёмы работы

                      ____________________________________________________________________________________________________________________________________

                      3. Думаю надо продумать

                      ____________________________________________________________________________________________________________________________________

                      Мое настроение

                       


                       

                      Источники:

                      https://nsportal. ru/shkola/algebra/library/2015/12/03/metapredmet-problema-na-urokah-matematiki

                      https://yandex.ru/images/search?text=картинки

                      2.2.3: Использование уравнений для решения задач

                      1. Последнее обновление
                      2. Сохранить как PDF
                    1. Идентификатор страницы
                      38106
                      • Иллюстративная математика
                      • OpenUp Resources

                      Lesson

                      Давайте использовать уравнения для решения задач, связанных с пропорциональными отношениями.

                      Упражнение \(\PageIndex{1}\): Разговор о числах: Частные с десятичными знаками

                      Не вычисляя, расположите частные этих выражений в порядке от наименьшего к наибольшему.

                      \(42,6\дел 0,07\)

                      \(42,6\дел 70\)

                      \(42,6\дел 0,7\)

                      \(426\дел 70\)

                      Поместите запятую в соответствующее место в частном: \(42,6\дел 7=608571\)

                      Используйте этот ответ, чтобы найти частное на предыдущих выражений.

                      Упражнение \(\PageIndex{2}\): Продажа билетов на концерт

                      Исполнитель рассчитывает продать 5000 билетов на предстоящий концерт. Они хотят заработать на этих билетах в общей сложности 311 000 долларов.

                      1. Если предположить, что все билеты имеют одинаковую цену, какова цена одного билета?
                      2. Сколько они заработают, если продадут 7000 билетов?
                      3. Сколько они заработают, если продадут 10 000 билетов? 50 000? 120 000? миллион? \(х\) билетов?
                      4. Если они заработают 404 300 долларов, сколько билетов они продали?
                      5. Сколько билетов нужно продать, чтобы заработать 5 000 000 долларов?

                      Упражнение \(\PageIndex{3}\): Переработка

                      Алюминиевые банки можно перерабатывать, а не выбрасывать в мусор. Вес 10 алюминиевых банок составляет 0,16 килограмма. Алюминий в 10 переработанных банках стоит 0,14 доллара.

                      1. Если семья выбросила 2,4 кг алюминия за месяц, сколько банок они выбросили? Объясните или покажите свои рассуждения.
                      2. Какова будет переработанная стоимость тех же банок? Объясните или покажите свои рассуждения.
                      3. Напишите уравнение для представления количества банок \(c\) при заданном их весе \(w\).
                      4. Напишите уравнение для представления стоимости переработки \(r\) банок \(c\).
                      5. Напишите уравнение, представляющее рециклированную стоимость \(r\) \(w\) килограммов алюминия.

                      Готовы ли вы к большему?

                      По оценкам EPA, в 2013 году среднее количество мусора, производимого в США, составляло 4,4 фунта на человека в день. При таких темпах, сколько времени потребуется вашей семье, чтобы произвести тонну мусора? (Тонна равна 2000 фунтов.)

                      Резюме

                      Помните, что если существует пропорциональная связь между двумя величинами, их связь может быть представлена ​​уравнением вида \(y=kx\). Иногда написание уравнения является самым простым способом решения проблемы.

                      Например, мы знаем, что Денали, самая высокая горная вершина в Северной Америке, находится на высоте 20 300 футов над уровнем моря. Сколько это миль? В 1 миле 5280 футов. Это отношение может быть представлено уравнением

                      \(f=5,280m\)

                      где \(f\) представляет расстояние, измеренное в футах, а \(m\) представляет то же расстояние, измеренное в милях. Поскольку мы знаем, что Денали находится на высоте 20 310 футов над уровнем моря, мы можем записать

                      \(20 310=5 280 м\)

                      Итак \(m=\frac{20 310}{5 280}\), что составляет примерно 3,85 мили.

                      Практика

                      Упражнение \(\PageIndex{4}\)

                      Автомобиль движется по шоссе с постоянной скоростью, описываемой уравнением \(d=65t\), где \(d\) представляет собой расстояние , в милях, что автомобиль проедет с этой скоростью за \(t\) часов.

                      1. Что 65 говорит нам в этой ситуации?
                      2. Сколько миль проедет автомобиль за 1,5 часа?
                      3. За какое время автомобиль проедет 26 миль с такой скоростью?

                      Упражнение \(\PageIndex{5}\)

                      У Елены есть несколько бутылок с водой, каждая из которых содержит 17 унций жидкости.

                      1. Напишите уравнение, связывающее количество бутылок с водой (\(b\)) и общий объем воды (\(w\)) в жидких унциях.
                      2. Сколько воды в 51 бутылке?
                      3. Сколько бутылок потребуется, чтобы вместить 51 жидкую унцию воды?

                      Упражнение \(\PageIndex{6}\)

                      В 1 миле примерно 1,61 километра. Пусть \(x\) представляет собой расстояние, измеренное в километрах, а \(y\) представляет то же самое расстояние, измеренное в милях. Напишите два уравнения, связывающие расстояние, измеренное в километрах, и то же самое расстояние, измеренное в милях.

                      (Из модуля 2.2.2)

                      Упражнение \(\PageIndex{7}\)

                      В канадских монетах 16 четвертаков равны по стоимости 2 мультяшкам.

                      количество кварталов количество мультяшек
                      \(1\)
                      \(16\) \(2\)
                      \(20\)
                      \(24\)
                      Таблица \(\PageIndex{1}\)
                      1. Заполните таблицу.
                      2. Что означает значение рядом с 1 в этой ситуации?

                      (Из модуля 2.1.2)

                      Упражнение \(\PageIndex{8}\)

                      Каждая таблица представляет собой пропорциональную зависимость. Для каждой таблицы:

                      1. Заполните недостающие части таблицы.
                      2. Нарисуйте круг вокруг константы пропорциональности.
                      \(х\) \(у\)
                      \(2\) \(10\)
                      \(15\)
                      \(7\)
                      \(1\)
                      Таблица \(\PageIndex{2}\)
                      \(a\) \(б\)
                      \(12\) \(3\)
                      \(20\)
                      \(10\)
                      \(1\)
                      Таблица \(\PageIndex{3}\)
                      \(м\) \(н\)
                      \(5\) \(3\)
                      \(10\)
                      \(18\)
                      \(1\)
                      Таблица \(\PageIndex{4}\)

                      (Из модуля 2. 1.2)

                      Упражнение \(\PageIndex{9}\)

                      Опишите некоторые вещи, которые вы можете заметить в двух многоугольниках, которые могут вам помочь решить, что они не были масштабированными копиями.

                      (из блока 1.1.4)


                      Эта страница под названием 2.2.3: Использование уравнений для решения задач распространяется под лицензией CC BY и была создана, изменена и/или курирована Illustrative Mathematics.

                      1. Наверх
                        • Была ли эта статья полезной?
                        1. Тип изделия
                          Раздел или Страница
                          Автор
                          Иллюстративная математика
                          Лицензия
                          СС BY
                        2. Теги
                            На этой странице нет тегов.

                        задачи с тремя переменными.0034

                        www.tsfx.edu.au › ресурсы › N8 — ch04-3_50413

                        В большинстве задач, которые мы будем решать, плоскости пересекаются в одной точке, как на рис. 4.6(a). Множество решений состоит из одной упорядоченной тройки. Однако …

                        [PDF] 2.4 Системы с тремя переменными.notebook 1 08 октября 2019 г.

                        www.ddtwo.org › site › handlers › filedownload › FileName=2.4 Sys…

                        02.10.2019 · Цель: Решить системы трех уравнений с тремя переменными. График уравнения вида. Ax + By + Cz = D — плоскость, где A, …

                        [PDF] 8.2 Системы линейных уравнений с тремя переменными

                        packages.dadeschools.net › rvancol › Chapter8 › Ch8_Section2

                        Решайте задачи, используя системы с тремя переменными. Данная система (повторяется). c x + z = 8 x + y + 2z = 17.

                        [PDF] 3-5 Systems in Three Variables — Stafford High School

                        staffordhs. ss8.sharpschool.com › общие › страницы › UserFile

                        TEKS ( 3)(B) Решите системы трех линейных уравнений с тремя переменными, используя исключение Гаусса, технологию с матрицами и подстановку.

                        [PDF] Решение систем линейных уравнений с тремя переменными

                        www.alamo.edu › системы › math0303-система-линейных-уравнений

                        ПРИМЕЧАНИЕ. Не все задачи имеют упорядоченную тройку в качестве решения! ! Пример 1: Решите следующую систему уравнений. 4x + 8y + z = 2 x + 7y – 3z = – …

                        [PDF] Системы уравнений – 3 переменные

                        www.wallace.ccfaculty.org › 4.4 Системы с тремя переменными.pdf

                        In эту вышеописанную проблему у легко устранить с помощью метода сложения. Однако иногда нам, возможно, придется немного поработать, чтобы исключить переменную.

                        [PDF] Алгебра 2 — системные задачи Word и три уравнения

                        heiligenbergmath.weebly.com › загрузки › алгебра_2_-_системы__работа…

                        Рабочий лист от Kuta Software LLC. Алгебра 2 … Системы: текстовые задачи и три уравнения … В первый день продажи билетов школа продала 3.

                        [PDF] Решение систем текстовых задач. Лист для

                        www.hoodriver.k12. or.us › cms › lib06 › Centricity › Домен ›

                        Указания взяты из TAKS, так что выполняйте все три (переменные, уравнения и решение), независимо от того, что задано в задаче. 1. Большая пицца в пиццерии Palanzio …

                        Nutzer fragen auch

                        Как решить задачи с тремя переменными?

                        Что является примером 3 переменных?

                        Что такое формула с 3 переменными?

                        Как решить 3 переменные в 3 уравнениях?

                        Bilder

                        Alle anzeigen

                        Alle anzeigen

                        [PDF] Решение систем трех уравнений с исключением — Kuta Software

                        cdn.kutasoftware.com › Рабочие листы › Alg2 › Системы трех уравнений…

                        Рабочий лист от Kuta Software LLC. Kuta Software — Бесконечная Алгебра 2. Название_________________________________. Период____.

                        Правило возведения в степень степени: Возведение степени в степень — урок. Алгебра, 7 класс.

                        «Степень и её свойства»


                        База данных защищена авторским правом ©dogmon. org 2023
                        обратиться к администрации

                        Приложение 2.
                        Уровневое развивающее задание

                        по теме «Степень и её свойства»

                        (на развитие тезауруса, говорения,

                        письма, памяти и мышления).
                        Тезаурус, говорение, письмо, память и мышление.

                        Степень и её свойства.

                        Уровень С.
                        Задание: Перед Вами несколько вопросов. Ответьте на них сначала письменно, а затем устно составьте из ответов текст по теме «Степень и её свойства» и перескажите его партнеру.
                        Вопросы:


                        1. Сформулируйте определение степени числа с натуральным показателем. Что называют основанием и показателем степени (можно показать на примере)?

                        1. Каким числом является: а) степень положительного числа; б) степень отрицательного числа с чётным показателем; в) степень отрицательного числа с нечётным показателем?

                        1. Каким числом является квадрат любого числа? Ответ запишите в виде неравенства.

                        1. Сформулируйте основное свойство степени.

                        1. Сформулируйте правило умножения степеней с одинаковыми основаниями.

                        1. Сформулируйте правило деления степеней с одинаковыми основаниями.

                        1. Дайте определение степени числа с нулевым показателем.

                        1. Запишите в виде формулы и сформулируйте правило возведения в степень произведения.

                        1. Запишите в виде формулы и сформулируйте правило возведения в степень степени.

                        1. Докажите свойство степени произведения.

                        Ответы на вопросы:


                        1.

                        2.

                        3.

                        4.

                        5.

                        6.

                        7.

                        8.

                        9.

                        10.

                        Тезаурус, говорение, письмо, память и мышление.

                        Степень и её свойства.

                        Уровень В.
                        Задание: Перед Вами несколько вопросов. Ответьте на них сначала письменно, а затем устно составьте из ответов текст по теме «Степень и её свойства» и перескажите его партнеру.
                        Вопросы:


                        1. Сформулируйте определение степени числа с натуральным показателем. Что называют основанием и показателем степени (можно показать на примере)?

                        1. Каким числом является: а) степень положительного числа; б) степень отрицательного числа с чётным показателем; в) степень отрицательного числа с нечётным показателем?

                        1. Каким числом является квадрат любого числа? Ответ запишите в виде неравенства.

                        1. Сформулируйте основное свойство степени.

                        1. Сформулируйте правило умножения степеней с одинаковыми основаниями.

                        1. Сформулируйте правило деления степеней с одинаковыми основаниями.

                        1. Дайте определение степени числа с нулевым показателем.

                        1. Запишите в виде формулы и сформулируйте правило возведения в степень произведения.

                        1. Запишите в виде формулы и сформулируйте правило возведения в степень степени.

                        Ответы на вопросы:


                        1.

                        2.

                        3.

                        4.

                        5.

                        6.

                        7.

                        8.

                        9.

                        Тезаурус, говорение, письмо, память и мышление.

                        Степень и её свойства.

                        Уровень А.
                        Задание: Перед Вами несколько вопросов. Ответьте на них сначала письменно, а затем устно составьте из ответов текст по теме «Степень и её свойства» и перескажите его партнеру.
                        Вопросы:


                        1. Сформулируйте определение степени числа с натуральным показателем. Что называют основанием и показателем степени (можно показать на примере)?

                        1. Каким числом является: а) степень положительного числа; б) степень отрицательного числа с чётным показателем; в) степень отрицательного числа с нечётным показателем?

                        1. Каким числом является квадрат любого числа? Ответ запишите в виде неравенства.

                        1. Сформулируйте основное свойство степени.

                        1. Сформулируйте правило умножения степеней с одинаковыми основаниями.

                        1. Запишите в виде формулы правило деления степеней с одинаковыми основаниями.

                        1. Дайте определение степени числа с нулевым показателем.

                        1. Запишите в виде формул правило возведения в степень произведения и правило возведения в степень степени.

                        Ответы на вопросы:


                        1.

                        2.

                        3.

                        4.

                        5.

                        6.

                        7.

                        8.

                        Каталог: articles -> 212071
                        articles -> Соционика, ментология и психология личности  2003
                        articles -> Пояснительная записка к программе Пояснительная записка Актуальность программы
                        articles -> Приложение I теоретические аспекты изучения агрессии и ценностных ориентаций в психологической науке. Агрессия подростков как социальная проблема современного общества
                        articles -> Жизнь без агрессии реальность или утопия?
                        articles -> Печатается по решению кафедры
                        articles -> Особенности введения вновь нанятого работника в организацию и в должность
                        articles -> Особенности адаптации первоклассников к школьной жизни
                        212071 -> «Степень и её свойства»


                        Скачать 54.81 Kb.


                        Поделитесь с Вашими друзьями:

                        Силовое правило | иканхасмат

                      2. Потоковая передача + загрузка

                        Включает неограниченную потоковую передачу через бесплатное приложение Bandcamp, а также высококачественную загрузку в формате MP3, FLAC и других форматах.

                        Можно приобрести с подарочной картой

                        1 доллар доллар США или больше

                      3. Купить полный цифровой альбом
                      4. Комплект из трех компакт-дисков!!! И неограниченный доступ к Архиву!

                        Компакт-диск (CD) + цифровой альбом

                        Все наши альбомы! Исчисление: Мюзикл! C-сечения и журнал: ритм. Каждый с привлекательным полноцветным буклетом. Любимый кошками повсюду, возможно, собаками и трубкозубами, у нас нет данных, чтобы делать такие заявления. Купите их сейчас в виде одного удобного пакета, который включает цифровой доступ ко всем нашим альбомам, включая Be Rational, и любым будущим работам.

                        Включает неограниченную потоковую передачу иканхасматтеатр! Архив. через бесплатное приложение Bandcamp, а также высококачественную загрузку в форматах MP3, FLAC и других форматах.

                        Продано

                      5. Полная цифровая дискография

                        5

                        выпуска

                        Получите все 5 выпусков icanhasmath , доступных на Bandcamp, и сэкономьте 55% .

                        Включает неограниченное потоковое вещание через бесплатное приложение Bandcamp, а также высококачественные загрузки иканхасматтеатр! Архив., Исчисление: Мюзикл! 3-е издание., Журнал: Ритм, Будь рациональным, и кесарево сечение. , и , .

                        Можно приобрести с подарочной картой

                        $11,70 доллар США или больше (
                        СКИДКА 55% )
                      6. Когда у вас есть A, умноженное на X, на B,
                        вы знаете, что всегда используете:
                        Силовое правило.

                        Посмотрите на A B X на B минус один,
                        — это производная.
                        Силовое правило.

                        Производные констант всегда имеют наклон нуля.
                        Квадратный корень — это половина мощности,
                        вам нечего бояться.

                        О! Как можно проиграть!

                        За всеми многочленами можно забыть все свои беды,
                        , потому что все знают, что вы используете:

                        Силовое правило!
                        1 над X просто,
                        Силовое правило!
                        Х на минус 1.
                        Силовое правило!
                        Фо Полиномы, Фу!

                        Ла-ла-ла, ла-ла-ла-ла-ла-ла-ла-ла-ла-ла-ла-ла-ла.
                        Силовое правило!
                        Ла-ла-ла, ла-ла-ла-ла-ла-ла-ла-ла-ла-ла-ла-ла-ла.
                        Силовое правило!

                        Производные констант всегда имеют наклон нуля.
                        Квадратный корень — это половина мощности,
                        вам нечего бояться.
                        О, слоновья обувь.

                        За всеми полиномами можно забыть все свои беды,
                        , потому что все знают, что вы используете:

                        Силовое правило!
                        1 над X просто,
                        Силовое правило!
                        Х на минус 1.
                        Силовое правило!
                        Даны нет, только Зул!

                        Правило мощности

                        — Криста Кинг Математика | Онлайн-помощь по математике

                        Сообщения с тегами правила мощности Степенное правило для показателей для упрощения степеней степеней

                        Думайте об этом правиле как о правиле «власть силе». Другими словами, что происходит, когда мы возводим показательное выражение (основание, возведенное в некоторую степень) в другую степень (когда одно показательное выражение становится основанием другого показательного выражения)?

                        Читать далее

                        Изучайте математикуКриста Кинг математика, учитесь онлайн, онлайн-курс, онлайн-математика, предварительная алгебра, основы, основы математики, степенное правило, степенное правило для степеней, правила степеней

                        Использование законов логарифмов (законов бревен) для решения логарифмических задач

                        Законы логарифмов (или законы журналов) включают правила произведения, частного и степени для логарифмов, а также общее правило для журналов (и изменение основной формулы, которое мы рассмотрим в следующем уроке), все они могут быть использованы. вместе, в любой комбинации, чтобы решить проблемы с логами.

                        Читать далее

                        Учим математикуКриста Кинг математика, учиться онлайн, онлайн-курс, онлайн-математика, алгебра, алгебра 2, правило степени для логарифмов, правило частного для журналов, упрощение журналов, общее правило журнала, упрощение логарифмов, решение журналов, решение логарифмов, правило произведения для журналов, правило произведения, правило мощности, правило частного, правило произведения для логарифмов, правило частного для логарифмов, логарифмы, законы логарифмов, правило степени для журналов, законы журналов, алгебра II, журналы

                        Как использовать цепное правило и силовое правило вместе

                        Цепное правило часто является одной из самых сложных концепций для понимания изучающими математический анализ. Он также является одним из самых важных и используется постоянно, поэтому убедитесь, что вы не покидаете этот раздел без четкого понимания.

                      7. © 2015 - 2019 Муниципальное казённое общеобразовательное учреждение «Таловская средняя школа»

                        Карта сайта